1. Trang chủ
  2. » Mẫu Slide

Nhieu phuong phap chung minh bat dang thuc hay nhuSOS Tron bien ABC Chuyen vi GLA vv

205 11 0

Đang tải... (xem toàn văn)

Tài liệu hạn chế xem trước, để xem đầy đủ mời bạn chọn Tải xuống

THÔNG TIN TÀI LIỆU

Thông tin cơ bản

Định dạng
Số trang 205
Dung lượng 3,62 MB

Nội dung

3) Chuùng toâi nghó laø caùc baïn seõ ñoàng yù raèng: neáu moät baøi toaùn ñaõ chuaån hoùa (töùc laø BÑT coù ñieàu kieän) thì noù seõ "gôïi yù" cho chuùng ta caùch doàn bieán (ph[r]

(1)

DongPhD Problems BookSeries

Một Số Chuyên Đề Về Bất Đẳng Thức

Tổng hợp Chuyên đề Bất đẳng thức chia sẻ mạng

vnMath.com Dịch vụ Toán học dichvutoanhoc@gmail.com

Sách Đại số

Giải tích

Hình học

Các loại khác Thơng tin

bổ ích (Free)

Tốn học vui Kiếm

tiền mạng Bài báo

Giáo án (Free)

(2)

Contributors

Bùi Việt Anh Võ Quốc Bá Cẩn Nguyễn Anh Cường

Phạm Kim Hùng Phan Thành Nam

Võ Thành Văn Phan Thành Việt

Editors DongPhD

Ghi chú

Sách gồm nhiều phương pháp chứng minh bất đẳng thức hay, chẳng hạn ABC, GLA, SOS, pqr, Mixing variables, etc.

(3)

PHƯƠNG PHÁP DỒN BIẾN

Phan Thành Việt Nội dung:

1 Giới thiệu

2 BĐT biến với cực trị đạt đối xứng Dồn biến kĩ thuật hàm số

4 BĐT biến với cực trị đạt biên BĐT biến

6 Dồn biến hàm lồi

7 Dồn biến giá trị trung bình Định lý dồn biến tổng quát Nhìn lại

10 Bài tập 1 Giới thiệu.

Các bạn thân mến, nhiều số BĐT mà ta gặp có dấu đẳng thức biến số Một ví dụ kinh điển

Ví dụ 1: (BĐT Cauchy) Cho x, y, z >0 x+y+z ≥3√3xyz

Có thể nói số lượng BĐT nhiều nhiều bạn thấy điều là hiển nhiên Tất nhiên, không hẳn Tuy nhiên,

trường hợp đẳng thức không xảy tất biến ta lại thường rơi vào trường hợp khác, tổng quát hơn: có số (thay tất cả) biến Ở dẫn ví dụ chứng minh phần sau

Ví dụ 2: (VMO) Cho x, y, zR, x2+y2 +z2 = 9 Thì

2(x+y+z)−xyz≤10

Trong BĐT dấu "=" xảy x=y= 2, z =−1 (và hốn

(4)

Có thể nhiều bạn ngạc nhiên biết cịn có bất đẳng thức mà dấu "=" xảy biến khác Ví dụ sau chứng minh phần sau

Ví dụ 3: (Jackgarfukel) Cho a, b, c số thực không âm có tối đa

một số Thì ta có:

a

a+b+ b

b+c+ c

c+a

5

a+b+c

Ở đây, dấu đẳng thức xảy a= 3b >0, c = 0(và dạng hốn vị)

Các bạn tự hỏi giá trị chẳng hạn (3,1,0) có đặc biệt

mà làm cho đẳng thức xảy Một cách trực giác, ta thấy dường điểm đặc biệt có biến Vì giả thiết biến khơng âm, nên biến cịn gọi biến có giá trị biên

Tóm lại, BĐT mà ta gặp, có trường hợp dấu "=" xảy thường gặp: trường hợp tất biến (ta gọi "cực trị đạt tâm"), tổng quát trường hợp có số biến (ta gọi "cực trị đạt có tính đối xứng"), trường hợp khác dấu "=" xảy có biến có giá trị biên (và ta gọi "cực trị đạt biên")

Phương pháp dồn biến đặt để giải BĐT có dạng Ý tưởng chung là: ta đưa trường hợp có hai biến nhau, biến có giá trị biên, số biến giảm Do BĐT đơn giản BĐT ban đầu, đặc biệt BĐT biến cách khảo sát hàm biến số ta chứng minh BĐT đơn giản Chính tư tưởng giảm dần số biến nên phương pháp gọi phương pháp dồn biến

(5)

hết sức tổng quát Tinh thần xuyên suốt muốn bạn đọc cảm nhận tính tự nhiên vấn đề Qua đó, bạn lý giải "tại sao", để tự bước đường sáng tạo

*Ghi chú: Chúng đánh dấu tốn theo mục Vì số lượng định lý nên chúng tơi khơng đánh dấu Chúng tơi cố gắng ghi tên tác giả nguồn trích dẫn tất kết quan trọng, ngoại trừ kết

2 BĐT biến với cực trị đạt đối xứng.

Xin phác họa lại tư tưởng sau Bài tốn có dạng f(x, y, z)≥0với x, y, z biến số thực thỏa mãn tính chất

nào Điều mong muốn có đánh giá f(x, y, z)≥f(t, t, z)

với t đại lượng thích hợp tùy theo liên hệ x, y, z (ta

gọi kĩ thuật dồn biến nhau) Sau kiểm tra

f(t, t, z) ≥ để hoàn tất chứng minh Lưu ý biến

chuẩn hóa bước cuối toán với biến

Trong mục này, xem xét ví dụ Bài toán 1. (BĐT Cauchy) Cho x, y, z > 0, chứng minh

x+y+z ≥3√3xyz

Lời giải:

Vì BĐT đồng bậc nên cách chuẩn hóa ta giả sửx+y+z =

(*) Viết lại toán dạng f(x, y, z)≥0 với f(x, y, z) = 1−27xyz Ta

thấy thay xy t= x+2y điều kiện (*) bảo tồn (tức

vẫn có t+t+z= 1), nên ta phải xem xét thay đổi xyz

Theo BĐT Cauchy với biến (chứng minh đơn giản) xyt2,

nên xyzt2z Vaäyf(x, y, z)≥f(t, t, z)

Cuối để ý z = 1−2t nên ta có:

f(t, t, z) = 1−27t2z = 1−27t2(1−2t) = (1 + 6t)(1−3t)2 ≥0

và toán chứng minh xong Đẳng thức xảy x=y và3t= 1, nghĩa

(6)

*Nhận xét:

1) Có thể nhiều bạn bỡ ngỡ với cách chuẩn hóa Chúng tơi xin nói rõ: khơng có bí ẩn Nếu thích, bạn hồn tồn chuẩn hóa theo cách khác, chẳng hạn giả sử xyz = chứng minh

f(x, y, z)≥0vớif(x, y, z) =x+y+z−3 Khi bước dồn biến chứng minh f(x, y, z) ≥ f(t, t, z) với t = √xy Đề nghị bạn đọc tự lý giải

trong lời giải ta xét t = x+2y lại xét t = √xy, sau

hồn thành chứng minh theo cách

2) Bạn đọc thắc mắc: khơng cần chuẩn hóa khơng? Câu trả lời là: được! Thật vậy, hồn tồn xét toánf(x, y, z)≥0 với f(x, y, z) =x+y+z−3√xyz Khi bước dồn biến chứng minh f(x, y, z)≥f(t, t, z)với t = x+2y hay t=√xy Thực chất, điều

hồn tồn dễ hiểu, tương ứng BĐT có điều kiện BĐT khơng điều kiện (qua kĩ thuật chuẩn hóa)

3) Chúng nghĩ bạn đồng ý rằng: tốn chuẩn hóa (tức BĐT có điều kiện) "gợi ý" cho cách dồn biến (phải đảm bảo điều kiện), nhiên, ngược lại toán chưa chuẩn hóa (BĐT khơng điều kiện) có nhiều cách để dồn biến (nói chung, ta chọn cách dồn biến cho bảo toàn "nhiều" biểu thức BĐT - điều tương đương với chuẩn hóa cho biểu thức có dạng đơn giản nhất) Do đó, phối hợp tốt kĩ thuật chuẩn hóa dồn biến điều cần thiết Tuy nhiên, quen với điều bạn thấy khơng có khác biệt đáng kể chúng

Bài toán 2. (BĐT Schur) Cho a, b, c≥0, chứng minh rằng:

a3+b3+c3+ 3abca2(b+c) +b2(c+a) +c2(a+b).

Lời giải:

Xétf(a, b, c) =a3+b3+c3+ 3abca2(b+c)−b2(c+a)−c2(a+b) Đặt

t= b+2c, ta hi vọng: f(a, b, c)≥f(a, t, t) Xét

d=f(a, b, c)−f(a, t, t) = h

b+c−5 4a

i

(bc)2

Ta thấy với a, b, c số khơng âm tùy ý khơng có d ≥0 Tuy

(7)

chứng minh f(a, t, t)≥ Nhưng BĐT tương đương với a(at)2 ≥ 0

nên hiển nhiên Bài toán chứng minh xong

*Nhận xét: Việc giả sử a=min{a, b, c}là thủ thuật thường áp dụng để dồn biến Nhắc lại BĐT biến đối xứng ta giả sử

abc (hoặc abc), trường hợp BĐT biến hốn vị vịng

quanh ta giả sử a=min{a, b, c} (hoặca =max{a, b, c})

Bài toán 3. Cho a, b, c số thực dương có tích Chứng minh

raèng:

1

a +

1

b +

1

c +

6

a+b+c ≥5.

Hướng dẫn:

Nếu toán ban đầu toán quen thuộc, tốn khó Với kinh nghiệm thu từ tốn 1, nghĩ tới việc dồn biến theo trung bình nhân để khai thác giả thiết tích ba số Một lời giải theo hướng bạn Yptsoi (Đài Loan) đưa lên diễn đàn Mathlinks, mà sau xin dẫn lại cách vắn tắt

Ta chứng minh f(a, b, c) ≥ f(a,

bc,

bc) giả sử abc

Tiếp theo, ta chứng minh f(a,

bc,

bc)≥5, hay laø

f

x2, x, x

≥5, với x= √

bc

BĐT tương đương với (x−1)2(2x4 + 4x3 −4x2−x+ 2) ≥ 0 Vì biểu

thức ngoặc thứ hai dương với x > 0nên chứng minh hoàn tất Đẳng

thức xảy a=b=c=

Qua ví dụ trên, thấy cách dồn biến trung bình cộng trung bình nhân thật hữu dụng Tuy nhiên, cách dồn biến vô phong phú uyển chuyển Ví dụ sau minh họa cho điều Bài tốn 4.(Iran 1996) Chứng minh vớia, b, c >0thì:

(ab+bc+ca)

1

(a+b)2 +

1 (b+c)2 +

1 (c+a)2

4.

Hướng dẫn:

(8)

là dấu "=" đạt ngồi a=b=ccịn có a=b, c→0

Các bạn nên thử để thấy cách dồn biến thơng thường trung bình cộng trung bình nhân dẫn đến BĐT vô phức tạp Lời giải sau lấy từ ý thầy Trần Nam Dũng, mà nhìn kĩ bạn thấy mối tương quan, khơng tính tốn mà tư duy, kĩ thuật chuẩn hóa dồn biến, mà đề cập nhận xét 3) tốn

Vì BĐT đồng bậc nên ta giả sử ab+bc+ca = (*) Bây

ta hi vọng có đánh giá f(a, b, c) ≥

4 với f(a, b, c) biểu thức thứ hai

vế trái BĐT cần chứng minh Ở đâyt phải thỏa liên hệ (*), nghĩa t2+ 2tc=

Bằng cách giả sử c = min{a, b, c} ta chứng minh f(a, b, c) ≥

f(t, t, c) Cuối cùng, ta kiểm tra f(t, t, c) ≥

4 Ở bạn đọc thay c= 1−2tt2 vào BĐT để thấy:

f(t, t, c) = (1−t

2)(1−3t2)2

4t2(1 +t2) ≥0

Bài toán chứng minh xong!

*Nhận xét: Ở bước cuối, bạn khơng chuẩn hóa mà quay lại BĐT đồng bậc:

(t2+ 2tc)( (t+c)2 +

1 4t2)≥

9

⇔(t2+ 2tc)(8t2+ (t+c)2)−9(t+c)2t2 ≥0⇔2tc(tc)2 ≥0

Cuối đến với ví dụ mà cực trị khơng đạt tâm, BĐT đối xứng Các bạn thấy rằng, đường phần quan trọng dồn hai biến nhau, sau cực trị đạt tâm hay khơng khơng phải điều mấu chốt

Bài tốn 5. (VMO) Cho x, y, z số thực thỏa mãn: x2 +y2 +z2 =

Chứng minh rằng: 2(x+y+z)−xyz≤ 10

Lời giải.

Đặt f(x, y, z) = 2(x+y+z)−xyz Chúng ta hi vọng cóf(x, y, z)≥

f(x, t, t), đót2 = (y2+z2)/2 (*) (chúng nghĩ bạn đọc

(9)

định ý ta xét t ≥0

Ta coù: d = f(x, y, z)−f(x, t, t) = 2(y+z−2t)−x(yzt2) Ta thấy

ngay y+z−2t ≤0 vàyzt2≤0 Do để cód ≤0 ta cần x≤0

Từ đó, ta giả sử x = min{x, y, z} Xét trường hợp x ≤ Khi

ta dồn biến phải chứng minh f(x, t, t) ≤ 10 Thay

t=p(9−x2)/2 ta coù:

g(x) = f(x, t, t) = 2x+ 2p2(9−x2)−x(9−x2

)/2

Ta coù:

g0(x) = 3x

2

2 − −

4x

18−2x2

Giải ta thấy phương trình g0(x) = 0 có nghiệm âm làx=−1 Hơn

nữa g0liên tục vàg0(−2)>0> g(0) nên suy rag0 đổi dấu từ dương sang âm

khi qua điểm x =−1 Vậy ∀xg(x) ≤g(−1) = 10 ta có ñieàu

phải chứng minh Trường hợp đẳng thức đạt tạix=−1, y =z=

Phần lại ta phải giải trường hợp x >0, tức số x, y, z

dương Lúc dấu BĐT thực ta cần đánh giá đơn giản thông qua dồn biến Nếu x≥3/4

f(x, y, z) = 2(x+y+z)−xyz≤2p3(x2 +y2+z2)−(3

4)

3

= √

27−27 64 <10

Nếu x≤3/4

f(x, y, z) = 2(x+y+z)−xyz≤2(p2(y2+z2)+ 3/4) ≤= 2(√18 + 3/4) <10

Bài tốn chứng minh xong!

3 Dồn biến kó thuật hàm số.

(10)

Trong $2 thấy để chứng tỏ f(x, y, z) ≥ f(t, t, z) ta

việc xét hiệu d =f(x, y, z)−f(t, t, z)rồi tìm cách đánh giá cho d ≥0

Tuy nhiên, dạng BĐT đơn giản, phù hợp với biến đổi đại số Giả sử ta phải làm việc với biểu thức f có dạng, chẳng hạn, như: f(x, y, z) = xk +yk +zk với k > 0 cách biến đổi đại số trở nên

rất cồng kềnh phức tạp

Kĩ thuật hàm số dùng để giải trường hợp Ý tưởng này, chẳng hạn để chứng minh f(x, y, z) ≥ f(x, t, t) với t = (y+z)/2, ta xét hàm: g(s) =f(x, t+s, ts)vớis ≥0 Sau chứng minh

g tăng với s ≥ (thông thường dùng công cụ đạo hàm tiện lợi), suy

g(s) ≥ g(0), ∀s ≥ 0, ta thu điều mong muốn Một

ví dụ quen thuộc với bạn dồn biến hàm lồi, nhiên quan sát kĩ thuật dồn biến bối cảnh tổng quát hơn, vấn đề hàm lồi trở lại mục sau toán với n biến

Chúng nhấn mạnh rằng, kĩ thuật khó, chứa đựng nét tinh tế phương pháp dồn biến Những ví dụ sau thể rõ vẻ đẹp sức mạnh phương pháp dồn biến

Bài toán 1. Cho k > a, b, c số khơng âm có tối đa

số Chứng minh rằng:

( a

b+c)

k

+ ( b

c+a)

k

+ ( c

a+b)

k

min{2,

2k} (∗)

Lời giải:

Tất nhiên ta cần chứng minh BĐT = 23k ⇔ k = ln3

ln2 −1 (caùc

bạn suy nghĩ BĐT cho trường hợp lại dẫn đến BĐT cho trường hợp tổng quát) Chú ý với k đẳng thức xảy

ra hai chỗ a=b=c a =b, c= (và hoán vị)

Khơng tổng qt giả sử a+b+c = bca Đặt t = b+2cm = b−2c, suy b=t+m, c=tm, a= 1−2t Khi vế trái

BĐT cần chứng minh là:

f(m) =

1−2t

2t

k +

t+m

1−tm

k +

tm

1 +mt

k

ca nên 3t−1≥m≥0, 1≥b+c= 2t nên 12 ≥t

3

(11)

Ta coù:

f0(m) = k(t+m) k−1

(1−tm)k+1 −

k(tm)k−1 (1 +mt)k+1 f0(m)≥0⇔ (t+m)

k−1

(1−tm)k+1 ≥

(tm)k−1

(1 +mt)k+1

g(m) := [ln(tm)−ln(t+m)]− k+

1−k [ln(1−tm)−ln(1 +mt)]≥0

Tiếp tục khảo sát g, ta coù: g0(m) =−

tm +

1

t+m

+ k+ 1−k

1−tm +

1 +mt

≥0 ⇔ −2t

(tm)(t+m)+

k+ 1−k.

2(1−t)

(1−tm)(1 +mt) ≥0 (1)

Đánh giá k+1

1−k ≥2, để chứng minh (1) ta cần chứng minh ⇔ −t

t2−m2 +

2(1−t)

(1−t)2−m2 ≥0 (1)

u(m) =−t+ 4t2 −3t3+ 3tm2−2m2 ≥0

Thật vậy, u0(m)<0 nên u(m)≥ u(3t−1) = 2(3t−1)(2t−1)2 ≥0

Vậy g(m) đồng biến suy g(m) ≥ g(0) = suy f0(m) ≥ suy

f(m)≥f(0) Nhớ m = b=c=t

Cuối cùng, ta cần chứng minh h(t) :=f(0) ≥2 Viết lại:

h(t) =

1−2t

2t

k +

t

1−t

k

Ta khảo sát h(t)trên miền t∈[0,13] Ta coù:

h0(t) = 2kt k−1

(1−t)k+1 − k

2k.

(1−2t)k−1

tk+1 ≤0

⇔2k+1t2k ≤[(1−t)(1−2t)]k−1 (2)

Trong BĐT cuối, vế trái hàm đồng biến theot vế phải hàm nghịch

biến theo t, lưu ý t

3 nên để chứng minh (2) ta cần:

2k+1

1

2k

≤[(1− 3)(1−

2 3)]

(12)

Bất đẳng thức đúng, nên h(t) nghịch biến, suy

h(t)≥h(1 3) =

Bài toán giải trọn vẹn!

Nhận xét: Để thấy nét đẹp tốn này, chúng tơi xin dẫn số trường hợp riêng nó, thân chúng toán hay biết đến cách rộng rãi

1) Trường hợp k = 1, ta thu BĐT Netbit:

a b+c+

b

c+a +

c

a+b

3

Đây BĐT tiếng Một cách chứng minh "kinh điển" là:

a b+c +

b c+a +

c

a+b + =

a+b+c

b+c +

a+b+c

a+c +

a+b+c a+b

= (a+b+c)(

b+c+

1

c+a +

1

a+b)

≥(a+b+c)

(b+c) + (c+a) + (a+b) =

2) Trường hợp k = 12, ta thu BĐT sau: r

a b+c +

r

b c+a +

r

c a+b ≥2

Đây toán đẹp, trước biết đến BĐT ngược chiều với BĐT Netbit Có lời giải đơn gản, dùng BĐT

Cauchy: r

a b+c =

2a

2pa(b+c)

≥ 2a

a+b+c

3) Trường hợp k

3, ta có BĐT sau:

( a

b+c)

k

+ ( a

b+c)

k

+ ( a

b+c)

(13)

Đây toán đẹp biết đến từ trước mở rộng cho BĐT Netbit (nó đăng tạp chí THTT với tên tác giả Trần Tuấn Anh) Từ kết toán tổng quát, ta biết 2/3 số tốt để có giá trị nhỏ 3/2k Tuy nhiên, số tốt theo nghĩa áp dụng BĐT Cauchy theo cách sau Để đơn giản chúng tơi trình bày với trường hợp k = 2/3

a+b+c=a+b+c +

b+c

2 ≥3 r

a(b+c )

2

⇒( 2a

b+c)

2

3 ≥ 3a

a+b+c

Cùng với toán 1, toán sau ví dụ đẹp cho kĩ thuật hàm số

Bài toán 2. Cho k >0, a, b, c≥0 vàa+b+c= Chứng minh rằng: (ab)k+ (bc)k+ (ca)kmax{3,(3

2) k}

(∗)

Lời giải:

Khơng tổng qt giả sử bc (còn việc cho a = min hay max tùy theo tình huống, ta điều chỉnh cách "hợp lí" cần

thiết)

Đặtt = b+2cm= b−2c suy b=t+m, c=tm Khi vế trái BĐT

cần chứng minh trở thành:

f(m) =ak[(t+m)k+ (tm)k] + (t2−m2)k

Ta khảo sát f(m) miền m ∈[0, t] Ta có:

f0(m) =kak[(t+m)k−1−(tm)k−1]−2km(t2−m2)k−1

f0(m)≥0⇔g(m) :=ak[(tm)1−k −(t+m)1−k]−2m ≥0

Tất nhiên ta cần xét k > (khi k ≤ tốn đơn giản)

Ta coù:

(14)

g0(m) đồng biến, có tối đa nghiệm (0, t) Vì g(0) =

0, g(t) = +∞ nên có hai khả năng:

g(m)>0 g(m) =− +

Tương ứng ta cóf(m)đi lên hoặcf(m)đi xuống lại lên Trong trường

hợp cực đại đạt biên

f(m)≤max{f(0), f(t)}

Nhắc lại m = 0⇔b=c=tm=tc=

Dễ thấy c= thì:

f(t) = 2(ab)k

3

2k

nên ta phải xét trường hợp lại Đặt:

h(t) :=f(0) = 2tkak+t2k = 2tk(3−2t)k +t2k

Ta coù:

h0(t) = −4k(3−2t)k−1tk+ 2k(3−2t)kbk−1+ 2kb2k−1 h0(t)≥0⇔ −2

3−2t

t

k−1

+

3−2t t

k

+ ≥0 ⇔u(x) :=xk −2xk−1+ ≥0 với x= 3−2t

t

Ta coù: u0(x) = [kx−2(k−1)]xk−2.u0(x) có tối đa nghiệm R+

nên u(x) có tối đa2 nghiệm R+, nghiệm x= 1

Từ đó, ta giả sử a=min{a, b, c} Khi ta việc xét t ≥1

tương ứng x≤1 Vìu(x) có tối đa nghiệm (0,1) nên h0(t)

chỉ có tối đa nghiệm (1,32)

Lưu ý lưu ýh0(1) = 0, h0(3

2)>0 Do đó, có hai khả hoặch(t)

đồng biến h(t) có dạng−0+ Trong trường hợp thìh(t)cũng đạt

max hai biên, suy ra:

h(t)≤max{f(1), f(3

2)}=max{3,( 2)

2k}

(15)

*Nhận xét: Ở không giả thiết a=min{a, b, c}ngay từ đầu

muốn nhấn mạnh rằng: việc dồn biến thực mà không cần thứ tự biến Tận dụng điều đó, làm cách khác để né việc khảo sát toán biến

Thật vậy, chứng minh ra, ta ln có BĐT sau mà khơng cần giả thiết thứ tự a, b, c:

f(a, b, c)≤ max{(3 2)

2k

, f(a,b+c

2 ,

b+c

2 )} (∗)

Từ đó, với a, b, c cố định, xét dãy số sau: (a0, b0, c0) = (a, b, c),

nZ+ ta định nghóa quy nạp:

(a2n−1, b2n−1, c2n−1) = (a2n−2,

b2n−2+b2n−2

2 ,

b2n−2+b2n−2

2 )

vaø:

(a2n, b2n, c2n) = (

a2n−1+b2n−1

2 ,

a2n−1+b2n−1

2 , c2n−1)

thì ta có

f(a, b, c)≤max{(3 2)

2k

, f(an, bn, cn)},nZ+

Dễ thấy dãy {an}, {bn}, {bn} hội tụ 1, nên chuyển qua giới hạn

ta có điều phải chứng minh

Kĩ thuật chuyển qua giới hạn tự nhiên Nó tổng quát lên thành định lý dồn biến tổng quát SMV UMV mà giới thiệu phần sau Cũng sử dụng tính liên tục hàm số với kĩ thuật khác, chúng tơi cịn đạt kết tổng quát

Sau có (*), cịn cách khác để đạt điều phải chứng minh mà cần sử dụng số hữu hạn lần thay Tuy nhiên, để khỏi trùng lắp chúng tơi giới thiệu mục BĐT biến (và mục sau), mà thực cần thiết

h Còn trường hợp biến, sử dụng cách tiếp cận đơn giản (dồn biến khảo sát), nhằm giữ tính sáng tư tưởng

(16)

thiết dồn trung bình cộng Sau ví dụ cho kiểu dồn biến trung bình nhân

Bài tốn 3: (Phạm Kim Hùng)

a)Cho số thực dương a, b, c có tích Chứng minh rằng: (i) 81(1 +a2)(1 +b2)(1 +c2)≤8(a+b+c)4

(ii) 64(1 +a3)(1 +b3)(1 +c3)≤(a+b+c)6

Lời giải:

(i) Đặt f(a, b, c) = 8(a+b+c)4−81(1 +a2)(1 +b2)(1 +c2) Ta giả

sử ab Xét hàm số g(t) =f(ta, b/t, c)với t ∈[pb/a,1] Ta có:

g0(t) = 32(ab

t2)(ta+ b t +c)

3−

81(ab

t2)(ta+ b

t)(1 +c

)

t ∈[pb/a,1]nên g0(t)≥0nếu:

32(d+c)3 ≥81d(1 +c2) với d=ta+ b

t

Ta coù: 32(d+c)3 >32d(d2+ 2dc+ 3c2)≥32d(3 √

d4c2+ 3c2

)>81d(1 +c2)

(lưu ý d2c≥4)

Vậy g0(t)≥0vớit ∈[pb/a,1] Do đó: g(1)≥g(pb/a) Vậy f(a, b, c)≥

f(s, s, c)với s= √

ab Thays = 1/cta được: f(s, s, c) =f(√1

c,

1 √

c, c) = 8(

2 √

c+c) 4−

81(1 +

c)

(1 +c2) = (

c−1

c )

2

(8c5+ 16c92 + 24c4+ 96c

2 + 87c3+ 78c 2+ +99c2+ 120c32 −21c+ 94

c+ 47)

≥0 (ñpcm)

Đẳng thức xảy a=b=c=

(ii) Bằng cách làm tương tự trên, bạn đọc tự chứng minh BĐT

(17)

tốn phức tạp Rồi thành cơng trước tốn khiến bạn tự tin Chúng dẫn tốn mà lời giải khiến nhiều bạn "khiếp sợ", nhiên chúng tơi hi vọng bạn bình tâm để thấy vẻ đẹp sáng ẩn đằng sau kĩ thuật tính tốn lão luyện

Bài toán 4. Cho a, b, c ≥ 0, a +b +c = Tìm giá trị lớn

biểu thức:

ab

3 +c2 + bc

3 +a2 + ca

3 +b2

Lời giải:

Lời giải sau anh Phan Thành Nam

Giả sử abc Đặt a = s+t, b = st vế trái BĐT cần chứng

minh laø:

f(t) := c(st) + (s+t)2 +

c(s+t) + (st)2 +

s2−t2

3 +c2

Ta khảo sát f(t) miền t ∈[0, sc] Ta coù:

f0(t) = −c

3 + (s+t)2 −

2c(s2 −t2)

(3 + (s+t)2)2 +

c

3 + (st)2 +

2c(s2−t2)

(3 + (st)2)2 −

2t

3 +c2

= 4cst

uv +

8cst(s2−t2)(u+v)

u2v2 −

2t

3 +c2 <0,t ∈(0, sc) (∗)

với u= + (s+t)2, v= + (st)2 (BĐT (*) chứng minh sau)

Vaäy ∀t ∈[0, sc] thì:

f(t)≤f(0) = 2cs +s2 +

s2

3 +c2 =

2s(3−2s) +s2 +

s2

3 + (3−2s)2 =:g(s) (1)

Xét g(s) với s∈[1,32] Ta có:

g0(s) = 24s−12s

2

(3 + (3−2s)2)2+

18−24s−6s2

(3 +s2)2 =

108(s2 −3s+ 4)(s−1)2(−s2−3s+ 6)

[3 + (3−2s)2]2[3 + s2]2

Dễ thấy s2−3s+ 4>0và−s2−3s+ = ( √

33−3

2 −s)(s+ √

33+3

2 ) neân g 0(s)

dương (1, s0) âm (s0,32) với s0 := √

33−3

2 = 1,372281323

Vaäy ∀s ∈[1,3 2] thì:

g(s)≤g(s0) =

11 √

33−45

(18)

Trong (1) (2), dấu "=" xảy đồng thời t = s = s0, tức a=b=s0 vàc= 3−2s0

Vậy giá trị lớn cần tìm 11√33−45

24 = 0,757924546 , đạt

a=b=

√ 33−3

2 = 1,372281323 …, c= 6−

33 = 0,255437353

Để kết thúc, ta chứng minh BĐT (*) Đây BĐT chặt Ta với t∈(0, sc) thì:

4cs uv <

1

3 +c2 (3) vaø

8cs(s2−t2)(u+v)

u2v2 ≤

1

3 +c2 (4)

là xong!

Chứng minh (3): Vì c+ 2s = s > nên cs < Hơn u = + (s+t)2 >4, v= + (st)2 >3 +c2 Từ suy ra(3)

Chứng minh (4): Dùng BĐT Cauchy ta có:

u2v2 = [[3 + (s + t)]2[3 + (s - t)]2]2 ≥16(s2 −t2),vaø

2cs(u+v)(3 +c2) = 4cs(3 +s2+t2)(3 +c2)≤

4cs+ +s2+t2+ +c2

3

3

Thay c= 3−2s vào, lưu ý tsc= 3s−3, ta có:

4cs+ +s2+t2+ +c2 ≤4(3−2s)s+ +s2+ (3s−3)2+ (3−2s)2 = 12 + 6(s−1)(s−2) ≤12

suy 2cs(u+v)(3 +c2)<43 Vaäy: 8cs(s2−t2)(u+v)

u2v2 = 4.

s2−t2 u2v2 .

2cs(u+v)(3 +c2)

3 +c2 ≤4.

1 44.

43

3 +c2 =

1 +c2

và toán giải xong!

4 BĐT biến với cực trị đạt biên.

Nếu phần trước hiểu "dồn biến" "đẩy hai biến lại gần nhau", trường hợp ta phải hiểu "dồn biến" nghĩa "đẩy biến biên" Chẳng hạn xét BĐT f(x, y, z) ≥

với x, y, z ≥ 0, ta hi vọng vào đánh giá f(x, y, z) ≥ f(0, s, t),

(19)

là kĩ thuật dồn biến biên) Tất nhiên ta chọn s, t cho hiệu d = f(x, y, z) ≥ f(0, s, t) đơn giản đánh giá thuận lợi Cuối

cùng ta việc kiểm chứng f(0, s, t)≥0

Trước hết, để bạn làm quen với cách dồn biến "mới mẻ" này, xin trở lại ví dụ phần trước

Bài tốn 1: (BĐT Schur) Cho a, b, c≥0 Chứng minh rằng:

a3+b3+c3+ 3abca2(b+c) +b2(c+a) +c2(a+b).

Lời giải:

Trong$2, giải cách dồn biến Tuy

nhiên nhận xét ngồi điểm a =b=c, đẳng thức cịn đạt a=b, c=

(và hốn vị) Do đó, kĩ thuật dồn biến biên có khả thành cơng!

Đặt f(a, b, c) = a3+b3+c3+ 3abca2(b+c)−b2(c+a)−c2(a+b)

Ta hi vọng có f(a, b, c)≥f(0, a+b, c) Xét hiệu:

d=f(a, b, c)−f(0, a+b, c) =ab(5c−4a−4b)

Như ta khơng thể có d ≥0, cho dù tận dụng kiện a, b, c

thể

Thật đáng tiếc! Tuy nhiên, bạn dừng lại cịn đáng tiếc Thay bỏ dỡ, ta xem lại khơng thể có d ≥ Nếu

tinh ý, bạn thấy f(a, b, c) nhỏ hai biến tiến lại gần

nhau (đó lý mà ta dồn hai biến

$2), thay bộ(a, b, c)bởi (0, a+b, c)thì "dường như" biến

càng cách xa Đó lý cách dồn biến thất bại Từ đó, ta nảy ý thay (a, b, c)bởi (0, b+a/2, c+a/2) Xét hiệu:

da=f(a, b, c)−f(0, b+a/2, c+a/2) =a(a+b−2c)(a+c−2b)

Điều thú vị ta giả sửda ≥0 Thật vậy, điều nhờ việc

thứ tự biến a, b, cmà hiệuda, db, dc

(trong đódb, dc hai hiệu tương tự nhưda) Vì tính đối xứng nên ta

giả sử da =max{da, db, dc} Khi da <0

(20)

và mâu thuẫn!

Vậy da ≥ nên f(a, b, c)≥ f(0, s, t) với s =b+a/2, t =c+a/2 Cuối

cùng, ta thấy

f(0, s, t) =t3+s3−t2sts2 = (t+s)(ts)2 ≥0

và chứng minh hoàn tất

*Nhận xét: Mặc dù BĐT Schur quen thuộc, cách chứng minh dồn biến ý gần Tuy nhiên, cách dồn hai biến "hợp lý", cách dồn biến biên kết thực bất ngờ Tất nhiên, chứng minh cách ngắn gọn nhất, chúng tơi muốn nhấn mạnh đến tự nhiên Nếu toán việc áp dụng kĩ thuật dồn biến biên gây bất ngờ, tốn sau đường tất yếu

Bài toán 2: (Hojoo Lee) Cho a, b, c ≥ 0, ab +bc + ca = (*) Chứng

minh raèng:

1

a+b +

1

b+c +

1

c+a

5

Lời giải:

Bài đẳng thức không xảy tâm, mà a =b= 1, c =

hoán vị Xét trường hợp riêng c= 0, tốn trở thành: "Chứng minh rằng:

a +

1

b +

1

a+b

5

2,với ab= 1.

Đặts=a+bthì điều tương đương vớis+1s

2, hay(2s−1)(s−2)≥

0 BĐT cuối hiển nhiên s=a+b≥2 √

ab=

Vậy ta cần dồn biến xong Cách làm sau lấy từ ý anh Phạm Kim Hùng Diễn Đàn Toán Học

Đặt f(a, b, c) vế trái BĐT cần chứng minh Ta hi vọng f(a, b, c) ≥

(21)

hieäu:

d=f(a, b, c)−f(a+b, a+b,0)

=

a+b+

1

a+ 1a−+abb +

b+1a−+abb !

a+b+a+b+

1

a+b+ a+1b !

.

=

1 +a2 +

1

1 +b2 −1−

1 + (a+b)2

Từ quy đồng lên ta thấy d ≥ 2(1−ab) ≥ ab(a+b)2 Nếu giả sử

c =max{a, b, c} 2(1−ab) = 2c(a+b) ≥ ab(a+b)2 Vậy lúc d ≥0

và toán chứng minh xong! *Nhận xét:

1) Lời giải đầy tiên đưa Diễn Đàn Toán Học anh Phan Thành Nam, cách chứng minh ngắn gọn Đặt x=a+b+c

Neáu x≥2 thì:

a+b =c+ ab

a+bc+ ab

a+b+cf(a, b, c)≥ x+

1

x

5 Nếu x≤2 giả sử a=max{a, b, c}ta có:

f(a, b, c) = (c+ ab

a+b) + (b+ ac a+c) +

1

b+c

= (b+c+

b+c) +

a(1 +bc)

ax+bc ≥2 +

1 =

5 (lưu ý 2a(1 +bc) = 2a+ 2abcax+bc, x≤2và 2a≥1 )

Tuy nhiên, lời giải dễ dàng nghĩ Về lời giải dồn biến trên, lần chúng tơi nhấn mạnh đến tính tự nhiên

2) Bài tốn tốn hay thu quan tâm nhiều bạn Tuy nhiên, bạn bất ngờ hệ đơn giản BĐT quen thuộc khác Đó BĐT Iran 1996 Thật vậy, với giả thiết

ab+bc+ca= từ kết BĐT Iran 1996 ta có ngay: (

a+b+

1

b+c+

1

c+a)

=

(a+b)2 +

1 (b+c)2 +

1 (c+a)2 +

4(a+b+c)

(a+b)(b+c)(c+a) ≥

4 + = 25

(22)

(lưu ý a+b+c= (a+b+c)(ab+bc+ca)≥(a+b)(b+c)(c+a)) Từ nhận xét trên, ta nhớ lại $2, BĐT Iran 1996 giải kĩ thuật dồn hai biến Từ có hai câu hỏi tự nhiên là, thứ nhất: tốn giải cách dồn hai biến không, thứ hai: BĐT Iran 1996 giải cách dồn biến biên không? Chúng đề nghị bạn tự giải đáp hai câu hỏi

3) Bài toán lại dẫn đến kết thú vị sau đây, mà tác giả bạn Zhao bin (Trung Quốc)

"Cho x, y, z số thực không âm có tối đa số Chứng

minh raèng:

1

x2+y2 +

1

y2+z2 +

1

z2+x2 ≥

10

(x+y+z)2.

Bằng hai toán "cũ" trên, chúng tơi muốn bạn đọc có cảm giác dễ dàng kĩ thuật dồn biến biên Tuy nhiên, hai kĩ thuật dồn bai biến phát huy tác dụng, khơng khỏi khó khăn việc thuyết phục bạn đọc sức mạnh kĩ thuật dồn biến biên Do đó, chúng tơi dẫn tốn sau đây, bạn thấy kĩ thuật dồn hai biến hồn tồn bế tắc, đơn giản đẳng thức đạt biến đôi khác Đây

trong ví dụ quan trọng kĩ thuật dồn biến biên mà chúng tơi muốn trình bày với bạn

Bài toán 3: (Jackgarfukel) Cho a, b, c số thực khơng âm có tối

đa số Chứng minh rằng:

a

a+b + b

b+c + c

c+a

5

a+b+c (∗)

Lời giải:

Trước công này, ta cần xem trường hợp dấu xảy ra: dễ thấy a=b=ckhông thỏa, cách tự nhiên ta nghĩ đến

trường hợp biên: c= Với c= BĐT(*) trở thành

a

a+b+

b

(23)

Chuẩn hóa a+b= Ta có (1)⇔1−b+

b≤ ⇔(

b−1/2)2 ≥0 (đúng!)

Vậy đẳng thức xảy a= 3b, c= (và hoán vị)

Như trường hợp dấu xảy ba biến rời nhau, phương pháp dồn hai biến xem khơng cịn tác dụng Do đó, dồn biến biên xem đường tất yếu

Khơng tổng qt giả sử a = max{a, b, c} a+b+c =

Đặt t= a+2c s= a−2c, suy raa =t+s, c=ts, b= 1−2t Ta coù

(∗)⇔ √ t+s

s+ 1−t +

1−2t

1−ts + ts

√ 2t

5 (1)

Đặtf(s) =V T(1)vớis ∈[0, t], ta chứng minhf(s)≤max(f(0), f(t))

Ta coù :

f0(s) = √

s+ 1−t

t+s

2(s+ 1−t)3/2 +

1−2t

2(1−ts)3/2 −

1 √

2t

Vì chưa xác định dấu f0(s) nên ta đạo hàm tiếp

f00(s) =−

(s+ 1−t)3/2 +

3(t+s) 4(s+ 1−t)5/2 +

3(1−2t) 4(1−ts)5/2 f000(s) =

4(s+ 1−t)5/2 −

15(t+s) 8(s+ 1−t)7/2 +

15(1−2t) 8(1−ts)7/2

= 18 + 3s−33t (1−ts)7/2 +

15(1−2t)

8(1−ts)7/2 >0, vìb= 1−2t ≥0

Vậy f000(s) > với s ∈ [0, t] nên theo định lí Rolle ruy f0(s) có

tối đa đa hai nghiệm [0, t] Mặt khác dễ dàng chứng minh f0(0) ≤

f0(t) ≥ 0 f0(s) đổi dấu tối đa lần (0, t),

nữa f0(s) có dạng sau: f0(s)> 0,s ∈ (0, t)

f0(s)<0,s∈(0, t) f0(s) có dạng − 0 + (0, t). Tuy nhiên

trường hợp f(s)cũng đạt cực đại biên

Vậy f(s)≤max(f(0), f(t))với mọis∈[0, t]nên ta cần chứng minh

BĐT sau xong:

(24)

Muốn ta chứng minh BĐT f(0) ≤ 5/4 f(t) ≤ 5/4

Việc chứng minh hai BĐT dễ dàng, nên đề nghị bạn đọc tự kiểm chứng

Hẳn nhiên bạn đồng ý cần thiết phương pháp dồn biến biên toán Tuy nhiên, nhiều bạn cho rằng: tốn không đối xứng nên không xảy trường hợp dấu "=" có hai biến Để phủ định nhận xét đó, chúng tơi kết thúc mục cách dẫn toán anh Phạm Kim Hùng THTT: Bài toán 4. Cho a, b, c≥0, a+b+c= Chứng minh rằng:

(a3+b3+c3)(a3b3+b3c3+c3a3)≤ 36(ab+bc+ca)

Lời giải:

Khơng tổng qt giả sử abc Đặt

f(a, b, c) = 36(ab+bc+ca)−(a3+b3+c3)(a3b3+b3+c3+c3a3)

Khi f(a, b+c,0) = 36a(b+c)−(a3+ (b+c)3)a3(b+c)3

Ta chứng minh f(a, b, c)≥f(a, b+c,0).Thật vậy, ý rằng:

36(ab+bc+ca)≥36a(b+c)

vaø

(a3+b3+c3)(a3b3+b3c3+c3a3)≤[a3+ (b+c)3]a3(b+c)3

(vì ta có a3+b3+c3 ≤a3+ (b+c)3 vàa3b3+b3c3+c3a3 ≤a3(b+c)3)

Do ta cần chứng minh tốn trường hợp c= 0, hay

36aba3b3(a3+b3)⇔36 ≥a2b2(a3 +b3)

Đặt t =ab, bất đẳng thức viết lại dạng t2(27−9t)≤ 36 ⇔ t3+ 4 ≥ 3t2 Nhưng lại BĐT Cauchy ba số t3/2, t3/2,4 Đẳng

thức xảy c = a+b= 3, ab = hay a = 2, b = 1, c = (và

(25)

*Nhận xét: Một ví dụ nữa, đơn giản hơn, tác giả Diễn Đàn Toán Học:

" Cho a, b, c≥0, a+b+c= Tìm giá trị lớn của: (a2 −ab+b2)(b2 −bc+c2)(c2−ca+a2).” Bài tốn khơng khó đề nghị bạn đọc tự giải 5 BĐT biến.

Sau nắm vững kĩ thuật dồn biến với số bạn đọc mục cách nhanh chóng Chúng tơi xin lưu ý đặc thù trường hợp biến: Khi có biến ta dồn biến theo cặp, chứng minh toán (chẳng hạn BĐT Cauchy) Tuy nhiên, thuận lợi thường xuất toán đơn giản Đối với phức tạp thường ta dồn cặp nhờ thứ tự biến Sau dồn hai biến (hoặc dồn biến biên) ta chưa có BĐT với biến, mà phải qua BĐT trung gian (2 hay biến) Tuy nhiên thường BĐT trung gian dễ để chứng minh trực tiếp đánh giá để quy biến Nói chung, chúng tơi nhấn mạnh điều cần thiết bạn cần quan sát thật kĩ mối liên hệ biến để có cách xử lý thích hợp

Chúng ta bắt đầu với ví dụ "kinh điển" cho kĩ thuật dồn biến với BĐT biến

Bài toán 1. (IMO SL, Việt Nam đề nghị) Choa, b, c, d≥0, a+b+c+d=

Chứng minh rằng:

abc+bcd+cda+dab≤ 27 +

176 27 abcd

Lời giải:

Bài đẳng thức xảy a = b = c = d = 1/4 a = b = c = 1/3, c = Do đó, đánh giá thông thường dễ rơi vào bế tắc

Đặt f(a, b, c, d) = abc+bcd+cda+dabkabcd với k= 176

(26)

Từ đó, ta hi vọng có f(a, b, c, d)≤f(t, t, c, d)vớit= a+2b Vì0≤abt2

nên để có điều ta cần c+dkcd ≥ Ở may mắn có

điều có điều ngược lại, nghĩa c+dkcd <0, BĐT ban đầu hiển

nhiên vì:

f(a, b, c, d) =ab(c+dkcd)+cd(a+b)≤cd(a+b)≤(c+d+ (a+b)

3 )

3

= 27

Vậy ta giả sử ln có f(a, b, c, d)≤f(a+2b,a+2b, c, d) Lưu ý ta

đã thực việc dồn biến mà không cần giả thiết phụ áp đặt lên biến a, b Do nhờ tính đối xứng ta dồn

biến biến Từ đó, đặt thêm s= c+2d ta có:

f(a, b, c, d)≤f(t, t, c, d)≤f(t, t, s, s) =f(t, s, t, s) ≤f(t+s

2 ,

t+s

2 , t, s)≤f(

t+s

2 ,

t+s

2 ,

t+s

2 ,

t+s

2 ) =f( 4,

1 4,

1 4,

1 4) =

1 27

và toán chứng minh xong! *Nhận xét:

1) Trong lời giải trên, thực chất bước ta lại phân trường hợp: có trường hợp dồn biến trường hợp mà BĐT hiển nhiên Do đó, lời giải khơng khỏi có phần rối rắm Bạn đọc nên trình bày lại cách phản chứng (giả sử có (a0, b0, c0, d0)sao cho f(a0, b0, c0, d0)> 271)

sẽ gọn gàng chặt chẽ Một cách khác gộp hai trường hợp lại:

f(a, b, c, d)≤max{ 27, f(

a+b

2 ,

a+b

2 , c, d)} (1)

2) Ở cịn có cách nhìn nữa, nhìn khơng khác ý (thậm chí dài dịng hơn), nhiên kĩ thuật có ích Ý tưởng lấy từ anh Phan Thành Nam anh Phạm Kim Hùng Diễn Đàn Mathlinks

Nhắc lại f(a, b, c, d) =ab(c+dkcd) +cd(a+b) Đặt g(x) =ab(c+

dkcd) +cd(a+b) g hàm tuyến tính, ab∈[0, t2] (vớit = a+b

2 ) nên g(ab)≤max{g(0), g(t2)} Chú ý g(0) =f(0, a+b, c, d) Vậy ta có:

(27)

Với cách viết BĐT(2) việc cực trị đạt tâm biên rõ ràng Thật ra, tốn ta có f(0, a+b, c, d) ≤

27

và chuyển (2) (1) Tuy nhiên, với phức tạp dạng (2) tỏ có ích, đặc biệt kĩ thuật dồn biến tổng quát cho n số mà

chúng trình bày phần sau

3) Các bạn tự giải toán tương tự sau Nguyễn Anh Cường

"Giả sử x, y, z, t số thực không âm thỏa mãn x+y+z +t = 4, chứng minh rằng:

3(x2+y2+z2+t2) + 4xyzt≥16 ”.

Chúng ta tiếp tục với toán mà kĩ thuật dồn biến thực rõ ràng

Bài toán 2. (Phan Thành Nam) Cho a, b, c, d số thực khơng âm

có tổng Chứng minh bằng:

abc+bcd+cda+dab+ (abc)2+ (bcd)2+ (cda)2+ (dab)2≤

Lời giải:

Lời giải sau tác giả toán Đặt f(a, b, c, d) V T BĐT cần

chứng minh Ta có:

f(a+b ,

a+b

2 , c, d)−f(a, b, c, d) = (ab

2 )

2

(c+d) + (ab )

2

[(a+b )

2

+ab](c2+d2)−(ab)

2

2 c

2 d2

≥(ab )

2

(c+d+ 4abcd−2c2d2)

Vậy c+d+ 4abcd ≥2c2d2 f(a+2b,a+2b, c, d)≥f(a, b, c, d)

Ta giả sử abcdthì theo ta có: f(x, x, c, d)≥f(a, b, c, d)với

x= a+2b Tương tự, ta xét: f(x, x,c+2d,c+2d)−f(x, x, c, d)

Nếu 2x+ 4x2cd≥ 2x4 f(x, x,c+d

2 ,

c+d

2 ) ≥f(x, x, c, d) Và ta cần

chứng minh f(x, x, y, y)≤8 với x+y = Điều đơn giản

(28)

vaø x2(c+d) ≤ (4/3)3 neân f(x, x, c, d) <= (4/3)3 + (4/3)6 < 8 Bài

tốn chứng minh xong! *Nhận xét:

1) Về điều kiện c+d + 4abcd ≥ 2c2d2 để dồn hai biến a, b nhau, ta

thấy cần abcd đủ Điều có nghĩa giả sử abcd

thì ta dồn hai biến biến a, b, c (hơn

nếu biến chưa BĐT thực sự, nghĩa sau dồn biến hàm f tăng lên đại lượng > 0) Liệu điều có dẫn đến:

f(a, b, c, d)≤f(t, t, t, c) với t= a+3b+c hay không?

Rõ ràng, giả sử f đạt cực đại (a, b, c, d) theo ta phải có

a = b=c Trên Diễn Đàn Mathlinks bạn Zhao Bin có lời giải với ý

tưởng Tuy nhiên, việc tồn cực đại hàm f (với biến)

là chuyện hiển nhiên (mặc dù rõ ràng mặc trực giác)

Một ý nữa, cách dồn biến liên tiếp biến a, b, c ta

dùng dãy số để chuyển qua giới hạn đưa biến Nhưng lần nữa, rõ ràng mặc trực giác cách làm không phù hợp với cách tiếp cận sơ cấp

Tuy nhiên, tốn bên chúng tơi cung cấp cho bạn cách làm thú vị để chuyển biến trường hợp

2) Nói thêm tốn Bài khơng khó theo lời tác giả tốn đặt để giải toán sau anh Phạm Kim Hùng: "Chứng minh với số không âm a, b, c, d có tổng thì:

1 5−abc +

1 5−bcd +

1 5−cda +

1

5−dab ≤1.

bằng cách sử dụng bổ đề sau đây: "Cho số xi ≥0thỏa mãn:

P4

i=1(xi+x2i)≤8 vàxi+xj ≤3 ,∀i6=j Thì:

5−x1

+ 5−x2

+

5−x3

+

5−x4

≤1.

(29)

Trong toán sau, câu a) anh Phạm Kim Hùng, câu b)

là kết mạnh mà chúng tơi tìm

Bài tốn 3. Choa, b, c, d≥0, a+b+c+d = ĐặtFk = (1 +ak)(1 +bk)(1 +

ck)(1 +dk) Chứng minh rằng:

a) F4 ≥F3

b) F2 ≥F1

Lời giải:

a) Ta chứng minh BĐT phản chứng Giả sử ngược lại tức tồn bốn số (a, b, c, d) thỏa mãn: a, b, c, d ≥ 0, a+b+c+d =

F4 ≤F3 (1)

Theo BĐT Bunhacôpski ta có: F4.F2 ≥ F32 , F3.F1 ≥ F22 , F2.F0 ≥ F2

1 (2) Từ (1) (2) suy F4 < F3< F2 < F1 < F0 = 16 (3) Từ (3) ta

coù F4 <16 suy max(a, b, c, d)<2

Để dẫn tới mâu thuẫn với (3), ta chứng minh F3 ≥ F1 (4) Thật

vaäy:

(4) ⇔(1−a+a2)(1−b+b2)(1−c+c2)(1−d+d2)≥1 ⇔(3

4+

(2a−1)2

4 )( 4+

(2b−1)2

4 )( +

(2c−1)2

4 )( 4+

(2d−1)2

4 )≥1 ⇔(1 + (2a−1)

2

3 )(1 +

(2b−1)2

3 )(1 +

(2c−1)2

3 )(1 +

(2d−1)2

3 )≥

4

4

⇔(1 +x2)(1 +y2)(1 +z2)(1 +t2)≥ "

1 +

x+y+z+t

4

2#4

(5)

(Trong x= 2a√−1 , y=

2√b−1 , z =

2√c−1 , t =

2√d−1 )

Từ xét BĐT

(1 +A2)(1 +B2)≥ "

1 +

A+B

2

2#2

(6) ⇔

8(AB)

2

(8−A2−6ABB2)≥0

Ta thấy A+B ≤2 BĐT

Khơng tổng quát giả sử abcd Kết hợp với a+b+

(30)

BĐT(6) ta có:

(1 +x2)(1 +t2)≥ "

1 +

x+t

2

2#2

(7)

(1 +y2)(1 +z2)≥ "

1 +

y+z

2

2#2

(8)

nhân (7) (8) vế theo vế suy ra:

(1 +x2)(1 +t2)(1 +y2)(1 +z2)≥

1 + (x+t )

2

1 + (y+z )

2

2

(9)

Từ x +t < y+z < suy ra: x+2t + y+2z < Do lại áp dụng

BĐT(6) ta được:

1 + (x+t )

2

1 + (y+z )

2

" +

x+y+z+t

4

2#2

(10)

Từ (9) (10) suy ra:

(1 +x2)(1 +y2)(1 +z2)(1 +t2)≥ "

1 +

x+y+z+t

4

2#4

Vậy (5) suy (4) (mâu thuẫn với (3)) Điều có nghĩa việc giả sử (1) sai tức ta có BĐT ngược lại F4 ≥F3 (đpcm)

b) Câu mạnh câu a) dùng "mánh lới" câu a khơng

ổn, nhiên " đường lớn tiến cơng" khơng gặp vấn đề gì:

Đặt f(a, b, c, d) = V TV P ta cần chứng minh f(a, b, c, d) ≥ Muốn

vậy, trước hết ta chứng minh mệnh đề sau: Mệnh đề: Nếu a+b≤2 vàaxb f(a, b, c, d)−f(x, a+bx, c, d)≥0

Thaät vaäy:

f(a, b, c, d)−f(x, a+bx, c, d)

= (ax)(xb) [(d+ 1)(c+ 1)−(d2+ 1)(c2+ 1)(abx2+ax+bx−2)]

(31)

Trở lại toán ta giả sử abcd Đặtx = a+3b+c thì: Chú

ý a+c≤2 vàcxa nên áp dụng mệnh đề ta có: f(a, b, c, d)≥f(a+cx, b, x, d) (1)

Chú ý x = (a+cx3)+b+x nên x = min{x, b, a+ cx} x =

max{x, b, a+cx}thìa+cx=b=xnênf(a+cx, b, x, d) =f(x, x, x, d)

và tốn cịn biến

Giả sử ngược lại, có hai trường hợp:

b < x < a+cx (2) a+cx < x < b (3)

Lại sử dụng mệnh đề cho ta:

(2) :f(a+cx, b, x, d)≥f(x, a+b+c−2x, x, d) =f(x, x, x, d) (3) :f(a+cx, b, x, d)≥f(a+b+c−2x, x, x, d) =f(x, x, x, d)

Nói chung trường hợp ta có

f(x, b, a+cx, d)≥f(x, x, x, d) (2)

Từ (1) (2) suy f(a, b, c, d)≥f(x, x, x, d) Để giải toán biến,

ta thay x= x+y3+z = 4−3d chứng minh:

f(4−d ,

4−d

3 , 4−d

3 , d)≥0 (4)

Thaät vaäy:

(4) ⇔ 729(d

6−

22d5+ 223d4−1268d3+ 4210d2 −7564d+ 6364)(d−1)2 ≥0

Bất đẳng thức cuối nên ta có điều phải chứng minh

*Nhận xét: Việc đổi biến trước dồn biến câu a) kì lạ đem lại hiệu không ngờ Kĩ thuật dồn biến câu b) mạnh, hoàn toàn sơ cấp (bởi số bước dồn biến hữu hạn) Kĩ thuật ứng dụng cực tốt cho biến Hơn thế, phần sau mở rộng để giải toán với n biến

Cuối cùng, đến với ví dụ cho trường hợp dồn biến biên Đây toán anh Phạm Kim Hùng

Bài toán 4. Cho a, b, c, d≥0 Chứng minh rằng:

a

b2 +c2+d2 +

b

a2+c2+d2 +

c

a2+b2+d2 +

d

a2+b2 +c2 ≥

4

(32)

Lời giải: Xét

f(a, b, c, d) = X

4

a

b2+c2+d2 −

4

a+b+c+d

Giả sử abcd Ta có: f(a, b, c, d)−f(a, b,

a2+b2,0)

= c

a2+b2 +d2 +

d

a2+b2+c2 −

4

a+b+c+d

−( √

c2+d2 a2+b2 −

4

a+b+ √

a2+b2)

(do a2+b2 ≥c2+d2 nên dễ thấy BĐT đúng)

Vậy vấn đề lại chứng minh f(a, b,

c2+d2,0)≥0 BĐT cuối chứng

minh khơng khó nên xin nhường lại cho bạn đọc

*Nhận xét: Cách dồn biến nhằm bảo toàn tổng a2+b2+c2+d2.

Tất nhiên, việc điều quan trọng, thích bạn bảo tồn a+b+c+d cách chứng minh f(a, b, c, d)≥

f(a+2b + a+2b, c, d), sau đánh giá f(t, t, c, d)≥ 2t

t2+ (c+d)2 +

c+d

(c+2d)2+ 2t2 −

4 2t+c+d

= x

x2/4 +y2 +

y

y2/4 +x2/2 −

4

x+y (trong x= 2t, y =c+d)

Bước cuối f(x, y)≥0 (chứng minh khơng khó bạn giả sử x+y= cho gọn)

Đến tạm kết thúc phần dồn biến cho BĐT "cụ thể" (có biến) để bước sang phần dồn biến cho BĐT n biến Như chúng

(33)

6 Dồn biến hàm lồi.

Các bạn thân mến, phương pháp dồn biến mà tìm hiểu mục trước khơng phải từ trời rơi xuống Thật ý tưởng dồn biến thể rõ BĐT cổ điển Do xếp theo dịng chảy thời gian lẽ mục phải nêu từ đầu Tuy nhiên, nghĩ thú vị trở lại gốc rễ sau bạn cảm nhận dồn biến phương pháp "hiện đại"

Một cơng cụ để dồn biến BĐT "dạng cổ điển" hàm lồi Đây khái niệm quen thuộc, nhiên để tiện lợi cho bạn đọc xin nhắc lại

Định nghĩa: Một hàm sốf : [a, b]→R gọi lồi nếu:

f(tx+ (1−ty))≤tf(x) + (1−t)f(y),x, y∈[a, b],t∈[0,1]

*Nhaän xét:

1) Nếu f khả vi lần tiêu chuẩn quan trọng để kiểm tra tính lồi

laø f00(x)≥0,x∈(a, b)

2) Nếu f lồi f liên tục Ngược lại, f liên tục tính lồi f

tương đương với điều "yếu hơn" là: f(x+2y)≤ f(x)+f(y)

2

Các bạn thấy, định nghĩa hàm lồi đánh vào mục tiêu dồn biến Chúng ta có kết quen thuộc sau:

Định lý: (BĐT Jensen) Cho f hàm số lồi [a, b]→R

(i) Với xi n số thuộc [a, b]ta có:

f(x1+x2 + +xn

n )≤

f(x1) +f(x2) + +f(xn)

n

(ii) Với xi n số thuộc A λi n số khơng âm có tổng ta có:

f(λ1x1+λ2x2+ +λnxn)≤λ1f(x1) +λ2f(x2) + +λ1f(xn)

(34)

Bài toán 1. (BĐT Cauchy) Cho n số thực dươngxi Chứng minh rằng:

x1+x2+ +xn

n

n √

x1x2 xn

Lời giải:

Laáy logarit vế, ta chuyển dạng:

ln(x1+x2+ +xn

n )≥

ln(x1) +ln(x2) + +ln(xn)

n

Hàm số f(x) =ln(x)đi từR+ →R khả vi lần vàf00(x) =−x−2 <0,x >

0 Do hàm g(x) = −f(x) thỏa g00(x)> 0,x >0 Vậy g lồi Từ đó,

áp dụng BĐT Jensen ta có điều phải chứng minh

*Nhận xét: Một cách khác thơng dụng dùng để chứng minh BĐT Cauchy, chứng minh quy nạp theo n Cách làm hay, ta có cảm

giác "cái cho n = cho n tùy ý" Các bạn

quan sát kĩ cách chứng minh đó, chứng minh lại BĐT Jensen, bạn thấy hàm lồi tổng quát nói lên chất vấn đề

Hàm lồi ứng dụng nhiều BĐT cổ điển, BĐT cổ diển lại giải nhiều tốn khác Tất nhiên, khơng phải công cụ "vạn năng", nhiên biết sử dụng khéo léo sức mạnh khơng nhỏ Chúng tơi dẫn ví dụ cho thấy áp dụng hàm lồi kết quả, song giúp giải trường hợp quan trọng mà trường hợp lại chứng minh đơn giản cách hay cách khác

Bài toán 2. Cho số thực x, y, z có tổng Chứng minh rằng: x

1 +x2 + y

1 +y2 + z

1 +z2 ≤

9 10

Lời giải:

Xét f(t) = 1+tt2 BĐT cần chứng minh tương đương:

f(x) +f(y) +f(z)≤3f(x+y+z

(35)

Do đó, −f hàm lồi coi tốn giải

Ta có:

f00(t) = 2t(3−t

2

) (1 +t2)3

nên −f00(t)≥0,t ∈[0,√3] Vậy nếux, y, z∈[0,√3]thì tốn giải

quyết

Trong trường hợp cịn lại chắn ta có dấu BĐT thực Do việc chia thành nhiều trường hợp để xét

Có thể giả sử xyz lưu ý x+y+z = vàx, y, z /∈[0,1] nên z phải

âm suy f(z)<0

*Nếu y âm suy x dương vàf(y)< 0, ta có f(x) +f(y) +f(z) < f(x)<

1/2<9/10

*Nếu ydương suy rax dương lưu ýf(y), f(x)nghịch biến trên[ √

3,+∞]

do f(x) +f(y) +f(z)< f(x) +f(y)< f( √

3) +f( √

3)<9/10

Bài toán chứng minh xong

*Nhận xét: Tất nhiên lời giải chưa phải ngắn gọn so với nhiều lời giải khác cho tốn mà chúng tơi biết Tuy nhiên tư tưởng hồn tồn sáng Ở đây, thay mong muốn dồn biến tồn cục (dồn lần biến) việc hi vọng hợp lý dồn biến lời giải ngắn Thật vậy, có biến x, y, z

thuộc đoạn [0,

3]thì dùng hàm lồi ta dồn biến nhau, tốn cịn biến, xem giải xong Trong phần cịn lại việc chia trường hợp đơn giản Như vậy, có thêm kĩ thuật để dồn biến sử dụng hàm lồi

Mặc dù công cụ tốt, điểm yếu dễ nhận BĐT, biến phải nằm biểu thức độc lập (để viết thành dạng f(x1) + +f(xn)) Trong đó, BĐT mà ta gặp phần lớn khơng có điều đó, ta phải làm việc với dạng tổng quát f(x1, , xn) Chúng ta phải thiết lập kết dồn biến cho dạng tổng quát mục sau

(36)

Định lý: Cho f : [a, b]→R hàm lồi Khi đó: f(x)≤max{f(a), f(b)},x∈[a, b]

Chứng minh:

f liên tục nên f đạt giá trị lớn x0 ∈[a, b] Xét |x0−a| ≤

|x0 −b| (nghĩa x0 gần a b) Thì x1 = 2x0 −a ∈ [a, b] Khi theo

định nghóa hàm lồi ta có:

f(a) +f(x1)≥2f( a+x1

2 ) = 2f(x0)

suy f(a) =f(x0) Với x0 gần b a chứng minh tương tự

*Nhận xét: Để bạn cảm nhận "cái đúng" định lý nêu hình ảnh f00(x)> 0,x ∈ (a, b) Khi đó, f0 đồng biến

nên có tối đa nghiệm (a, b), nói cách khác đổi dấu tối đa lần Do f rơi vào trường hợp sau đây: đồng biến, nghịch biến, "đi

lên xuống", "đi xuống lên" Và trường hợp ta thu kết cần thiết (Một chứng minh khác trường hợp giả sử f đạt cực đại x0 ∈(a, b)thì f00(x0)≤0, mâu thuẫn.)

Chúng tơi dẫn tốn mà chúng thực tốn khó cho dù giải biến đổi đại số hay quy nạp

Bài toán 3. Cho 0< p < q, n số thực xi∈[p, q] Chứng minh rằng:

(x1+x2+ +xn)(

x1

+

x2

+ +

xn

)≤n2 +

n2

4

(pq)2 pq

trong kí hiệu [x]là phần nghuyên x

(*Ghi chú: Đây tổng quát, trường hợp n = USAMO 77, n= đề thi Olympic 30−4 năm 2001 )

Lời giải:

Từ giả thiết xi ∈ [p, q], ta dễ dàng đoán rằng: GTLN đạt

xi ∈[p.q]với i Khi đó, g/s n số xik số pnk số q thì:

V T = (kp+ (nk)q)(k

q +

nk

q ) =k

2

+ (nk)2+k(nk)(p

q +

(37)

=n2+k(nk)(pq)

2

pq =n

2

+1

n2−(n−2k)2(pq)

2 pq

Vì k nguyên nênn2−(n−2k)2 ≤n2(khi n chẳn) vàn2−(n−2k)2 ≤n2−1

(khi n lẻ) Từ đó, ta thu BĐT ban đầu đồng thời trường hợp dấu xảy

Đến đây, ta nhận ra: mấu chốt vấn đề nhận xét: "GTLN đạt xi = p xi =q với i" Và thật bất ngờ, nhận xét

này chứng minh dễ

Với i, ta xem vế trái hàm theo xi, ta chứng tỏ: f(xi) ≤

max{f(p), f(q)}, vaø dấu xảy xi ∈ {p, q}

Ta có: f(x) =Ax+Bx +C Có thể khảo sát hàm để kết (suy

ra dấu xảy xi ∈ {p, q}) Song trình bày cách sơ

cấp Để ý:

f(xi)−f(p) = (xip)(A

B xip

)

f(xi)−f(q) = (xiq)(A

B xiq

)

Từ f(xi)> max{f(p), f(q)} rõ ràng xi ∈ {/ p, q} và:

AB

xip

>0, AB

xiqB

xip

< A < B xiq

mâu thuẫn p < q Vậy f(xi)≤max{f(p), f(q)}

Cần nói thêm trường hợp dấu bằng: g/s f(xi) =max{f(p), f(q)}mà

xi ∈ {/ p, q} Neáu f(xi) = f(p) A = xB ip >

B

xip, f(xi)−f(p) < (mâu thuẫn) Tương tự, f(xi) = f(q) mâu thuẫn Vậy f(xi) =

max{f(p), f(q)}tương đương với xi ∈ {p, q} *Nhận xét: Ta có tốn mở rộng sau:

"Cho ai ∈[a, A], bi∈ [b, B] với 0< aA 0< bB Tìm giá trị lớn

T = (a

2

1+ +a

n)(b

2

1+ +b

n)

a1b1+ +anbn

.” Nhà toán học Polya cho chặn là:

4

q AB

ab + q

ab AB

2

(38)

một điều tự nhiên đặt chặn BĐT Cơsi ? Nếu bạn tị mị xem tiếp tốn sau đây:

Bài toán 4. (Phan Thành Nam) Cho < a < b, n số thực xi ∈ [p, q]

Chứng minh rằng:

T = x1

x2

+ x2

x3

+ +xn

x1

n+ hn

2

i(pq)2 pq

Lời giải:

Với i, thay xi p hay q trường hợp T phải tăng

lên, T không tăng buột xi ∈ {p, q}.

Cho i chạy từ tới n, với i ta thay xi p hay q cho T tăng

lên (hoặc giữ nguyên hai trường hợp không tăng)

Sau bước biến đổi ta có xi ∈[p, q]với i.Nếu xi=q với

i T = n, khơng phải GTLN, cần xét ∃xi =p.Do hốn vị

vịng quanh nên giả sử x1 =p. Khi x3 =p hay q ta thay x2 q T không giảm Sau thay x2 q ta lại thay x3 p

thì T không giảm Cứ ta xen kẽ p, q số xn T

khơng giảm Sau thực trình lúc ta có

T = n

2(

p

q +

q

p) (neáu n chẵn) T = n−1

2 (

p

q +

q

p) + (neáu n leû)

Ta viết lại trường hợp dạng:

T =n+hn

i(pq)2

pq ,n

và vế phải BĐT cần chứng minh Đẳng thức xảy

xi ∈ {p, q} xen kẽ kể từ x1 tới xn (không kể vịng xn, x1) Bài tốn đến

đây giải trọn vẹn !

Như vậy, thấy ý tưởng dồn biến xuất sớm cách tiếp cận cổ điển Chúng ta gặp lại kĩ thuật dồn biến

(39)

biệt trường hợp cực trị đạt tâm, hàm lồi cho ta kiểu dồn biến thú vị mà tìm hiểu mục sau Mặc dù với loạt BĐT xuất gần công cụ cổ điển không đủ (hoặc khó khăn), lần nữa, chúng tơi nhấn mạnh tầm quan trọng ý tưởng "cổ điển", mà dựa vào "đứng vai người khổng lồ"

7 Dồn biến giá trị trung bình.

Cho đến bây giờ, phương pháp dồn biến chúng ta, số lần thực thao tác dồn biến hữu hạn, nhờ lời giải rõ ràng hồn tồn sơ cấp Đây điều tốt mà muốn trì tiếp tục mục

Trước hết, giới thiệu thêm cách dồn biến dành cho hàm lồi Ta gọi kĩ thuật dồn biến giá trị trung bình, mà bạn thấy rõ điều qua kết sau:

Định lý: Cho f hàm lồi [a, b]→R Ta coù:

f(a) +f(b)≥f(x) +f(a+bx),x∈[a, b]

Chứng minh:

x ∈ [a, b] nên: x = ta+ (1−t)b với t ∈ [0,1] Khi đó: a+bx = (1−t)a+tb Áp dụng định nghĩa hàm lồi, ta có:

f(x) +f(a+bx) =f(ta+ (1−t)b) +f((1−t)a+tb) ≤[tf(a) + (1−t)f(b)] + [(1−t)f(a) +tf(b)] = f(a) +f(b)

Ứng dụng kết này, ta có chứng minh cho BĐT Jensen Nhắc lại: Định lý: (BĐT Jensen) Cho f hàm số lồi [a, b]→ R Thì với xi ∈ [a, b]

là n số có trung bình cộng T, ta coù:

f(x1) +f(x2) + +f(xn)≥nf(T)

Chứng minh:

Ta cho thực thuật toán sau:

(40)

*Bước 2: Vì khơng có xi = T,i nên phải có biến lớn hơn T

1 biến nhỏ T, mà ta giả sử x1 > T > x2 Khi thay

(x1, x2, , xn) (T, x1+x2−T, , xn) Sau trở lại bước

Như lần thực bước có trung bình cộng T, nhiên làm cho biểu thức f tăng lên Mặt khác lần thực

hiện bước số biến T tăng lên 1, sau hữu hạn (có thể lấy n−1) lần thực bước 2, ta phải dừng lại bước Chú ý

là trình thay biểu thức f tăng lên, ta có điều phải

chứng minh

Vậy có thêm cách dồn biến Sỡ dĩ không dưa cách dồn biến mục trước, có giá trị dồn biến tâm, mà với n = kĩ thuật dồn biến

nhau đủ sử dụng Tuy nhiên, kĩ thuật phát huy tác dụng số biến tăng lên, cụ thể với trường hợp n biến tổng quát Lý đơn giản: BĐT với n biến, cho dù ta dồn biến chưa thu đáng kể, trường hợp sau hữu hạn lần dồn biến đưa trường hợp biến (chứ chưa nói đưa trường hợp biến nhau) Tuy nhiên, sử dụng kĩ thuật dồn biến biên dồn biến giá trị trung bình tình hình lại khác: sau lần dồn biến số lượng biến có giá trị cố định tăng lên (là giá trị biên giá trị trung bình), cần hữu hạn lần dồn biến ta đưa tất biến giá trị cố định toán xem giải xong

Tất nhiên, khả để dồn biến biên giá trị trung bình khơng cao Tuy nhiên, quan trọng tinh thần nó: dồn biến giá trị cố định Bạn đọc thấy ý tưởng hiệu trường hợp biến (xem câu c), Bài toán 3, $5) Trong mục này,

chúng tiếp tục giới thiệu tốn khác, mà ý tưởng dồn biến giá trị trung bình cho lời giải bất ngờ Đây toán đặc sắc anh Phạm Kim Hùng, mà việc giải chúng đem lại cho nhiều ý tưởng cho phương pháp dồn biến

Bài toán 1. Cho n số thực dương a1, a2, , an có tích Chứng minh

rằng với k= 4(n−1) ta ln có:

a1

+

a2

+ +

an

+ k

a1+a1+ +an

n+k

n (1)

(41)

Với n = 1, n = tốn đơn giản, nên ta xét n

Trước hết, ta khảo sát trường hợp dồn biến rút ra:

Mệnh đề 1: Kí hiệu f(a1, a2, , an) biểu thức vế trái BĐT cần chứng

minh

(i) Neáu a1 ≤xa2 vàa1a2 ≤1thì f(a1, a2, , an)≥f(x,

a1a2

x , a3, , an)

(ii) Neáu (1−a1)(1−a2)[ka1a2−(

Pn i=1ai)(

Pn

i=3ai+a1a2 + 1)]≥0thì f(a1, a2, , an)≥f(1, a1a2, a3, , an)

(iii)Neáu a1, a2 ≥1≥a3 thì:

f(a1, a2, , an)≥min{f(1, a1a2, a3, , an), f(1, a2, a1a3, a1ai, , an)} Chứng minh mệnh đề 1:

Để viết cho gọn ta đặt A=Pni=3ai (i) Ta có:

f(a1, a2, , an)−f(x,

a1a2

x , a3, , an)

=

a1

+

a2

x

x a1a2

+ k

A+a1+a2

k

A+x+ a1a2 x

= (xa1)(a2−x)[(A+a1+a2)(A+x+ a1a2

x )−ka1a2]

a1a2(A+a1 +a2)(A+x+ a1xa2)

Theo BÑT Cauchy:

(A+a1+a2)(A+x+ a1a2

x ))≥n

2

≥4(n−1) =kka1a2

và ta có đpcm

(ii) Cũng từ đẳng thức cho x= ta có:

f(a1, a2, , an)−f(1, a1a2, a3, , an)

= (1−a1)(1−a2)[ka1a2−(A+a1+a2)(A+a1a2+ 1)]

a1a2(A+a1+a2)(A+a1a2+

vaø ta có đpcm

(42)

Trường hợp 1: Nếu ka1a2 ≥(

Pn i=1ai)((

Pn

i=3ai+a1a2+ 1)])thì dùng (ii)ta

f(a1, a2, , an)≥ f(1, a1a2, a3, , an) Trường hợp 2: Nếuka1a2 ≤(P

n i=1ai)((

Pn

i=3ai+a1a2+1)])thì vìa3 ≤1≤a2

nên:

ka1a3≤ (

n X

i=1 ai)(

X i6=1,3

ai+a1a3+ 1)

(thaät vaäy:

P

i6=1,3ai+a1a3+ a1a3

= Pn

i=1aia1−a3+ a1a3

+ ≥

Pn

i=1aia1−a2+ a1a2

+ = Pn

i=3ai+a1a2+ a1a2

)

Do đó, dùng (ii) ta có: f(a1, a2, , ai, , an)≥f(1, a2, , a1ai, , an)

Mệnh đề chứng minh xong! Nó cho phép ta đưa toán biến Mệnh đề 2: Ta ln đưa tốn trường hợp có n −1 biến

bằng ≤1 Chứng minh mệnh đề 2:

*Bước 1: Đưa trường hợp có n−1 biến ≤1

Giả sử cịn có nhiều biến lớn 1, mà ta giả sử a1, a2

Thì sử dụng mệnh đề 1(iii)ta ln thay bộ(a1, , an)bởi khác,

vẫn có tích 1, làm cho f khơng tăng, có số biến

tăng lên Do sau hữu hạn lần thay (khơng q n−1) ta

n−1 biến≤1

*Bước 2: Đưan−1biến ≤1

Giả sử a1 ≤ a2 ≤ an−1 ≤ n −1 biến có trung bình nhân x Nếu n−1 biến chưa a1 < x < an−1 dùng mệnh

đề (i) ta thay (a1, a2, , an−1, an) (x, a2, ,

a1an−1

x , an) Khi

đó f khơn giảm số biến x tăng lên Ta lưu ý

a1an−1

x

a1

x ≤1(vìa1 số nhỏ trongn−1sốa1, , an−1 nên a1 ≤x),

do việc thay đảm bảon−1biến ≤1, điều cho phép

việc thay thực liên tiếp Vậy sau hữu hạn (khơng qn−1)

lần thay ta có n−1biến ≤1

Cuối cùng, ta giải toán biến, tức chứng minh:

f(x, x, , x,

(43)

Đặt:

g(x) := f(x, x, , x, xn−1) =

n−1

x +x

n−1

+ k

(n−1)x+ xn1−1

với x∈(0,1]

Ta coù:

g0(x) =−n−1

x2 + (n−1)x

n−2

k[n−1− n−1

xn ] ((n−1)x+xn1−1)2 = (n−1)x

n−1

x2

(n−1)xn−1 (n−1)xn+ 1

2

lưu ý k = 4(n−1)

Ta thấy g(x)≤0với x∈(0,1], nên g(x)≥ g(1) ta có đpcm

Bài toán chứng minh xong!

*Ghi chú: Bài toán ban đầu anh Phạm Kim Hùng với k = 3n, n≥ Kết mạnh hơn, bạn thấy chứng minh cho trường hợp biến só k = 4(n−1) "hợp lý"

Bài toán 2. Cho n số thực dương a1, a2, , an có tích Chứng minh

raèng:

(1 +a21)(1 +a

2) (1 +a

n)≤ 2n

n2n−2(a1+a2+ +an) 2n−2

Lời giải:

Với n = 1, n = đơn giản nên ta chứng minh cho n ≥ Ta

thấy toán tương đương với f(a1, a2, , an)≥0 tương đưong với

g(a1, a2, , an)≥0, đó:

f(a1, a2, , an) =k(a1+a2+ +an)2n−2−(1 +a21)(1 +a

2) (1 +a

n)

g(a1, a2, , an) = ln(k) + (2n−2) ln(a1+a2+ +an)+ −ln(1 +a21)−ln(1 +a

2

2)− −ln(1 +a

n)

(về việc phải xét fg bình luận sau)

Khảo sát sơ trường hợp dồn biến, ta có:

Mệnh đề 1:

(i) Nếu a1 ≥1≥a2, a3 thì:

(44)

(ii) Nếu a1 =max{ai}in=1 a1 ≥xa2 ≥1 thì: g(a1, a2, , an)≥g(x,

a1a2

x , a3, , an)

Chứng minh mệnh đề 1: (i) Xét hiệu

f(a1, a2, , an)−f(1, a1a2, a3, , an) =ks2n−2−ku2n−2+ [2(1 +a21a

2

2)−(1 +a

1)(1 +a

2)](1 +a

3) (1 +a

n)

(vớis=a1+a2 + +an, u= +a1a2+ +an) =k(a1+a2−1−a1a2)(s2n−3+s2n−4u+ +u2n−3)+

+(1−a21)(1−a

2)(1 +a

3) (1 +a

n) =−(1−a1)(1−a2)[k(s2n−3+ +u2n−3)+

−(1 +a1)(1 +a2)(1 +a23) (1 +a2n)]

Sử dụng lại đẳng thức với a3 đổi chỗ cho a1, ta có: f(a1, a2, , an)−f(a1,1, a2a3, , an)

=−(1−a2)(1−a3)[k(s2n−3 + +v2n−3)+

−(1 +a2)(1 +a3)(1 +a21)(1 +a

4) (1 +a

n)]

(với v= +a2a3+a1+a4+ +an)

Từ đẳng thức trên, ta thấy:

∗ neáu k(s2n−3+ +u2n−3)−(1 +a1)(1 +a2)(1 +a23) (1 +a

n)≥0 (2)

Thì f(a1, a2, , an)≥f(1, a1a2, a3, , an) ∗neáuk(s2n−3+ +v2n−3)−(1+a

2)(1+a3)(1+a21)(1+a

4) (1+a

n)≤0 (3)

Thì f(a1, a2, , an)≥f(1, a1a2, a3, , an)

Do đó, ta cần chứng minh BĐT (2) (3) có

đúng xong! Chẳng hạn, ta giả sử (2) sai, chứng minh (3)

Muốn vậy, ta cần chứng minh: uvvà(1+a1)(1+a32)≤(1+a3)(1+a21)

là xong! Điều có từ việc tính tốn đơn giản:

uv=a3+a1a2−a1−a2a3 = (1−a2)(a1−a3)≥0

(1 +a1)(1 +a23)−(1 +a3)(1 +a21) = (a3−a1)(a1a3 +a1+a3−1) ≤0

Vậy mệnh đề (i) chứng minh xong!

(45)

vào ta dùng đạo hàm Xét:

g(t) = ln(k) + 2(n−1) ln(ta1+ a2

t +a3 + +an)+

−ln(1 +t2a21)−ln(1 + a2

2

t2)−ln(1 +a

3)− −ln(1 +a

n)

với t ∈[pa2/a1,1]

Ta coù:

g0(t) = 2(n−1)(a1 − a2 t2)

ta1+ at2 +a3+ +an

− 2ta

2 1−

2a2 t3 (1 +t2a2

1)(1 +

a2 t2) = 2(a1−

a2 t2)[

(n−1)

ta1+ at2 +a3+ +an

ta1+ a2

t (1 +t2a2

1)(1 +

a2 t2)

]

t ∈[pa2/a1,1] nên a1− at2 ≥ Do đó, gọi T thừa số lại, ta

cần chứng minh T ≥0 suy rag đồng biến (trên [pa2/a1,1])

Để viết cho gọn, ta đặt

c= r

(1 +t2a2 1)(1 +

a2

t2), d=ta1+ a2

t

Ta coù:

T ≥0⇔ n−1

d+a3+ +and

c2 ⇔(n−1)c

d2+d(a3+ +an)

cd (BĐT Bunhiacopski) nên để có BĐT ta cần:

(n−2)ca3+ +an

Điều c > a1a2 ≥a1 ≥max{a3, , an}

Lấy t0 =max{x,a1xa2}/a1,

t0 ∈[

r

a2 a1

,1], t0a1=max{x, a1a2

x }, a2

t0

=min{x,a1a2

x }

g đồng biến [pa2/a1,1]nên g(1)≥g(t0) ta có đpcm

(46)

Trở lại tốn, ta nói (a1, a2, , an) thay (b1, b2, , bn)) f(a1, a2, , an) ≥ f(b1, b2, , bn) hoặc g(a1, a2, , an) ≥

g(b1, b2, , bn)

Mệnh đề 2: Luôn đưa trường hợp có n−1 biến ≥1 Chứng minh mệnh đề 2:

*Bước 1: Đưa trường hợp có n−1 biến ≥1

Giả sử cịn có biến a2, a3<1 Khi phải có biến>1, mà ta

giả sử a1 Sử dụng mệnh đề (i), ta thay (a1, a2, , an) (1, a1a2, a3, , an) (a1,1, a2a3, , an) Chú ý cho dù thay

nào, số biến tăng lên Do đó, động tác thay phải dừng lại sau khơng q n−1 lần Khi đó, ta có n−1

biến ≥1

*Bước 2: Ta chứng minh ln thay n−1 biến ≥ trung bình

nhân chúng Thật vậy, giả sử a1 ≥ a2 ≥ an−1 ≥ ≥ an

đặt x = n−√1 a

1, a2, , an−1 ≥ Nếu n−1 biến cịn

biến khác x a1 > x > an−1 Sử dụng mệnh đề (ii) ta thay

(a1, a2, , an−1, an) (x, a3, ,a1xa2, an) Chú ý a1an−1

xan−1 ≥

(vì a1 số lớn số {ai}in=1−1 nên a1 ≥ x) việc thay

thế đảm bảo n −1 biến ≥ (để thay liên

tiếp) Chú ý sau thay số biến xtăng lên

Do đó, sau khơng qn−1lần thay cản−1biến bằngx

Cuối ta giải toán biến Xét hàm số h(x) :=g(x, x, , x,xn1−1)

= ln(k) + 2(n−1) ln((n−1)x+

xn−1)−(n−1) ln(1 +x

)−ln(1 +

x2n−2)

với x≥1

Ta coù:

h0(x) = 2(n−1) n−1− n−1

xn (n−1)x+xn1−1

−2(n−1)x +x2 −

−2(n−1) x2n−1 + x2n1−2 = 2(n−1)

x

(n−1)(xn−1) (n−1)xn+ 1 −

x2

1 +x2 +

1 +x2n−2

= 2(n−1)

x

(n−1)(xn−1) (n−1)xn+ 1 −

x2n−1 (1 +x2)(1 +x2n−2)

(47)

Chú ý x≥1nên để có h0(x)≥0 ta cần: n−1

(n−1)xn+ 1 ≥

xn+ (1 +x2)(1 +x2n−2)

Ta đạt điều đánh giá đơn giản:

n−1

(n−1)xn+ 1 ≥

xn+ 1 ≥

xn+ (1 +x2)(1 +x2n−2)

(Có dấu ≥ thứ hai BĐT Bunhiacopski )

Vậy với x≥1 h0(x)≥0 nên h(x) đồng biến, suy h(x)≥h(1) = 0

ta coù ñpcm

Vậy toán chứng minh xong! Đẳng thức xảy a1 =a2= =an= với n≥3 (cịn với n= 1, n= có đẳng thức)

*Nhận xét:

1) Bài tốn anh Phạm Kim Hùng đặt dạng toán mở chứng minh chúng tơi chứng minh cho

2) Ở việc xét đồng thời hàm f, g cho phép ta mở rộng khả dồn

biến: xét f đơn giản hơn, xét g đơn giản Trong tốn

1 vấn đề đơn giản nên cần hàm f đủ

8 Định lý dồn biến tổng quát.

Các bạn thân mến, nói định lý dồn biến phải nhắc tới kết ấn tượng, định lý dồn biến mạnh (SMV) anh Phạm Kim Hùng định lý dồn biến không xác định (UMV) bạn Đinh Ngọc An Trong đó, "xương sống" định lý bổ đề dãy số, kết cho ta cảm giác rõ ràng dồn biến

Trong mục này, cung cấp cho bạn định lý dồn biến tổng quát − định lý GMV anh Phan Thành Nam − với

cách tiếp cận Có thể trình bày ngắn gọn cách dẫn định lý chứng minh nó, nhiên chúng tơi khơng làm muốn chia với bạn đường (trong tư duy) để xây dựng Hi vọng sau xem xong, bạn có cảm giác có vơ số định lý dồn biến

(48)

Định nghóa 1:

• Khơng gian Rn tập hợp thứ tựx= (x

1, x2, , xn) vớixiR,i •Một dãy {xm= (x1,m, , xn,m)} trongRn gọi hội tụ z = (z1, , zn)∈

Rn dãy xi,m hội tụ zi m→ ∞, ∀i= 1,2, , n

• Cho DRn Một hàm số f :DR gọi liên tục D nếu: với

dãy {xm} ⊂ D với zD cho {xm} hội tụ z, ta có:

f(xm) hội tụvề f(z)

Định nghóa 2: Cho DRn Ta nói:

• D đóng với dãy {xm} ⊂ D với zRn cho {xm} hội

tụ z, ta có zD

• D bị chặn tồn số thực M cho: ∀x = (x1, , xn) ∈ D}, |xi| ≤M,i= 1,2, , n

Ví dụ tập hợp hữu hạn đóng bị chặn Xuất phát điểm kết tuyệt đẹp sau đây:

Định lý 1: Cho D đóng bị chặn Rn, f : DR liên tục

Thì f đạt giá trị nhỏ D, nghĩa tồn x0 ∈ D cho: f(x0)≤f(x),xR

Đây kết có chương trình phổ thơng nước, nhiên nước ta xem thuộc "Tốn cao cấp" Tuy nhiên, để tiện lợi cho bạn đọc dẫn chứng minh mà bạn hồn tồn hiểu với kiến thức phổ thơng

Chúng tơi nhắc lại kết có SGK: " dãy số thực đơn điệu bị chặn hội tụ" "Tiên đề" sử dụng để chứng minh kết dãy

Định nghĩa 2: Cho dãy số {am}∞m=1 (trong R R

n) Một dãy {amk}

k=1 gọi dãy dãy {am}∞m=1 nếu{mk}∞k=1 dãy

tăng ngặt số nguyên dương

*Ví dụ: {a2m}∞m=1 dãy dãy {am}∞m=1 Dưới cận

của số bỏ qua không gây hiểu lầm

Bổ đề 1: (Weierstrass) Mỗi dãy am bị chặn R có dãy hội tụ

Chứng minh:

Ta chứng minh có dãy đơn điệu xong Xét tậpT :={mZ+|∃m0 > m cho a

(49)

dãy {am} giảm kể từ số Nếu T vơ hạn ta trích

1 dãy tăng Trong hai trường hợp ta ln có dãy đơn điệu Bổ đề 2: (Weierstrass) Mỗi dãy am bị chặn trongRn có dãy hội tụ

Chứng minh:

Xét {am = (x1,m, , xn,m)} dãy bị chặn Rn Khi dãy {x1,m} bị chặn R nên có dãy {x1,mk1} hội tụ Dãy {x2,mk1} bị chặn R nên có dãy {x2,mk

2} hội tụ Bằng cách lấy

"dãy dãy con" liên tiếp vậy, cuối ta thu dãy

{amk = (x1,mk, , xn,mk}maø∀i= 1,2, , n, ta có dãy{xi,mk}hội tụ R

Điều có nghĩa dãy {amk} hội tụ R n

Bổ đề 3: (Tính đầy đủ R) Cho A tập bị chặn R Thì tồn

MR cho: MA (nghóa Ma,aA) có dãy{ak} A

hội tụ M Ta kí hiệu M =inf A

Chứng minh:

Ta chứng minh ∀ε > 0, ∃aA, aεA Giả sử ngược lại Khi

đó lấy x1 ∈A tùy ý, quy nạp ta xây dựng dãy {xm} A

cho xm+1 ≤xmε, ∀mZ+ Khi ta có: xmx1−(m−1)ε, ∀mZ+

và điều mâu thuẫn với A bị chận Như vậy, ∀mZ+, tồn a

mA choamm1 ≤A Vì dãy{am}bị

chặn nên có dãy {amk} hội tụ M R Ta chứng minh MA xong Thật vậy, lấy aA amk −

1

mk ≤ a, ∀kZ

+, neân

cho k→ ∞ suy Ma

Chứng minh định lý 1: Xét A = f(D) Ta chứng minh A có phần tử

nhỏ

Ta A có tính chất sau: dãy {am} chứa A amα

thìαA Thật vậy, theo định nghóa ta cóxmD chof(xm) =amα

Vì dãy{xm}bị chặn (chứa D) nên có dãy con{xmk} hội tụ vềctrong

Rn Vì D đóng nên cDf(xm)→ α nên Vì f(xmk)→α Mặt khác,

vì {xmk} → c f liên tục nên f(xmk) → f(c) Vì giới hạn nên f(c) =α

Bây giờ, ta thấy A bị chặn (vì từ lập luận với α = −∞ ta

gặp mâu thuẫn) Do tồn tạiM =inf A Do định nghĩa inf tính chất

của A vừa trên, suy MA Vậy A có phần tử nhỏ M

(50)

Định lý mở rộng kết quen thuộc có SGK: "Cho [a, b] khoảng đóng R vàf : [a, b]→R liên tục, f có giá

trị nhỏ [a, b]" Do đó, mặt trực giác định lý rõ ràng

Tuy nhiên, bạn khó hình dung định lý liên quan đến vấn đề dồn biến? Hệ sau định lý "chìa khóa" cho định lý dồn biến Lưu ý tất kết mục không cần điều kiện f đối xứng

Định lý 2: Cho:

• D tập đóng, bị chận Rn, Λ tập đóng DT :DD phép biến đổi

f :DR hàm số liên tục thỏa mãn f(x)> f(T(x)),xD

Thì ta có GTNN f đạt Λ, nghĩa là:

f(x)>min

y∈Λ{f(y)},xD.

Chứng minh:

Do định lý 1, tồn x0 ∈ D cho f(x0) ≤ f(x),xD Nếu x0

không thuộc Λ f(x0)> f(T(x0)), mâu thuẫn Vậy x0 ∈Λ ta có điều

phải chứng minh

*Ghi chú: Ta thấy phép biến đổi T :DD thực định lý

trên địi hỏi tính chất T D\Λ Do với x ∈ Λ T(x) có

thể lấy giá trị tùy ý ta xem T(x) = x Quy ước

sử dụng phần lại, nghĩa T(x) =x,x ∈ Λ ta quan tâm

giá trị T D

Đây hệ đơn giản phải không bạn, nhiên ý tưởng dồn biến lộ rõ Để minh họa, dẫn chứng minh cho BĐT Cauchy

Bài toán 1: (BĐT Cauchy) Chon số thực không âmx1, , xn Chứng minh

raèng:

x1+ +xnnn √

x1 xn

Chứng minh:

Bằng cách chuẩn hóa, ta giả sử x1 xn = chứng minh

x1+ +xnn Tất nhiên ta cần xét xin,i

Xét: D ={x= (x1, , xn)|xi ∈[0, n], x1 xn= 1}thì dễ thấy D đóng

bị chặn Xét Λ ={x0 = (1,1,1, ,1)}

(51)

thì f(x) = x1 + +xn Xét T : D\Λ → D sau: Với x = (x1, , xn) ∈ D\Λ, tồn xi 6= xj ta đặt T(x) thu từ

x sau thay xi xj trung bình nhân chúng, dễ thấy

f(x)−f(T(x)) = (√xi− √

xj)2 >0

Vậy ta áp dụng định lí để suy raf(x)≥f(x0),xD,

daáu ” = ” xảy x=x0

Trong nhiều trường hợp, hàm f khơng đủ tốt ta

có có điều kiện f(x) ≥ f(T(x)) Tất nhiên ta khơng thể áp dụng

định lý Một đòi hỏi hợp lý phép biến đổi T phải đủ tốt để bù lại (nhớ phép biến đổi T ta chọn) Điều đưa đến:

Định lý 3: Cho:

• D tập đóng, bị chận Rn, Λ tập đóng D

T : DD phép biến đổi cho tồn hàm số h liên tục DR thỏa mãn: h(T(x))< h(x),∀xD

f :DR hàm số liên tục thỏa mãn f(x)≥f(T(x))∀xD

Thì ta có GTNN f D GTNN f trênΛ, nghóa là:

f(x)≥min y∈Λ

{f(y)},xD.

Mặc dù trường hợp riêng định lý tổng quát cuối bài, tầm quan trọng định lý nên dẫn chứng minh cho

Chứng minh:

Lấy y0 ∈Λ cho f(y0) =

y∈Λ

{f(y)} Giả sử phản chứng tồn

zD cho f(z) < f(y0) Tất nhiên ta giả sử h(x) ≥ 0,xD

(nếu không việc thay h h0=hM, với M GTNN htrên D)

Chọn ε > đủ nhỏ ta có: f(z) +εh(z)< f(y0) Đặt g(x) := f(x) +εh(x),

xD Thì g : DR liên tục, g(x) > g(T(x))∀xD\Λ vaø g(z) < f(y0)≤min

y∈Λ

{g(y)} Điều mâu thuẫn với định lý

Sau hệ ấn tượng định lý Hệ 1: (SMV-Strongly Mixing Variables) Cho:

DRn, D = {x = (x

1, , xn)|xiα, P

xi = ns = const} vaø s0 :=

(s, s, , s)∈D

• Phép biến đổi T :DD sau: với phần tử a = (a1, , an)∈ D,

(52)

ai6=aj, thay ai, aj bới trung bình cộng chúng

f :DR hàm số liên tục thỏa mãn: f(a)≥f(T(a)),aD

Khi đó: f(a)≥f(s0),aD

Chứng minh: Với phép biến đổi T vậy, ta chọnh(x) =Pin=1x2i,x= (x1, , xn)∈D Áp dụng định lý (ở Λ ={s0})

*Nhận xét: Thông thường, áp dụng ta lấy ai, ajminmaxcủa {a1, , an} Khi đó, chứng minh từ phần tử D, sau vô

hạn lần lặp T thu (s, s, , s), sử dụng tính liên tục củaf ta

thu kết luận

Riêng trường hợp (min max) khơng thiết thay ai, aj trung bình cộng mà tổng qt hơn:

Hệ 2: Cho:

DRn đóng bị chặn Gọi Λ tập hợp phần tử D có dạng

(s, s, , s), giả sửΛ khác rỗng

•Phép biến đổi T :DD sau: với phần tửa= (a1, , an)∈D\Λ,

ta choïn số i 6=j cho ai, aj minmax {a1, , an}, sau

đó thay ai, aj α, β ∈(ai, aj)

f :DR hàm số liên tục thỏa mãn: f(a)≥f(T(a)), ∀aD

Khi đó:

f(x)≥min y∈Λ

{f(y)},xD.

Chứng minh: Một cách tự nhiên, ta hi vọng vào hàm

h(a) =max{a1, , an} −min{a1, , an},a= (a1, , an)∈D

Tuy nhiên, ta khơng có h(a) > h(T(a)), ∀aD\Λ Đó

n số a1, , an có nhiều số max hay min {a1, , an} Nhưng ta việc thay T T∗ = Tn (Tk nghĩa lặp lại T

với k lần) h(a)> h(T∗(a)),aD\Λ ta áp dụng định lý

Tuy nhiên, phép biến đổi T khơng đủ, ví dụ ta chưa biết xác dồn biến biên hay tâm Do đó, định lý mở rộng thành định lý dồn biến tổng quát sau

Định lý 4: (GMV − General Mixing Variables) Cho:

• D tập đóng, bị chận Rn, Λ tập đóng D

Tj : DD phép biến đổi cho tồn hàm số hj liên tục

(53)

f :DR liên tục thỏa mãn f(x)≥ j∈{1, ,k}

{f(Tj(x))},xD

Thì f(x)≥min y∈Λ

{f(y)},xD

Ta sử dụng lại chứng minh định lý với cải tiến nhỏ Chứng minh:

Laáy y0 ∈Λ cho f(y0) =

y∈Λ

{f(y)} Giả sử phản chứng tồn

zD cho f(z) < f(y0) Tất nhiên ta giả sử hj(x) ≥ 0,x

D,j = 1, , k Chọnε >0đủ nhỏ ta có: f(z)+εhj(z)< f(y0),j = 1, , k

Đặt gj(x) :=f(x) +εhj(x), ∀xD,j = 1, , k

Đặt g(x) = min{g1(x), , gk(x)},xD Thì g : DR liên tục,

g(x) > g(T(x))∀xDg(z) < f(y0) ≤

y∈Λ

{g(y)} Điều mâu

thuẫn với định lý

*Chi chú: Ta sử dụng kết gj hàm liên tục g =

min{g1, , gk} hàm liên tục Tất nhiên ta cần chứng minh

với k = 2, trường hợp cần để ý min{g1, g2} =

2(g1+g2− |g1−g2|) Cịn kiệng(x)> g(T(x)),xD\Λ rõ ràng,

uj > vj,j = 1, , k min{u1, , uk}> min{u1, , uk}

Các bạn thân mến, hình thức phát biểu ngắn gọn GMV có tầm ứng dụng rộng rãi Cứ (hay vài) phép biến đổi T thích hợp ta lại có định lý dồn biến Chúng kết thúc mục hệ GMV, mà xem mở rộng SMV Hệ Cũng xin lưu ý kết có tên SMV UMV tổng quát so với định lý tên mà dẫn ban đầu Hệ 3: (UMV− Undefined Mixing Variables) Cho:

D ⊂ {x= (x1, , xn)∈ Rn|xi ≥ 0,i= 1, , n}, D đóng bị chặn Gọi Λ tập hợp phần tử D có t thành phần nt thành

phần (t≥ 0)

•2 phép biến đổiT1, T2 :DD sau: với phần tửa= (a1, , an)∈

D\Λ, chọn số i 6= j cho ai = min{at > 0, t = 1, , n} vaø

aj = max{a1, , an}, sau thay ai, aj bới α, β ∈ (ai, aj) (ứng với T1) α0 < a

i < aj < β0 (ứng vớiT2)

f :DR liên tục thỏa mãn: f(a)≥min{f(T1(a)), f(T1(a))}, ∀aD

Thì

f(x)≥min y∈Λ

(54)

Chứng minh:

Choïn h1(a) = max{a1, , an} −min{a1, , an} vaø h2(a) = −h1(a),

a = (a1, , an) ∈ D Tương tự hệ 2, ta thay T1 T1∗ = T

n

1 vaø T2∗ =T

n

2 để có: h1(a)> h1(T1∗(a)), h2(a)> h2(T2∗(a)), ∀a= (a1, , an) ∈D

Áp dụng GMV ta có điều phải chứng minh 9 Nhìn lại.

Các bạn thân mến, có lẽ lúc tạm dừng để nhìn lại hành trình vừa qua Như chúng tơi nói trong$6, dồn biến biết đến từ

rất sớm thông qua hàm lồi dẫn đến kết tuyệt đẹp BĐT Jensen xem tiêu chuẩn để dồn biến tâm cách toàn cục Về kết này, bạn tìm đọc cách đầy đủ "Bất đẳng thức" tiếng nhà toán học HardyP olyaLittewood

Trong trường hợp biến, có lẽ quen thuộc với bạn đọc BĐT lượng giác, chẳng hạn như:

sinA+sinB+sinC

√ (1)

cosA+cosB+cosC

2 (2)

với A, B, C cạnh tam giác

BĐT (1) thu cách áp dụng BĐT Jensen cho hàm

lồi BĐT (2) tinh tế hơn, hàm f(x) = −cosxf00(x) =cosx nên

lồi [0, π] Do ta khơng thể áp dụng BĐT Jensen cho biến

A,B,C Tuy nhiên, ta giả sử ABC A, B ∈ [0, π]

nên ta dồn biến A, B Sau tốn cịn

biến trở nên đơn giản

(55)

Các bạn thân mến, dành mục để khảo sát vấn đề dồn biến cho BĐT biến để bạn nắm tư tưởng phương pháp, liệt kê tất kĩ thuật cần thiết Chẳng hạn dồn biến BĐT lượng giác với BĐT tuyệt đẹp Jackgarfulkel (xem phần tập) thú vị Tuy nhiên chúng tơi nghĩ trình bày tất nhàm chán vơ vị, nắm tư tưởng bạn áp dụng trường hợp khác

Đọc xong phần BĐT biến, có lẽ bạn đọc có cảm giác BĐT chuyển trường hợp biến biến đạt giá trị biên Phải nói điều cho hầu hết BĐT mà gặp Tuy nhiên, sau cung cấp cho bạn ví dụ nằm ngồi "thơng lệ" Trong ví dụ này, chí BĐT xét đa thức đối xứng biến Ví dụ lấy từ ý tưởng anh Bùi Việt Anh

Bài toán 1. Cho a, b, c≥0 Khi BĐT:

(a3+b3+c3−6abc)2 + ((a+b+c)3−36abc)2 ≥0

chỉ xảy dấu "=" trường hợp (a, b, c) = (t,2t,3t), t≥0 (và hốn

vị)

Bạn đọc tự kiểm tra điều

Như vậy, bạn yên tâm phương pháp dồn biến có ý nghĩa Với tốn biến thơng thường phải thực lần động tác dồn biến nên phức tạp Trong trường hợp n biến tổng quát việc dồn biến trở nên khó khăn Ngồi BĐT Jensen cho phép dồn lúc n biến (nhưng đáng tiếc, giải lượng nhỏ BĐT) gần ta khơng có cơng cụ khác Trong trường hợp này, thông thường quy nạp ý hay Chúng dẫn ví dụ cho thấy tinh tế chứng minh quy nạp BĐT

Bài toán 2. (Phạm Kim Hùng) Cho n số thực dương a1, a2, , an có tích

bằng Chứng minh với k > thì:

(1 +a1)k

+

(1 +a2)k

+ + (1 +an)k

≥min{1, n

2k}

(56)

làm việc với toán tổng quát hơn:

"Cho n số thực dươnga1, a2, , an có tích s≥1 Chứng minh với k > 0thì:

1 (1 +a1)k

+

(1 +a2)k

+ + (1 +an)k

≥min{1, n

1 +√ns}.

Với tốn tổng qt lại chứng minh quy nạp Thật vậy, xét tốn với n số, ta giả sử an = min{a1, , an} Khi

đó áp dụng giả thiết quy nạp cho (n−1) số a1, a2, , an−1 có tích ≥ 1, ta

đưa tốn biến Cơng việc cịn lại khảo sát hàm biến

Một kĩ thuật khác để đưa BĐT n biến biến dồn biến

giá trị trung binh $7 Như ra, ý tưởng cách dồn

dựa cách dồn biến giá trị trung bình cho hàm lồi Đây cách dồn biến tốt có tính hữu hạn Tuy nhiên, áp dụng cho cực trị đạt tâm

Bây ta phải đối mặt với khả cực trị đạt tâm biên Rõ ràng khả dồn biến khơng cao Do hi vọng vào điều tốt có cách dồn biến toàn cục, BĐT Jensen Với mục tiêu đó, định lý tuyệt đẹp phải kể đến định lý SMV (dồn biến mạnh) UMV (dồn biến khơng xác định) Hai định lý nói "anh em song sinh" SMV dùng để "chuyên trị" BĐT cực trị đạt tâm, cải tiến đáng kể khơng cần dồn biến mà cần dồn biến lớn biến nhỏ UMV địi hỏi giả thiết đặt lên biến bất kì, nhiên cho phép ta dung hịa trường hợp cực trị đạt tâm biên dạng tổng quát Để cho hình thức đơn giản, định lý xét cho hàm đối xứng

Chúng quan sát kết nhận thấy không cần thiết việc tách rời trường hợp, tìm kết hội tụ đầy đủ ưu điểm định lý Tuy nhiên trường hợp riêng Hệ $8 Định lý GMV không đơn tổng quát định lý

kể trên, mà mở chân trời với kiểu dồn biến Một điều kì lạ đòi hỏi: x = (x1, x2, , xn) chưa rơi vào

trường hợp "tới hạn" (tức thuộc Λ), ln thay

(57)

nhỏ nhất, dẫn đến cảm nhận rõ ràng n biến tiến giá trị trung bình, trường hợp bổ đề dãy số khơng cịn tác dụng Tuy nhiên, kết

Các bạn thân mến, bạn chúng tơi hành trình, mà chúng tơi chọn tốt khơng phải đầy đủ Có nhiều vấn đề chúng tơi khơng đưa ra, khơng trình bày kĩ, chúng tơi không coi trọng đầy đủ Cái mà coi trọng cố gắng để bạn thấy vấn đề cách nhanh chóng, rõ ràng hợp lý Hi vọng với tư tưởng mà khơi gợi bạn đủ cảm hứng khả để tiếp bước đường sáng tạo

Cuối cùng, muốn gửi lời cảm ơn đặc biệt tới anh Phan Thành Nam anh Phạm Kim Hùng, người có nhiều kết ý tưởng sử dụng Chúng xin chân thành cảm ơn tất tác giả tốn, nguồn trích dẫn, có thầy Phạm Văn Thuận −

người cung cấp cho chúng tơi tài liệu dồn biến có giá trị 10 Bài tập.

Sau số tập dành cho bạn đọc Hi vọng bạn tìm nhiều niềm vui thử sức với chúng (ghi chú: Θbài dễ, Φbài trung bình, Ξ khó, Ψ cực khó)

Θ Bài tập 1: (Asian Pacific Math.2004) Giả sử a, b, c số dương tùy ý

Chứng minh BĐT

(a2+ 2)(b2 + 2)(c2+ 2)≥9(ab+bc+ca)

Θ Bài tập 2: (MOSP 2001) Chứng minh a, b, c số dương có

tích ta có BĐT

(a+b)(b+c)(c+a)≥4(a+b+c−1)

Θ Bài tập 3: Choa, b, c không âm thỏa mãn a2+b2+c2 = Chứng minh

raèng

(58)

Θ Bài tập 4: (Huỳnh Tấn Châu) Cho x, y, zx+y+z = 1.Chứng

minh raèng:

x3+y3+z3+ 6xyz

Θ Bài tập 5: Chứng minh x, y, z số thực không âm thỏa

mãn điều kiện x2+y2+z2 = ta có BĐT: 7(xy+yz+zx)≤12 + 9xyz

Φ Bài tập 6: (Chọn đội tuyển Việt Nam 1996) Cho a, b, c số thực bất

kì, chứng minh rằng:

F(a, b, c) = (a+b)4+ (b+c)4+ (c+a)4− 7(a

4

+b4+c4)≥0

Φ Bài tập 7: (Phạm Văn Thuận−Zhao Bin) Giả sử x, y, z ba số thực

khơng âm có nhiều số Chứng minh

1

x3+y3 +

1

y3+z3 +

1

z3+x3 ≥

20 (a+b+c)3

Φ Bài tập 8 (Phạm Kim Hùng) Chứng minh với số thực a, b, c

không âm ta có BĐT:

1 √

4a2+bc+

1 √

4b2+ca+

1 √

4c2+ab

4

a+b+c

Φ Bài tập 9: (Murray Klamkin) Chứng minh với số thực khơng

âm a, b, c có tổng 2,

(a2+ab+b2)(b2+bc+c2)(c2+ca+a2)≤3

Ξ Bài tập 10: (Tổng quát RMO2000) Cho a, b, c≥0và a+b+c= Tìm

hằng số k > 0nhỏ cho BĐT sau đúng:

ak+bk+ckab+bc+ca

(59)

Chứng minh rằng:

1

a2−bc+ 1 +

1

b2−ca+ 1 +

1

c2−ab+ 1 ≤3

Ξ Bài tập 12: (mathlinks) Cho a, b, c≥0 vàab+bc+ca= Chứng minh

raèng:

1 +a2b2

(a+b)2 +

1 +b2c2

(b+c)2 +

1 +c2a2

(c+a)2 ≥

5

Ξ Bài toán 13 Cho a, b, c∈[p, q] với 0< pq Tìm giá trị lớn của: a

b+c+ b c+a +

c a+b

Bài toán 14.(Jackgarfulkel) Cho tam giác nhọn ABC Chứng minh rằng:

Φ a)

sinA

2 +sin

B

2 +sin

C

2 ≥

3(1 +sin

A

2sin

B

2sin

C

2) Φ b)

cosA

2 +cos

B

2 +cos

C

2 ≥ √

3(1 +cos

A

2cos

B

2cos

C

2) Φ Bài toán 15.(Jackgarfulkel) Cho tam giác ABC Chứng minh rằng:

cos(AB

2 ) +cos(

BC

2 ) +cos(

CA

2 )≥ √

3(sinA+sinB +sinC) ΞBài toán 16 (Phan Thành Nam) Cho ba sỗ thựcx, y, zkhơng âm có tổng

bằng Chứng minh p

x+y2+py+z2 +√z+x2≥ 2

Ξ Bài tập 17 (Vasile Cirtoaje) Xét ba số thực không âm a, b, c thỏa điều

kiện a2+b2+c2 = 1. Chứng minh rằng:

1 1−ab+

1 1−bc +

1 1−ca

(60)

ΞBài tập 18: (Phan Thành Nam) Choa, b, c≥0và thỏa mãna+b+c=

Chứng minh rằng: a) (VMEO1)

r

a+ (bc)

2

12 + r

b+ (ca)

2

12 + r

c+(ab)

2

12 ≤ √

3

b) p

a+k(bc)2+pa+k(bc)2+pa+k(bc)2 ≤√3

Trong k = 1−

Ξ Bài toán 19 (Phan Thành Việt) Cho tam giác ABC có độ dài cạnh

BC =a, CA=b, AB =c Gọi p nửa chu vi tam giác vàma, mb, mc

là độ dài ba đường trung tuyến tương ứng hạ từA, B, C xuống cạnh đối

diện Chứng minh rằng:

ma+mb+mc ≤ r

3p2+

2[(ab)

2+ (bc)2+ (ca)2]

Bài tập 20: (Phan Thành Nam) Cho x, y, z ∈ [−1,1] vaø x+y+z =

Chứng minh

Ξ a) p

1 +x+y2+p1 +y+z2 +

1 +z+x2 ≥3

Ψ b) r

1 +x+7 9y

2+

r

1 +y+ 9z

2+

r

1 +z+ 9x

2 ≥3

Φ Baøi tập 21 (Phạm Kim Hùng) Cho x, y, z, t ≥ vaø x+y +z +t = 4.

Chứng minh rằng:

(1 + 3x)(1 + 3y)(1 + 3z)(1 + 3t)≤125 + 131xyzt

(61)

ΦBài tập 23(Phạm Văn Thuận−Nguyễn Anh Tuấn) Xét số thựca, b, c, d

thỏa mãn a2+b2+c2+d2 = 1.Chứng minh

1 1−ab+

1 1−bc +

1 1−cd +

1 1−da +

1 1−db +

1

1−ca ≤8

Ξ Bài tập 24 (Phạm Kim Hùng) Cho số thực không âm a, b, c, d, k

tổng 4. Chứng minh

(abc)k + (bcd)k + (cda)k+ (dab)kmax{4,(4 3)

3k }

Ξ Bài tập 25 (Phan Thành Nam)

Cho số thực x, y, z, tthỏa: max{xy, yz, zt, tx} ≥1 Chứng minh p

1−xy+y2+p1−yz+z2+√1−zt+t2+√1−tx+x2

≥p16 + (xy+zt)2

Ψ Bài tập 26 (Phan Thành Nam) Cho số thực x, y, z, t ∈ [−1,1] thỏa

mãn x+y+z+t= Chứng minh p

1 +x+y2 +p1 +y+z2+

1 +z+t2 +

1 +t+x2 ≥4

(*Ghi chú: Bài xuất phát từ trường hợp ba số 20a, dĩ nhiên

sẽ khó nhiều BĐT tương tự với số khơng cịn nữa)

Φ Bài tập 27 (Vasile Cirtoaje) Chứng minh a1, a2, , an khơng

âm có tổng n

(n−1)(a21+a

2+ +a

n) +na1a2 ann2

Ξ Bài tập 28: (Phạm Kim Hùng) Giả sử a1, a2, , an số thực khơng

âm có tổng n Tìm gtnn biểu thức

S=a21+a

2+ +a

n+a1a2 an(

a1

+

a2

+ +

(62)

Ξ bài tập 29 Tìm số dương km tốt để BĐT sau với

mọi dãy số thực không âm x1, x2, , xn có tổng n

(1 +mx1)(1 +mx2) (1 +mxn)≤(m+ 1)n+km(x1x2 xn−1)

trong m số dương

Bài tốn 30 (Phan Thành Việt) Cho a1, a2, , an, s, k số thực dương

thỏa mãn: a1a2 an=snn−1 = (1+ns)k Xét BĐT:

(1 +a1)k

+

(1 +a2)k

+ + (1 +an)k

n−1 Θ a) Chứng minh BĐT nói chung khơng

(63)

-NGUYỄN ANH CƯỜNG - A Li gii thiu

Một lần tơi lại có dịp gặp lại bạn với phương pháp chứng minh bất đẳng thức Nếu phương pháp phương hố khơi dậy ta thích thú thỏa thuê hàng trăm bất đẳng thức khó ngã rạp trước sức mạnh tơi tin bạn hạnh phúc với phương pháp Các bạn tin khơng, trước phải cực khổ lấy giấy nháp biến đối giải tốn với lướt nhìn Nào thưởng thức viên kim cương cắt bánh chưng J

B Phương pháp ABC

Tôi xin mởđầu phương pháp việc xét số toán sau:

Bài 1:

Cho ab+bc+ca=1

i) a+b+c=m,m∈[−∞,− 3] [∪ 3,+∞] Tìm điều kiện abc cho a,b,c số thực ii) a+b+c∈[ 3,+∞],a,b,c≥0 Tìm điều kiện abc cho a,b,c số thực không âm

Giải:

Chúng ta có hai đại lượng trung bình a,b,c Sự xuất abc khiến liên tưởng tới định lý Viete, ta nghĩ tới việc xét phương trình;

(*)

3 − + − =

abc X mX X

Yêu cầu đề tương đương với việc, tìm điều kiện abc để i) Phương trình (*) có ba nghiệm thực

ii) Phương trình (*) có ba nghiệm khơng âm Đặt f(X)= X3 −mX2 +Xabc

Ta có: f'( )X =3X2 −2mX +1.Phương trình có hai nghiệm

3 ;

3

3

2

1

− −

= −

+

= m m X m m

X

X −∞ X2 X1 +∞ f'( )X + - +

f( )X

Phương trình có ba nghiệm f( )X2 ≥0, f( )X1 ≤0

Từđây suy ra: ( ) (1)

9

)

( 1

2

m X m abc

m X

m + −

≤ ≤ − +

(64)

Câu ii) , nhận xét để a,b,clà số thực dương ngồi việc phải thoả mãn ( )1 , abc chịu thêm ràng buột 0≤abc, ngược lại với (1),abc≥0,a+b+c≥0,ab+bc+ca≥0 a,b,c≥0 Vậy nên đáp số là:

( ) (2)

9 ) ( , max 2 m X m abc m X

m + −

≤ ≤       + −

Như ta hoàn thành hai câu hỏi nêu toán

- Bài tóan giúp ta rút hai nhận xét sau:

Nhận xét i)

Ø Điều kiện cần đủđể tồn số thực a,b,c biết trước giá trị ab+bc+ca=1 [−∞− ] [∪ +∞]

∈ =

+

+b c m,m , 3,

a ( )

9 )

( 1

2 m X m abc m X

m + −

≤ ≤ − +

Ø Điều kiện cần đủđể tồn số thực không âm a,b,c biết trước giá trị

1 = + +bc ca

ab a+b+c∈[ 3,+∞] ( )

9 ) ( , max 2 m X m abc m X

m + −

≤ ≤       + −

o Nhận xét suy ta trực tiếp từ toán nêu, ý a+b+clại bị ràng buộc chạy đoạn Có hai cách giải thích sau:

• (a+b+c)2 ≥3(ab+bc+ca)=3

f'(X)=3X2 −2mX +1 buộc phải khơng hồn tồn dương, hay nói cách khác phương trình f'(X)=0phải có nghiệm, tức ∆' =m2 −3≥0

o Nhận xét cho ta thêm điều gì, thay phải sử dụng (a,b,c) với a,b,cR để

biễu diễn tất phần tử tập R3 thõa ab+bc+ca=1, ta sử dụng

(a+b+c,ab+bc+ca,abc) với ràng buộc a+b+c abc nhưđã nêu Cũng hoàn toàn tương tự ta muốn biễu diễn tất phần tử tập R+3 thoã ab+bc+ca=1

Nhận xét ii)

Ø a.Với số thực (a0,b0,c0) tìm hai (x0,x0,y0) (; z0,z0,t0) cho

0 0 0 0 0 0 0 0 0 0 0 0 0 0 0 0 0 0 * * * t z z c b a y x x z t t z z z x y y x x x a c c b b a t z z y x x c b a ≤ ≤ + + = + + = + + + + = + + = + +

Đẳng thức xảy hai ba biến (a0,b0,c0)

Ø b.Với số thực khơng âm (a0,b0,c0) ta tìm hai (x0,x0,y0) (; z0,z0,t0) (0,x0,y0) (; z0,z0,t0) thõa mãn điều kiện sau

0 0 0 0 0 0 0 0 0 0 0 0 0 0 0 0 0 0 * * * t z z c b a y x x z t t z z z x y y x x x a c c b b a t z z y x x c b a ≤ ≤ + + = + + = + + + + = + + = + +

Đẳng thức xảy hai ba biến (a0,b0,c0) Hay 0 0 0 0

*a b c x y z z t

(65)

Đẳng thức xảy ba biến (a0,b0,c0)

o Nhận xét hai khơng hiển nhiên nguồn gốc phương pháp Nhận xét có điều thú vị, ý biểu thức đối xứng f(a,b,c) theo ba biến a,b,c biễu diễn thành g(A,B,C) thông qua dại lượng trung bình

§ A=a+b+c § B=ab+bc+ca § C =abc

Do theo lẽ thơng thường với suy nghĩ giảm số biến, ta cốđịnh A,B cho C chạy Ta mong đợi hàm g đạt cực trị C đạt giá trị biên Tuy nhiên ta khơng cần biết cách cụ thể C sẽđạt gía trị biên nào, ta cần biết cách trừu tượng Cđạt giá trị biên a,b,c có hình thù sao, nhận xét cho ta lời giải cho câu hỏi

o Bây sẽđi vào việc chứng minh chi tiết nhận xét 2:

a) Trước hết ta chứng minh toán với số thực (a0,b0,c0) thỏa mãn: a0 +b0 +c0 =m 0 0

0b +b c +c a =

a Thơng qua tốn 1, với ký hiệu X1,X2 giữ nguyên, ta có:

( m )X m Max

c b a m X m

Min= + − ≤ ≤ + − =

9 ) ( 0 2

Bây xét thử xem a0b0c0 đạt giá trị biên hình thù a0,b0,c0 Xét phương trình f( )X = X3 −mX2 +XMin=0(*) Ta có:

( )

' X = X2 − mX +

f có hai nghiệm

3 ; 3 2 − − = − +

= m m X m m

X nhưđã nêu lên

bài toán Hơn f( )X2 =0, hay nói cách khác f( )x =0 ( ) ' =

x

f có nghiệm X2 Vậy nên phương trình f( )X =0 phải có nghiệm kép, giả sử nghiệm kép x0,x0 nghiệm lại

0

y Như theo định lý Viet ta có :

0 0 0 0 0 0 0 0 0 0 0 0 * * * c b a Min y x x a c c b b a x y y x x x c b a m y x x ≤ = + + = = + + + + = = + +

Chứng minh hoàn toàn tương tự với tồn (z0,z0,t0)

Như ta chứng minh toán nêu trường hợp a0 +b0 +c0 =m 0 0

0b +b c +c a =

a Với tư tưởng hoàn toàn tương tự, bạn đọc chứng minh tồn cho trường hợp: a0 +b0 +c0 =m a0b0 +b0c0 +c0a0 =−1

Bây giả sử a0 +b0 +c0 =M a0b0 +b0c0 +c0a0 =±N liệu (x0,x0,y0)

(z0,z0,t0) có tồn khơng Câu trả lời có, thực vậy, xét số        = N c N b N a c b

a 0

1

1, , ) , ,

( thỏa

mãn điều kiện:

N M c b

a1 + 1+ 1 = ,a1b1 +b1c1+c1a1 =±1 Như theo chứng minh

(66)

( )

( )

( ) ( ) ( ) ( ) 1 0

3 1 0 1 1 0 1 1 1 0 0 0 1 1 1 0 0 0 0 0 0 1 1 1 0 * ) ( * ) ( * t z z t z z N c b a N c b a c b a N y x x N y x x N N z t t z z z N z t t z z z x y y x x x N x y y x x x c b a M N M N t z z t z z N z x x N y x x = ≤ = = ≤ = ± = ± = + + = + + = + + = + + + + = = = + + = + + = + + = + +

Như ta chứng minh hoàn chỉnh câu ( )a Ý tưởng chứng minh câu ( )b hoàn toàn tương tự xin nhường cho bạn đọc

-

Bài 2:

Mọi đa thức f đối xứng theo biến a,b,c biễu diễn dạng đa thức theo biến

c b a ca bc ab

abc, + + , + + Và ( ) ( )

3 deg

deg abcf

Giải:

Nhận xét ta cần chứng minh biễu diễn cho dạng đa thức sau, đa thức đối xứng có thểđược biễu diễn thông qua kết hợp dạng phép nhân thêm hệ số +,−

(elementary operation): ( ) ( ) n p m p n m n p m p n m n p m p n m n m n m m n n m m n n m n n n b a c b a c c a b c a b c b a c b a p n m III c a a c b c c b b a b a n m II c b a n I + + + + + = + + + + + = + + = ) , , (

, mnp≥0

Tuy nhiên, nhận xét

Ø IIIcó thể biễu diễn qua II với abcnhư sau: C =( )abc pII(mp,np) Ø II biễu diễn qua I sau: II = I( ) ( ) (m I nI m+n)

Như ta cần chứng minh I biễu diễn qua abc,ab+bc+ca,a+b+c Ta chứng minh điều phương pháp quy nạp:

Với n=0,1, mệnh đềđã cho hiển nhiên

Giả sử ta chứng minh I( )k biễu diễn thành đa thức thông qua biến

K k c b a ca bc ab

abc, + + , + + ,∀ ≤ Nhận xét điều II( )m,n III(m,n,p) K

n m p

n

m≥ ≥ ≥ + ≤

∀ 0:

Bây ta chứng minh tính biễu diễn I(K+1) Ta có:

(K 1) (a b c) ( ) ( )I K II K,1

I + = + + −

Đồng thời: II( ) (K,1 = ab+bc+ca) (I K−1)−III(K −1,1,1)

Do đó: I(K+1) (= a+b+c) ( )I K −(ab+bc+ca)I(K−1)+III(K −1,1,1)

Mặt khác, theo giả thiết quy nạp, biểu thức I(K),I(K −1),III(K −1,1,1)đều biễu diễn dạng đa thức theo biến abc,ab+bc+ca,a+b+c Điều cho ta kết luận tính đắn mệnh đềđã nêu

Như vậy, mệnh đềđã nêu chứng minh thông qua nguyên lý quy nạp

Tính chất ( ) ( )

3 deg

deg abcf suy hiển nhiên, lẽ biễu thức abccó bậc biến

c b

(67)

Bạn đọc hiểu đa thức tính theo bậc abcnhư sau Giả sử:

(a b c) (a b c)abc ab bc ca

f , , = + + + + + vốn đa thức bậc theo a,b,c Nhưng tính bậc đa thức theo biến abc, ta xem a+b+c ab+bc+ca số m, Khi n đa thức viết lại là:

(a b c) ( )g abc mabc n

f , , = = + đa thức bậc theo biến abc

-

Thông qua hai tốn trên, có đầy đủ kết cần thiết (background) để bước vào giới ABC, Abstract Concreteness J

Chứng minh:

Cả ba định lý chứng minh thông qua nhận xét ii)

Định lý 1:

• Với số (a0,b0,c0)∈R3 tìm hai (x0,x0,y0) (; z0,z0,t0) cho

0 0 0 0 0 0 0 0 0 0 0 0 0 0 0 0 0 0 * * * t z z c b a y x x z t t z z z x y y x x x a c c b b a t z z y x x c b a ≤ ≤ + + = + + = + + + + = + + = + +

Đẳng thức xảy hai ba biến (a0,b0,c0)

• Với số thực khơng âm (a0,b0,c0) ta tìm hai

(x0,x0,y0) (; z0,z0,t0) (0,x0,y0) (; z0,z0,t0) thõa mãn điều kiện sau

0 0 0 0 0 0 0 0 0 0 0 0 0 0 0 0 0 0 * ) ( * * t z z c b a y x x z t t z z z x y y x x x a c c b b a t z z y x x c b a ≤ ≤ + + = + + = + + + + = + + = + +

Đẳng thức xảy hai ba biến (a0,b0,c0) Hay

Định lý 1: Nếu f(abc,ab+bc+ca,a+b+c) hàm đơn điệu R theo abc cực đại cực tiểu xảy ba số a,b,c có hai số nhau, cịn tập R+ xảy có số hay có hai số

Định lý 2: Nếu f(abc,ab+bc+ca,a+b+c) hàm lồi R theo abc cực đại xảy ba số

c b

a, , có hai số nhau, cịn tập R+ xảy có số hay có hai số

(68)

0 0 0 0 0 0 0 0 0 0 0 0 0 0 0 * ) ( * * t z z c b a z t t z z z y x a c c b b a t z z y x c b a ≤ ≤ + + = = + + + + = + + = + +

Đẳng thức xảy hai ba biến (a0,b0,c0) Ø Cách 1: (Direct Proof)

Do f hàm đơn điệu theo biến a0b0c0 nên hàm sốđạt cực đại hay cực tiểu điểm biên a0b0c0, giả sử f tăng ta cần tìm cực đại (các trường hợp lại chứng minh tương tự), ta có

( )

( 0 0 0 0 0 0) ( 0 0 0 0 0 0)

0 0 0 0 0 0 , , , , , , t z z t z t z z z t z z f c b a a c c b b a t z z f c b a a c c b b a c b a f + + + + = + + + + ≤ + +

Vậy nên cực đại xảy có hai biến

Trong trường hợp a,b,cR+, trường hợp tăng tìm cực đại, ta chứng minh hoàn toàn tương tự Trong trường hợp tăng tìm cực tiểu (trường hợp giảm chứng minh tương tự), cốđịnh

ca bc ab c b

a+ + , + + lúc abc đạt cực tiểu có hai biến mà đơi có biến Do đó:

( )

( 0 0 0 0 0 0) ( 0 0 0 0 0 0)

0 0 0 0 0 0 , , , , , , y x x x y y x x x y x x f c b a a c c b b a y x x f c b a a c c b b a c b a f + + + + = + + + + ≥ + +

hoặc

( )

( 0 0 0 0 0) ( 0 0)

0 0 0 0 0 0 , , , , , , x x y x f c b a a c c b b a f c b a a c c b b a c b a f + = + + + + ≥ + +

Vậy nên cực tiểu xảy có hai biến biến Kết hợp trường hợp ta rút kết luận nhưđịnh lý

Ø Cách 2: (Contradiction Proof)

Ta chứng minh cho trường hợp tăng tìm cực đại Giả sử hàm sốđạt cực đại điểm (a0,b0,c0) a0,b0,c0 khác đôi cực đại M Tuy nhiên lại tồn (z0,z0,t0) thoã mãn:

( )

( 0 0 0 0 0 0) ( 0 0 0 0 0 0)

0 0 0 0 0 0 , , , , , , t z z t z t z z z t z z f c b a a c c b b a t z z f c b a a c c b b a c b a f M + + + + = + + + + < + + =

Điều vô lý cho phép ta kết luận cực đại xảy có hai biến

Trong trường hợp a,b,cR+, ta lại xét trường hợp tăng tìm cực tiểu, hàm sốđạt cực tiểu điểm (a0,b0,c0) a0,b0,c0 khác đơi khơng có biến 0, đặt cực đại M Một hai trường hợp sau xảy ra:

( )

( 0 0 0 0 0 0) ( 0 0 0 0 0 0)

0 0 0 0 0 0 , , , , , , y x x x y y x x x y x x f c b a a c c b b a y x x f c b a a c c b b a c b a f M + + + + = + + + + > + + = ( )

( 0 0 0 0 0) ( 0 0)

0 0 0 0 0 0 , , , , , , x x y x f c b a a c c b b a f c b a a c c b b a c b a f M + = + + + + > + + =

Điều vô lý cho phép ta kết luận cực đại xảy có hai biến biến

(69)

Từ kết ta rút số hệ lí thú sau:

Chứng minh:

Hệ 1:

Đa thức bậc mx+ y hàm đơn điệu Do đó, theo định lý hàm f(a+b+c,ab+bc+ca,abc), đa thức bậc theo abc, đơn điệu nđạt cực đại cực tiểu tập R có hai biến nhau, tập R+ có hai biến hay số

Hệ 2:

Tam thức bậc hai với hệ số dương m2x2 +nx+ p

hàm lồi đoạn liên tục Do theo định lý hàm số f(a+b+c,ab+bc+ca,abc), tam thức bậc hai theo abc hệ số bậc cao dương, tức hàm lồi nên đạt cực đại tập R có hai biến nhau, tập R+ có hai biến hay số

Hệ 3:

Theo toán số 2, đa thức đối xứng ba biến a,b,cbậc bé hay biễu diễn thành đa thức f(a+b+c,ab+bc+ca,abc) đa thức bậc theo abc (do ( )

3

deg )

deg(abcf = suy

( )

degabc = ) Do theo hệ đa thức đạt cực đại cực tiểu tập R có hai biến nhau, tập R+ có hai biến hay số

Hệ 4:

Theo toán số 2, đa thức đối xứng ba biến a,b,cbậc bé hay biễu diễn thành đa thức f(a+b+c,ab+bc+ca,abc) tam thức bậc hai theo abc (do ( )

3

deg )

deg(abcf = suy

( )

deg abc = ), hệ số a2b2c2 lại không âm nên theo hệ ta đến kết luận âm đa th ức

H qu 1: Hàm số f(a+b+c,ab+bc+ca,abc) đa thức bậc theo abc đạt cực đại cực tiểu tập R có hai biến nhau, tập R+ có hai biến hay số

H qu 2: Hàm số f(a+b+c,ab+bc+ca,abc) tam thức bậc hai theo abc hệ số bậc cao dương đạt cực đại tập R có hai biến nhau, tập R+ có hai biến hay số

H qu 3: Mọi đa thức đối xứng ba biến a,b,cbậc bé hay đạt cực đại cực tiểu tập

R có hai biến nhau, tập R+ có hai biến hay số

(70)

Các hệ thật vũ khí lợi hại Với chúng, ta có thể tóm gọn phần lớn bất

đẳng thức đối xứng ba biến , vốn thể loại thường xuyên kì thi học sinh giỏi

C ABC ng dng

Bây xét qua số ví dụ cụ thể xem phương pháp vận dụng J

Giải:

P sẵn dạng f(x+ y+z,xy+yz+zx,xyz), điều kiện đối xứng không ràng buộc xyz mà phụ thuộc vào x+y+z,xy+ yz+zx (*) Vậy nên ta có thểđưa tóan việc giải quyết:

Cho 2a2 +b2 =9

Tìm giá trị lớn P=4a+2ba2b Để tìm giá trị lớn ta thay

2

9

b

a= − vào P, cần tìm giá trị lớn của:

( ) ( ) ( ) 9 2

2 b b

b b b

f

P= = − + − −

( )

( ) 87 87 ( 1)(9 78 9)

2

4 2

2

' − + = ⇒ − + − = ⇔ − − + =

− −

= b b b b b b b

b b b

f

Thay nghiệm phương trình đạo hàm vào f ta nhận fmax =10 a=2,b=−1 (*) Lý luận giúp ta áp dụng ABC có thểđược giải thích theo hai cách sau:

- Trong ABC, ta quan tâm đến abc, cịn ab+bc+ca,a+b+c ta coi số Do chúng bị rang buộc không quan trọng

- Chúng ta chuẩn hố tốn thành: Tìm giá trị lớn của:

( )( )

( 2 2 2)3

2 2 27 z y x xyz z y x z y x P + + − + + + +

= , đa thức bậc tính theo biến

abc, x2 + y2 +z2 xy+ yz+zx,x+ y+z Bài [Sưu Tầm]

Cho x,y,zR thỏa x2 + y2 +z2 =9 Tìm giá trị lớn P=2(x+y+z)−xyz

Bài [Nguyễn Anh Cường]

Cho số thực dương a,b,c Chứng minh rằng:

(71)

Giải:

i) Bất đẳng thức rõ ràng viết dạng đa thức đối xứng bậc

( ) ( ) ( )( )

3

2 3 3 3

2

2 + + + + + − + + + + ≥

=abca b c a b c a b c ab bc ca

P

Và ta cần xét cực tiểu có hai giá trị ba biến hay biến Trường hợp hai biến nhau, giả sử a=c bất đẳng thức tương đương với:

( ) ( ) ( ) ( )

2 2 2 2 3 2 2 2 3 ≥ + − ⇔ ≥       + + − + − ⇔ + + ≥ +

+ a b a b a b

b a b a b a b a ab a b a b a

Trường hợp có số 0, giả sử c, bất đẳng thức tương đương với:

( )

3

2 2

2

2 + ⇔ + + − ≥

a b a b

b a

ab

ii) Một đa thức đối xứng bậc bảy, bạn đừng lo, đa thức bậc abc J, theo ta lại áp dụng ABC trường hợp Ta xét hai trường hợp:

Trường hợp hai biến nhau, giả sử a=c bất đẳng thức tương đương với:

( ) ( ) ( ) ( )

( )

( ) [((2 ) )]

2 2 4 2 4 2 2 2 2 2 2 2 2 3 2 2 3 ≥ + − − ⇔ + + + − ≥ + − ⇔ −     + + ≥     + − ⇔     + + ≥ + + a a b b a ab a ab b a b a b a b a b a ab a b a b a b a ab a b a b a b a

Trường hợp có biến 0, bất đẳng thức cho hiển nhiên

Bài tóan Iran 96 tiếng, đa thức đối xứng bậc bậc hai abc (**):

( )( )( )

[ ] 4( )([ ) ( ) ( ) ( ) ( ) ( ) ] a+b b+c c+aab+bc+ca a+b b+c + b+c c+a + c+a a+b

Vậy nên hàm sốđạt cực đại có hai giá trị hay số Trường hợp có hai biến nhau, bất đẳng thức tương đương với

( ) ( ) ( ) ( ) ( ) ( )

2

2 2 2 2 2

2 ≥ ⇔ − ≥

      + − + + − ⇔ ≥     + +

+ b a b

b a b a a b a b a b a a ab a

Trường hợp có biến nhau, giả sử c, bất đẳng thức tương đương với:

( ) ( ) 4( ) ( ) (4 )

1 1

1 2

2

2

2 ≥ ⇔ − + + ≥

     + − − ⇔ ≥     + +

+b a b a b ab a b a b a b ab

a ab

Bài [Iran Olympiad 1996]

Chứng minh bất đẳng thức sau với số thực dương a,b,c

( ) ( ) ( ) ( ) 1 2

2 ≥

(72)

( )([ ) (2 ) (2 ) (2 ) (2 ) (2 )2]

4ab+bc+ca a+b b+c + b+c c+a + c+a a+b chứa abc bậc 1, đa thức bậc 6, thực ta cần xét bậc abctrong [(a+b) (2 b+c) (2 + b+c) (2 c+a) (2 + c+a) (2 a+b)2], vốn đa thức bậc

Còn biểu thức 9[(a+b)(b+c)(c+a)]2 vốn đa thức bậc 6, nhiên (a+b)(b+c)(c+a) chứa nhiều abc bậc 1, bình phương lên hệ số a2b2c2 không âm

Như vậy, ta khơng cần khai triển tồn dang biểu thức nào, ta cần lý luận hệ số a2b2c2 đủđể áp dụng định lý ABC

Bản chất phương pháp đánh giá abc, nên việc gặp tóan có bậc đẹp đẽ điều tốt đẹp lúc vậy, nên thủ sẵn biến đổi sau đểđánh giá hàm số theo abclà điều cần thiết

MOT SO DANG THUC

Để thuận tiện phần tiếp theo, ta quy ước a=x+y+z,b=xy+ yz+xz,c= xyz

Ta xét qua đại luợng hốn vị vịng quang biến x,y,z sẽđược biễu diễn qua đại lượng

( ) ( ) ( ) ( ) ( ) ( ) c c ab y z x x z y z y x ac b b a a z y x c ab a z y x ac b b a x z zx z y yz y x xy c ab x z zx z y yz y x xy b a z y x 2 3 2 4 4 3 3 2 2 2 2 2 2 − = + + + + + + + − = + + + − = + + − − = + + + + + − = + + + + + − = + +

( ) ( ) ( )xy + yz + zx =b2 −2ac

( ) ( ) ( )3 3

3

3abc c

b zx yz

xy + + = − +

( 2 )( 2 2) ( )

6

9c a abc b

zx yz xy x z z y y

x + + + + = + − +

Dưới xin cung cấp cho bạn sốđánh giá a,b,c miền khác nhau:

Định lý 1:

Phương trình bậc ba có nghiệm thực x,y,z −27c2 +(18ab−4a3)c+a2b2 −4b3 ≥0(1)

Định lý 2:

Phương trình bậc ba có nghiệm thực dương x,y,z có ( )1 a>0,b>0,c>0

Định lý 3:

Phương trình bậc ba có nghiệm ba cạnh tam giác có ( ) ( )1, a3 −4ab+8c>0 Các hệ quảđã nêu hữu hiệu gặp nhiều hạn chế

Sau số ví dụ ta làm trực tiếp từ hệ quảđã nêu mà phải nhờ vào số biến đổi hay định lý nêu

Bài [Russia Olympiad 2005]:

Cho a,b,clà số thực dương thỏa mãn:

2

2 + + =

c b

a Chứng minh rằng:

≥ +

+ b c

(73)

Một bất đẳng thức đối xứng, đìêu kiện khơng ràng buột abc, việc đưa vềđa thức đối xứng dễ dàng, nhiên lại lên tới bậc 9, bậc ba theo abc L Điều nằm ngịai kiểm sóat hệ quả, nên ta phải có chút thủ thuậ biến đổi nho nhỏ

Đặt c ab z b ac y a bc

x= ; = ; = Bất đẳng thức tương đương với:

= + +yz xz

xy Chứng minh rằng:

( )* 1 ≥ + + + +

+z yz x zx y

xy

Bây chuyện trở nên dễ dàng rồi, đa thức bậc hai theo xyz với hệ số bậc cao khơng âm, ta cần tìm cực đại Ta cần xét trường hợp có hai biến nhau, hay biến Trường hợp 1: x=z Bât đẳng thức tương đương với:

3

2 + ≥ +

+x x y

xy với

2 = +x xy Thay x x y 1−

= , ta cần chứng minh 3, [ ]0,1

2 1 2 2

2 − ≥ ∈

+ + + − x x x x x

x Điều không khó khăn

và bạn giải dễ dàng

Trường hợp 2: z=0 Bất đẳng thức tương đương với:

1

1 + + ≥

y x

xy với xy=1

Ta có 1+ ≥ ⇒ +1+ ≥ + =3

xy xy y x xy xy y x

Giấc mộng đưa dạng đa thức đối xứng coi tan vỡ Ta đành đánh giá trực hàm biến

xyz Vẫn với quy ước a=x+y+z,b=xy+ yz+zx,c=xyz, sử dụng đẳng thức ta biến bất đẳng thức dạng:

2 2 2 2 2 + ≥ − + − + + − b a b ac b ac b b a a

Hàm theo c hàm bậc nên đơn điệu Theo định lý một, cực tiểu xảy có hai biến hay biến

Trường hợp 1: x=z Bất đẳng thức tương đương với:

( ) 1 2

2 2 2 2 2 4 + ≥ + + + + + y x xy x y x x y x

Do tính bất đẳng thức nên ta giả sử x=1, ta cần chứng minh:

Bài [Nguyễn Anh Cường]:

Cho số thực dương x,y,z Chứng minh :

( ) 1 2

(74)

( )

( )

( )( ) (( )2)(2 1)

1 2 2 2 2 1 2 2 2 2 2 + + + + − ≥ + + + + − ⇔ + + − ≥ − + + y y y y y y y y y y y y

Ta có thểđánh giá mẫu số cách nhẹ nhàng đảm bảo tính đắn dấu lớn sau đánh sau:

( )( ) ( )( )

( 2)(2 1) 2 ( 2)(2 1) 2

2 2 2 2 2 2 2 2 4 + + + ≥ + + + + ⇒ + ≥ + + + + + ≤ + + + + ⇒ + ≤ + + y y y y y y y y y y y y y y y y Do cơng việc cịn lại chứng minh:

( )

(5 2) (5 2 0)

2 2 2 2 2 ≥ + + − + + ⇔ + + + ≥ + + + + y y y y y y y y y

Trường hợp 2: z=0 Bất đẳng thức tương đương với: 2 2 2 4 + ≥ + + + y x xy y x y x

Ta có: ( ) 2

2 2 4

2 x y

y x y

x + ≥ + ≥ , đó:

2 2 2 2 2 2 2 2 2 4 2 2 4 + ≥ + + − ≥     + + + + +     − ≥ + ≥ + + + xy y x y x xy xy y x y x y x y x xy y x y x

Cuối số tập để bạn làm quen với phương pháp này:

Bài tập áp dụng Bài [Sưu tầm]:

Cho số thực dương a,b,c Chứng minh rằng:

ca bc ab c b a ab c ca b bc

a2 + + + + + ≥ + + + + +

6 2

Bài 2:[Darij Grinberg- Old and New Inequality]

Cho a,b,c số thực dương Chứng minh rằng:

( ) ( ) (a b) (a b c)

c a c b c b a + + ≥ + + + + + 2

Bài 3: [Mircea Lascu – Old and New Inequality ]

Cho a,b,c số thực dương Chứng minh rằng:

      + + + + + ≥ + + + + + b a c a c b c b a c b a b a c a c b

(75)

( ) (4 ) (4 )4 ( 4 4)

4

c b a a

c c b b

a+ + + + + ≥ + +

Bài 5:[Nguyễn Anh Cường]

Cho số thực dương a,b,c Chứng minh :

2

2 2 2 2 ≥ +

+ +

+ + +

+ + + + +

c b a

ca bc ab b

a c a

c b c

b a

D.ABC Upgrade

Như phần lướt qua sức mạnh phương pháp này, hy vọng bạn thấy thích thú với tơi trình bày Trong phần sau, lướt qua điểm yếu phương pháp giải chúng cách đáng kể nhất.Việc ngày nâng cấp phương pháp vấn đềđang phát triển, hoan nghênh ý tưởng bạn việc nâng cấp phương pháp

Như bạn thấy, tốn bất đẳng thức có ràng buột biến a,b,c, rang buột liên quan đến a+b+c ab+bc+ca định lý ABC có hội phát huy sức mạnh Thế toán mà thân abcbị ràng buột Với ý tưởng vậy, nâng cấp định lý ABC đểđịnh lý có khả đối phó với tốn thuộc dạng

Chứng minh

i) Giả sử a+b+c=1 abc=m(trường hợp a+b+c=n có thểđưa trường hợp cách dễ dàng cách đặt a=nx,b=ny,c=nz, bạn đọc xem qua kỹ thuật phần chứng minh định lý ABC phần trên.) Ta chứng minh ab+bc+ca đạt giá trị nhỏ lớn có hai ba biến a,b,c hai trường hợp a,b,cR a,b,cR+ Đặt ab+bc+ca=S Một lần nữa, ta lại đưa toán tồn nghiệm phương trình:

Đặt f(X)= X3 −X2 +SXm

Ta có: f'( )X =3X2 −2X +S.Phương trình có hai nghiệm

3 1 ;

3 1

2

S X

S

X = + − = − −

Phương trình có ba nghiệm f( )X2 ≥0, f( )X1 ≤0

Ta có : f( )X2 ≥0⇔(6S−2)X2 +S−9m≥0, giả sử có tập nghiệm RX2

( )X1 ≤0⇔(6S−2)X1+S−9m≤0

f , giả sử có tập nghiệm RX

i) Cho a,b,c đồng thời số thực số thực dương Khi đại lượng abc,a+b+c cho trước (nghĩa cốđịnh sẵn) ab+bc+ca sẽđạt giá trị lớn nhỏ có hai ba biến a,b,c

(76)

Gọi Smin,Smax giá trị nhỏ lớn tập

2

1 X

X R

R ∩ (giao hai tập khác rỗng, nêu khơng phương trình khơng có nghiệm với giá trị S) Nhận xét hai giá trị nghiệm hai phương trình (6S−2)X1+S−9m=0 hay (6S−2)X2 +s−9m=0 (các giá trị chắn tồn tại, khơng phương trình bậc có nghiệm S chạy tới cộng vơ âm vơ cùng), ta có Smin ≤SSmax.Bây ta kiểm tra S đạt hai giá trị liệu có tồn ba số thực a,b,c khơng hình thù a,b,c

Điều rõ ràng,

2

max

min,S RX RX

S ∈ ∩ Điều thứ hai, Smin,Smax nghiệm hai phương trình (6S−2)X1+S−9m=0 hay (6S−2)X2 +s−9m=0, nên f( )X1 =0,

( )X2 =0

f Khi phương trình bậc ba cho có nghiệm kép, hay nói cách khác, lúc hình thù

(a,b,c) (x,x,y)

Bây ta chứng minh cho trường hợp a,b,cR+ Trước hết ta cần phải có m≥0 Nhận xét

2

0∉RXRX , khơng tồn số thực a,b,c thoả mãn a+b+c=1,ab+bc+ca=0,abc≥0 Điều có nghĩa

2

1 X

X R

R ∩ tách biệt thành khoản mà cận âm dương Gọi R3 tập

2

1 X

X R

R ∩ bỏđi đoạn âm Gọi Smin,Smaxlà giá trị nhỏ nhỏ tập R3 Và ta lại có: Smin ≤SSmax Lý luận tương tự trường hợp R, ta suy S chạm biên hai ba biến (a,b,c)

ii) Bạn đọc dễ dàng suy chứng minh cho định lý ABC Upgrade sau xem qua chứng minh Tuy nhiên a+b+c khơng có rang buộc cận a,b,c≥0 Tuy nhiên có cận

c b

a+ + bị chặn Do nên trường hợp tồn Smin, vậy, ta tới kết luận a+b+cđạt giá trị nhỏ hai ba biến a,b,c

Các bạn thấy tư tưởng trừu tượng cụ thể (abstract concreteness) rõ rang chứng minh Chúng ta không cần biết giá trị nhỏ ab+bc+ca hay a+b+c cụ thể chứng minh đạt giá trị hai ba biến a,b,c

Chúng ta lướt qua số ví dụđể xem cách áp dụng định lý ABC Upgrade nào:

Bài 1: [Hojoo Lee] Cho số thực dương a,b,c thoả mãn abc≥1 Chứng minh rằng:

a c c

b b

a+ + + + + + +

+ ≥ 1 1 1

Giải:

Nhận xét hàm số bên phải giảm biến a tăng Do cần chứng minh abc=1(đối với trường hợp , abc=k ≥1, ta có:

1 1 1 1 1 1 1 ≤ + + + + + + + + ≤ + + + + + + + + k a c c b b k a a c c b b a )

Bất đẳng thức cho tương đương với:

(1 )(1 )(1 ) (1 )(1 ) (1 )(1 ) (1 )(1 )

) , ,

(a b c = +a+b +b+c +c+a − +a+b +b+c − +b+c +c+a − +c+a +a+b

f

Nhận xét bậc đại lượng ab+bc+ca f(a,b,c) bậc ( f(a,b,c) bậc theo

c b

a, , ) Vậy nên cốđịnh a+b+c f(a,b,c) đạt giá trị nhỏ ab+bc+ca đạt giá trị nhỏ nhất,

(77)

1 2 1 ≤ + + +

+ x x y

Thay 12

x

y= vào bất đẳng thức ta cần chứng minh: ) )( ( ) ( ) ( 1 2 1 2 ≤ + + + + − − ⇔ ≤ + + +

+ x x x

x x x x x x

Như bất đẳng thức chứng minh hoàn toàn

Bài 2: [Sưu Tầm]: Cho số thực dương a,b,c thoã mãn: a+b+c=3abc.Chứng minh rằng: ) ( ) ( ) (

2 ab a b bc b c ca c a

c ab a bc b ac + + + + + ≥       + +

Bài [Bùi Việt Anh]: Cho số thực dương a,b,c Chứng minh rằng:

a b c a c b c b a + + + +

+ + (a+b)(b+c)(c+a) ≥2

abc

Giải:

Đối với tốn dù có cốđịnh hai đại lượng khơng thu hàm số tốt với biến lại (tốt ởđây ngụ ý lồi, lõm hay đơn điệu) Tuy nhiên ởđây, ta thấy liên hệ hai đại lượng

a b c a c b c b a + + + +

+ (a b)(b c)(c a)

abc

+ +

+ , chúng tổng tích đại lượng

b a c a c b c b a + +

+ , , Như thử chuyển biến theo hướng xem

Đặt a b c z a c b y c b a x + = + = +

= , , , vấn đề mối liên hệ x,y,z Ởđây xin đưa mối quan hệ, với mối quan hệ tìm ngược trở lại a,b,c Và từđó ta có tốn tương đương sau:

Cho x,y,z≥0 thoã mãn: 2

1 1

1

1 = ⇔ + + + =

+ + + +

+x y z xyz xy yz zx Chứng minh rằng:

2

2 ≥

+ +

+ y z xyz

x

Như ta cốđịnh xyz xy+ yz+zx đưa toán sau:

Cho a,b≥0 thỏa mãn: 2a2b+a2 +2ab=1 Tìm giá trị nhỏ của: 2a+b+2a b

Thay ( 1)

2 2 2 ≤ − = + − = a a a a a a

b , ta cần chứng minh:

2 ) ( 2

2 + − + aa

a a

a Thực vậy, bất đẳng thức tương đương với:

(78)

Mặt khác a a a a a a a a a 2 2 ) ( 2 ≥ + − ≥ − = −

Do đó: 2 2

2 2 2 1 2 4 ≥ = ≥ + ≥ − + a a a a a a a

Tóm lại bất đẳng thức chứng minh hoàn toàn

Cuối cùng, phần quen thuộc khơng nên thiếu, phần tập cho bạn áp dụng:

Bài 1:[Sưu tầm]

Cho số thực dương a,b,c Chứng minh rằng:

( xy yz zx)

xyz z y x + + ≥ + + + 3

Bài 2: [Sưu tầm]

Cho số thực dương a,b,c thoã mãn: abc=1 Chứng minh rằng:

1

1+ + + + + + ≥

+

+ a b

c a c b c b a

Bài 3: [Nguyễn Anh Cường]

Cho số thực dương a,b,c thỗ mãn: (a+b)(b+c)(c+a)=8 Tìm giá trị lớn nhỏ của:

c b a abc

P= + + +

E.M ca đóng

Trong phần này, xét qua lớp toán mà biến bị chặn tập đóng, vốn vấn đề cịn chưa phát triển phần vềđịnh lý ABC nói Đối với lớp toán này, có phương pháp khác để giải Tơi giới thiệu phương pháp trước, sau sẽ nghiên cứu cách ứng dụng ABC toán sau:

Gặp tình này, dấu bất đẳng thức thường xảy giá trị biên, nghĩa a hay b Trong trường hợp vậy, ta cố gắng chứng minh

(x x xn) {f(a x xn) (f b x xn)}

f 1, 2, , ≥min , 2, , , , 2, , Cuối ta thu giá trị nhỏ hàm số đạt số giá trị a giá trị lại b Phương pháp gọi phương pháp tiếp cận dấu Ta lướt qua số ví dụ sau:

Bài toán:

Cho x1,x2, ,xn∈[ ]a,b Chứng minh f(x1,x2, ,xn)≥C a,b,C số cho trước

Bài [Thi đội tuyển tốn trường THPT Năng Khiếu] Cho a,b,c∈[ ]0,1 Tìm giá trị lớn của:

(a b c) a b c ab bc ca

(79)

Ta chứng minh f(a,b,c)≤max{f(0,b,c) (,f 1,b,c)}(*) Ta có chứng minh trực tiếp phương pháp đại số sau: (f(a,b,c) (− f 0,b,c)) ((f a,b,c) (− f 1,b,c)) (=c c−1)(1−ab)2 ≤0 , từđó suy ( )*

Hay nhìn giải tích, ta suy nhanh chóng với nhận xét hàm sốđơn điệu theo biến a Và từđó trực tiếp suy ( )*

Hoàn toàn thực bất đẳng thức tương tựđổi với biến b, , ta sc ẽ suy tính chất sau:

{ }

( , , ) ax f(0,0,0);f(0,0,1);f(0,1,1);f(1,1,1)

f a b cm Từđây dễ dàng kết luận giá trị lớn

(a b c)

f , , Dấu xảy chẳng hạn như: a=b=0,c=1

Đối với toán này, ta áp dụng việc chứng minh C(x,y,z)≤max{C(1,y,z) (,C 2,y,z)} gặp nhiều khó khăn Ởđây áp dụng mẹo nhỏđể làm cho công việc tương đối đơn giản hơn:

( ) x z y z y x y x z z y x f C 1 1 1 1 , , + + + + + =

= Đến bạn có thểđốn làm tiếp

theo Đặt

    ∈ = =

= ,1

2 , , ; , , c b a z c y b x

a ta cần tìm giá trị lớn của:

( ) b a c a c b c b a c b a g + + + + + = , ,

Ta xẽ coi b, hc ằng số, a biến :

) ( ) ( ) , , (

'' 3 3 >

+ + + = a b c a c b c b a

g Như g hàm lồi

theo biến a, tức g đạt giá trị lớn a chạm biên

Từđây ta suy điều mong muốn: g(a,b,c)≤max{g(1,b,c) (,g 2,b,c)}

Hoàn toàn thực bất đẳng thức tương tựđổi với biến b, , ta sc ẽ suy tính chất sau:

( )      

≤ ); (1,1,1)

2 , , ( ); , , ( ); , , ( max ,

,b c g g g g

a

g Từđây dễ dàng suy giá trị lớn

g 12 19

, đạt ( ) 

     = ,1,1

2 ,

,b c

a , hay giá trị lớn f 12 19

, đạt (x,y,z)=(2,1,1) Như qua hai ví dụ bạn phần làm quen với phương pháp Ởđây nêu phương pháp khác, chứng minh dựa vào định lý ABC

Trong phần trên, ta thấy ABC chí áp dụng biến chạy hoàn toàn R R+, nên muốn áp dụng ABC, ta phải kéo dãn đoạn cho thành R R+

Bài [Nguyễn Anh Cường]

Cho x y z, , ∈[ ]1, Tìm giá trị lớn biểu thức sau:

xy xz yz

C

xz yz xy yz xy xz

= + +

+ + +

Bài [Thi đội tuyển toán trường PTNK]

[ ]

(80)

Ta quay lại tốn này, ta tìm cách kéo giãn đoạn [ ]0,1 thành R+ Nhận xét nghịch đảo

a a→1, ta thu số nằm đoạn [1,+∞] Vẫn chưa biến thành R+ được, ta lại tiếp tục trừ số cho

1

1 → −

a

a ta thu số nằm đoạn [0,+∞] Như ta chuyển aa−1=x

1

để kéo

[ ] [0,1 → 0,+∞]

Từ ý tưởng trên, ta sẽđi tới lời giải sau: Đặt z c y b x a + = + = + = 1 , 1 , 1

x,y,zR+ Biểu thức cho trở thành:

( ) ( ) ) )( ( ) )( ( ) )( ( 1 1 1 , , , , x z z y y x z y x z y x g c b a f + + − + + − + + − + + + + + = = ( )( ) ( )( ) ( )( ) ) )( )( ( 1 1 1 + + + + + + + + + + + + = + + + − − − − + + + + + + + + = z y x zx yz xy xyz z y x zx yz xy z y x z y x y x x z z y

Đến bạn đọc dễ dàng thấy g(x,y,z)≤1, khơng cần áp dụng thêm định lý ABC nữa, nhiên biểu thức áp dụng ABC cách dễ dàng đểđưa trường hợp hai biến Có hai lợi kéo dã tập đóng thành tập R+ này, thứ ta áp dụng định lý ABCnhư trường hợp trên, thứ hai ta có thểđánh giá dễ dàng hơn, ởđây ta sẽđánh giá biến với số 0, điều lúc dễ dàng Chẳng hạn biểu thức trên, đánh

giá xyz≥0 ≥0

+ + + +

+yz zx x y z

xy

Một lần ta sử dụng kỹ thuật kéo dãn biến Ta sử dụng ABC, trước tiên kéo dãn [ ]1,2 →R+

sau: a

x

x − =

1

Như vậy, đặt

Bài [Olympic 30-4 – Thành phố Hồ Chí Minh] Chox,y,z∈[ ]1,2 Chứng minh rằng:

( )(1 1) 10

(81)

+ ∈ + + = + + = + +

= a b c R

c c z b b y a a

x ; , ,

1 ; ;

Ta cần chứng toán sau:

10 ) )( )( )( )( )( ( 10 2 1 2 ) , , ( ≤ + + + + + + ⇔ ≤       + + + + + + + +       + + + + + + + + = c b a c b a MN c c b b a a c c b b a a c b a f

Ởđó: M =(a+2)(b+1)(c+1)+(a+1)(b+2)(c+1)+(a+1)(b+1)(c+2)

( +1)( +2)( +2)+( +2)( +1)( +2)+( +2)( +2)( +1)

= a b c a b c a b c

N

Thật đáng tiếc bất đẳng thức: MN−10(a+1)(b+1)(c+1)(a+2)(b+2)(c+2)≤0 không thểđánh giá định lý ABC được, hệ số a2b2c2 cuối thu âm (bạn đọc nhẩm dễ dàng) Thế không bỏ cách dễ dàng, nhờ ABC Upgrade để giải trường hợp Cốđịnh abc a+b+c, ta thấy biểu thức thu đa thức bậc theo ab+bc+ca Vậy nên ta đến kết luận bất đẳng thức cho đạt cực trị có hai biến Như ta chì cần chứng minh bất đẳng thức:

( )

( )( )

0 2 2 2 10 2 2 2 , , ≤ + + − ⇔ ≤           + + −       + +           + + −       + + ⇔ ≤       + + + + +       + + + + + = a ab b ab b b a a b b a a b b a a b b a a b a a f

Như toán chứng minh Sau số tập cho bạn áp dụng:

Bài 1[Tạp chí tốn học tuổi trẻ]:

Cho a b c, , ∈[ ]1, Tìm giá trị lớn của: 3

3

a b c

A

abc

+ + =

Tạp chí Tốn Học Tuổi Trẻ

Bài 2: [Sưu tầm]

Cho x,y,z∈[ ]1,2 Chứng minh rằng:

( )  + +  + + + ≥     + + + + y x z x z y z y x z y x z y

(82)

F.Định lý ABC m rng

Như giải hai yếu điểm ABC Yếu điểm thứ ba dễ nhận ABC áp dụng cho tốn ba biến, cịn nhiều biến nào??? Chúng ta giải vấn đề với định lý ABC mở rộng

Trước hết, làm quen với khái niệm nằm định lý ABC

I Khả ABC

1) Định nghĩa

Xét biểu thức ba biến f(a,b,c)

Ta gọi f(a,b,c) biểu thức khả ABC bất đẳng thức f(a,b,c)≥0 có thểđược chứng minh dựa vào định lý ABC đểđưa hai trường hợp sau:

i) Hai biến ii) Một biến

2) Phương pháp chứng minh khả ABC

Để chứng minh biểu thức f(a,b,c) khả ABC, ta chuyển biểu thức f(a,b,c) thành biểu thức g biến A=a+b+c,B=ab+bc+ca,C =abc Biểu thức f(a,b,c) biểu thức khả ABC g(A,B,C) hàm lồi theo biến C

Ví dụ :

Chứng minh biểu thức sau khả ABC

) ( ) ( ) (

) , ,

(a b c a3 b3 c3 abc ab a b bc b c ca c a

f = + + + − + − + − +

Giải:

Đặt A=a+b+c,B=ab+bc+ca,C =abc, ta có:

C AB A

C AB C C AB A

C B A g c b a

f( , , )= ( , , )= −3 +3 +3 − +3 = 3−4 +9

Xét g(A,B,C) theo biến C Ta có: g' =9,g'' =0 g hàm lồi theo biến C Vậy biểu thức f(a,b,c) khả ABC

II Định lý ABC mở rộng

Xét biểu thức đối xứng n biến f(a1,a2, ,an), f có cực tiểu n≥3 Ta coi

(a a an)

f 1, 2, , biểu thức ba biến g(a1,a2,a3) với số a4,a5, ,an coi số

Khi g khả ABC bất đẳng thức f(a1,a2, ,an)≥0 có thểđưa xét hai trường hợp sau: )

i m biến nhau, n-m biến )

ii biến

Bất đẳng thức chứng minh đắn chứng minh tính đắn hai trường hợp

Chứng minh:

Ta giả sử f có cực tiểu giả thiết nêu cực tiểu xảy điểm (x1,x2, ,xn)(*)

Nếu x1x2 xn =0 hay xi có khả nhận hai giá trị cốđịnh cực tiểu khơng âm theo giả thiết nêu định lý

Nếu xi có khả nhận ba giá trị khác khác 0, ta giả sử (x1,x2,x3) mà ba giá trị khác khác đôi Ta cốđịnh biến x4,x5, ,xn số xét hàm

(x x x x x )

(83)

Theo giả thiết g khả ABC, nên đạt cực tiểu có hai số nhau, hay số Điều có nghĩa tồn (a,b,c) để g(x1,x2,x3) (>g a,b,c) hay

(x x x x xn) (f a b c x xn)

f 1, 2, 3, 4, , ≥ , , , 4, , Điều mâu thuẫn với giải thiết (*)

Tóm lại định lý ABC chứng minh

III Ứng dụng ABC mở rộng

Bài toán 1:

Chứng minh bất đẳng thức sau với giá trị thực dương a,b,c,d

( ) 3( ) (*)

1

3 4 4 2 2

cd bd bc ad ac ab d c b a abcd d c b a + + + + + + + + + ≥ + + +

Giải:

Bất đẳng thức tương đương với:

(a +b +c +d )(ab+ac+ad+bc+bd +cd)−4abcd(ab+ac+ad+bc+bd+cd)

3 4 4

( )

12 + + + ≥

abcd a b c d

Đặt: ( )( ) ( ) ) ( 12 ) , , , ( 2 2 4 4 d c b a abcd cd bd bc ad ac ab abcd cd bd bc ad ac ab d c b a d c b a f + + + − + + + + + − + + + + + + + + =

Nhận xét cốđịnh biến d f hàm đối xứng theo ba biến, f khả ABC Như theo định lý ta phải xét trường hợp sau (ta xứ lý với (*)):

0 )a=

i , bất đẳng thức hiển nhiên

ii)a=b= x,c=d = y, bất đẳng thức tương đương với:

( ) ( ) ( ) xy y x y x y x y x y x xy y x y x y x y x 2 ) ( ) ( 2 2 2 2 2 2 4 + + − ≥ − + ⇔ + + + + ≥ +

Áp dụng BDT (x+ y)2 ≥4xy x2 + y2 ≥2xy ta chứng minh trường hợp

y d x c b a

iii) = = = , = , bất đẳng thức tương đương với:

( ) ( ) ( ) ( ) ( ) xy x y x y x y xy x y x xy x y x y x y x 3 3 3 3 3 2 2 2 2 4 + − ≥ + + − ⇔ + + + ≥ +

Áp dụng bất đẳng thức 3x2 +2xy≥3x2 3x2 +2xy+ y2 ≥4xy ta chứng minh trường hợp nỳa Tóm lại theo định lý ta có đìêu phải chứng minh

Bài toán 2:

Cho số thực a1,a2, ,an ≥0 thỏa mãn a1 +a2 + +an =n Chứng minh rằng: 1 2 2 − + ≥ + + + − + + +

+ n n

a a a n n a a

(84)

Cốđịnh a4,a5, ,an ta hàm ba biến khả ABC Như ta cần xét trường hợp:

)a1 =

i Bất đẳng thức hiển nhiên )

ii mx+(nm)y=n Chứng minh:

1 ) ( 2

2 + − ≥ + −

− +

+ n n

y m n mx n n y m n x m

Ta đưa toán:

( )

( )

( )

( )( ) ( )( )

( )

( ( ) )( )

1 2 ) ( ) ( 2 2 2 2 2 ≥ − − − − + ⇔ − + − − − ≥ − − ⇔ − + ≥ − + − + − +     + − − + y x xy n y m n mx y m n n nmx y x m n m n nxy y x m n m n n y m n mx n y m n mx n y m n x m n y m n mx

Bất đẳng thức đắn do: mx2 +(nm)y2 ≥2 m(nm)xy≥2 n−1xy Tóm lại bất đẳng thức chứng minh hoàn toàn

IV Bài tập

Bài 1: Chứng minh bất đẳng thức sau cho số thực dương a,b,c,d

( )       + + + + + + ≥ + + + + d c b a d c b a abcd d c b

a 1 1

14 4 4 Bài 2:

Cho số thực dương a1,a2, ,an thoã: ∑

≤ < ≤ = n j i j ia a

1 Chứng minh

( )           − + − ≥ + +

+ 2

2 1 , n n n n n k n a a ka a a Bài 3:

Chứng minh x1,x2, ,xn số thực thoã: 2

2

1 +x + +xn =

x ta có bất đẳng thức sau:

( 2)( 1)

6

1 2 3

3 ≤ + − + + +

+ ∑x x x

n n

x x

x n

G Kết lun

(85)

GEOMETRIZE ALGEBRA (GLA) LỜI MỞ ĐẨU

Trong trào lưu bất đẳng thức phát triển vũ bão loạt phương pháp ffầy giá trị tên tuổi tiếng bạn say mê bất đẳng thức đời thif việc phương pháp không thật bật cho dù mạnh trở nên nhạt nhịa bị lãng qn chẳng có khó hiểu Với phương pháp việc giải bất đẳng thức kì thi quốc gia, quốc tế khơng cịn khó khăn với lượng lớn bạn học sinh Tuy nhiên, lời giải đẹp sáng cho toán điều vươn tới Chẳng thể có phương pháp mà lời giải tốn phương pháp đẹp Chính điều tạo nên quyến rũ khơng nhàm chán bất đẳng thức Là người u thích mơn học

đầy kì bí này, tơi đúc kết cho riêng phương pháp có tên GLA, tạm dịch “hình học hóa đại số” Thực chất ứng dụng phương pháp p, R, r đại số mà thơi Trong bất đẳng thức hình học, việc qui đại lượng nhưđộ dài, sin, cos tam giác vềp, R, rđã khắp nơi giới nghiên cứu từ lâu người có hiểu biết riêng chưa có sách nói thật chi tiết Có lẽ, bất đẳng thức lượng giác chưa bao giừo xuất kì thi quốc tế mà người cho với nghiên cứu vềp, R, r đủ không nghiên cứu tiếp Và bất đẳng thức lượng giác p, R, r có sức mạnh hủy diệt đủ

(86)

riêng tơi bất đẳng thức đại số ví phạm trù riêng cịn bất đẳng thức hình học ví phạm trù chung triết học: “Cái riêng toàn bộ, phong phú hơ chung, chung phận, sâu sắc riêng” Tơi mạnh dạn sâu vào tìm hiểu ứng dụng p, R, r đại số tách riêng thành phương pháp có tên GLA trước hết nhận thấy dạng tốn định cho lời giải đẹp; sau muốn góp phần cơng sức tìm lại tiếng nói cho bất đẳng thức hình học Tơi muốn chứng minh phần quan điểm nêu Có thể tơi q ngơng cuồng qua viết tơi khơng chứng tỏđược khả hạn chế

chưa thể phủđịnh quan điểm

Trong trình viết phần lý thuyết sẽđược đặt khơng tn theo qui tắc thông thường Phân đầu viết cố xây dựng kiến thức thật áp dụng để giải tập mà không cần dùng đến phần “lý thuyết tổng quan” cuối viết Tại kì thi học sinh giỏi bất đẳng thức kinh điển

được áp dụng trực tiếp cịn lại tất áp dụng phải chứng minh Do

đó để bạn hiểu lý thuyết để giải tập dùng lý thuyết cách máy móc

Những tập phần viết khơng q khó, bạn đọc muốn tìm hiểu cao xin liên hệ với qua địa chỉở cuối viết Đồng thời xin chân thành cảm ơn ý kiến đóng góp từ bạn đọc

(87)

A CỞ SỞ CỦA PHƯƠNG PHÁP

Xin nói trước tơi trình bày viết khơng giống trình bày phương pháp khác họđó xây dựng lý thuyết vào giải tập xem thử sức mạnh phương pháp Ởđây tơi chỉđi sơ lược cần thiết để giải toán đối xứng biến Sau trình bày tương

đối hồn chỉnh với biến ta bắt đầu tìm hiểu xem GLA cịn có ứng dụng mặt thật Việc trình bày theo cách khơng hồn tồn vơ lý lẽ sau giải loạt toán biến bạn nắm kiến thức sở GLA để dễ dàng tiếp thu lý thuyết cao xa Những mà tơi trình bày phần từ

A đến E với kiến thức học sinh THCS hiểu gần tồn Xóa nhịa ranh giới tuổi tác điều tơi cố gắng thực phần từ A đến E

Xét bất đẳng thức biến đối xứng với điều kiện biến không âm: a, b, c

Bằng cách đặt x= +b c y, = +c a z, = +a b x= b+c y, = c+a z, = a+b nhiều cách khác ta suy , ,x y z độ dài cạnh tam giác Như ta chuyển bất đẳng thức đại số thành hình học Trường hợp biến a, b, c có biến O tam giác suy biến thành đường thẳng Ta coi

đó tam giác có r=

(88)

B CÁC HẰNG ĐẲNG THỨC VÀ BỔ ĐỀ ÁP DỤNG TRONG BÀI VIẾT:

Qui ước: Khi nhìn thấy kí hiệu a, b, c ta hiểu độ dài cạnh tam giác Còn p, R, r nửa chu vi, bán kính đường trịn ngoại tiếp nội tiếp

∆ABC

VT kí hiệu vế trái, VP kí hiệu vế phải a) ab+bc+ca= p2 +4Rr+r2

b) ( ) 2

2 ab+bc+ca =a +b +c +16Rr+4r2

2

c) a2 +b2 +c2 =2p2 −8Rr−2r

( )( )( )

( )( )( )

2

2

1

d) 2 2

9 18

e) 3

4 32 p

Rr r b c a c a b a b c

p p

Rr r b c a c a b a b c

p

− − = − + − + − + − − − = − + − + − + −

Chng minh: Ta dễ dàng nhận thấy đẳng thức cần chứng minh tương đương với nên cần chứng minh cho đẳng thức a) đủ

Ta có: cotg A

p− =a r a=2 sinR A ⇒ sin ; tg

2a 2A r A

R p a

= =

Mặt khác áp dụng công thức:

2

2 tg sin

1 tg A A

A

= +

( )

( )

( )

2 2

2

2

2 1

r

r p a p a

a

R r p a r

p a

⋅ −

− ⇒ = =

− + +

( ) ( )

2 2 4 2 2 4 4

ap pa a ar Rr p a a pa a r p Rr Rrp

⇒ − + + = − ⇒ − + + + − =0 (1)

Xét phương trình: x3 −2px2 +(r2 + p2 +4Rr x) −4Rrp=0 (*) Từ (1) ta thấy a, b,

c nghiệm (*) Do theo định lý Viet ta có:

2 4

ab+bc+ ca= p + Rr +r

d) Hệ thức chứng minh lần nhà toán học P Nuesch vào năm 1971 tạp chí “Elementary Math”, No 26, 1971 trang 19 Đây hệ

thức phức tạp Rất tiếc chưa đọc cách chứng minh nên

đành chứng minh tam sách sau đây:

( )( )( )

2

2

2 2

9 18 p

2

Rr r b c a c a b a b c

p

(89)

( )( )( )

2

36Rrp 18pr 2p b c 2a c a 2b a b 2c

⇔ − − = − + − + − + − (1) VT(1) = 9abc 18S2 2p3 9abc 18(p a)(p b)(p c) 2p3

p

− − = − − − − −

Đặt p− =a x p, − =b y p, − =c z ⇒ + + = − + − + − =x y z p a p b p c p;

, ,

a= +y z b= +z x c= +x y ⇒ VP(1) = −(2x− −y z)(2y− −z x)(2z− −x y) VT(1) = ( )( )( ) (

9 x+y y+z z+x −18xyz−2 x+ +y z) Tức ta cần chứng minh:

( ) ( )( ) ( )3 ( ) ( ) ( )

9 x+y y+z z+x −18xyz−2 x+ +y z = − 2x− −y z 2y− −z x 2z− −x y (2)

Ta có: VT(2) ( ) ( ) ( )

( ) ( )

2 2

3 3 2

9

2 3

18

x y z y z x z x y xy xyz

x y z x y z z x y xyz

⎡ ⎤

= ⎣ + + + + + + ⎦−

⎡ ⎤

− ⎣ + + + + + + + ⎦

( ) ( ) ( ) ( )

2 2 3

3⎡x y z x y z z x yx y z 12xyz

= ⎣ + + + + + ⎦− + + − (3)

Đến việc chứng minh A = B đơn giản nhiều Nếu khơng tìm cách chứng minh hay bạn đọc chịu khó ngồi phân tích nhân tử Việc làm tốn chút công sức không cần suy nghĩ nhiều có trước kết mà khơng cần nháp, xin trình bày để bạn tham khảo:

Ta nhận thấy (3) biểu thức đối xứng dễ dàng thấy đặt (3)

( , , )

f x y z 2x= +y z nghiệm f x y z( , , ) Do tính đối xứng

( , , )

f x y z có bậc nên 2y= +z x, 2z= +x y nghiệm f x y z( , , ) có nghiệm Dấu x3,y3,z3 f x y z( , , ) dấu trừ nên viết: f x y z( , , )= −(2x− −y z)(2y− −z x)(2z− −x y)

Đây mẹo nhỏ q trình phân tích biểu thức có tính chất đối xứng Cịn việc thi có sử dụng tính chất khơng bạn tham khảo thầy giáo có uy tín nhé!

e) Hệ thức chứng minh lần nhà toán học P Nuesch vào năm 1972 tạp chí “Elementary Math”, No 27, 1972 (trang 16-17) Các bạn chứng minh tương tự cach chứng minh d)

(90)

Đẳng thức cho tương đương với:

( )( )( )

2

128Rrp−32pr −8p = 3a− −b c 3b− −c a 3c− −a b

( )( )( ) ( )3 ( )( )( )

32abc 32 p a p b p c a b c 3a b c 3b c a 3c a b

⇔ − − − − − + + = − − − − − −

Đặt ( ) ⇒

2

2 0, 0,

2

a x y z

b y z x x y y z z x

c z x y

= + + ⎧

= + + + > + > + > ⎨

= + + ⎩

( )

4

a+ + =b c x+ +y z (1) Ta dễ dàng nhận thấy với cách đặt điều kiện a, b, c độ dài cạnh

tam giác không bị vi phạm và:

, ,

3

3

3

p a y z p b z x p c x y

a b c x

b c a y

c a b z

− = + − = + − = + ⎧

− − = ⎪

− − = ⎪

− − = ⎩

(2)

Từ (1) (2) ta cần chứng minh:

( )( )( ) ( )( )( ) ( )3

32 2x+ +y z 2y+ +z x 2z+ +x y −32 x+y y+z z+x −64 x+ +y z =64xyz

⇔ (2x+ +y z)(2y+ +z x)(2z+ +x y) (− x+ y)(y+z)(z+x)−2(x+ +y z)3 =2xyz

Đến ta chứng minh (m+n n)( + p)(p+m)+mnp=(m+ +n p mn)( +np+ pm) (*) áp dụng với m= +y z n, = +z x p, = +x y ta có:

(*) ⇔ ( )( )( ) ( )( )( )

( )3 ( )( )( )

2 2

2

x y z y z x z x y x y y z z x

x y z x y y z z x xyz

+ + + + + + + + + +

⎡ ⎤

= + + + ⎣ + + + + ⎦

⇔ ( ) ( )( ) ( )( ) ( )( )

( )3 ( )( )

2

2 ( )

x y z x y y z y z z x z x x y

x y z x y z xy yz zx

⎡ ⎤

+ + ⎣ + + + + + + + + ⎦ = + + + + + + +

(91)

2 Các định lý:

Định lý 1: Cho tam giác ABC, D điểm thuộc BC Khi đó:

( )

2 2

nc +mb = d +mn a AD = d, BD = m, DC = n Chứng minh:

Ta có 2 2 cosn

m +dc = md ADB (1), n2 +d2 −b2 =2ndcosnADC (2) Nhân vế (1) với n vế (2) với m ta được:

( 2 2) 2 cosn ( )3 , ( 2 2) 2 cosn ( )4

n m +dc = mnd ADB m n +db = mnd ADC

Cộng vế theo vế (3) với (4) ta được:

( ) ( ) 2 2 (cosn cosn) ( 2)

mn m+n + m+n dncmb = mnd ADB+ ADCmn+d a=nc +mb2

Định lý 3: p2 ≤2R2 +10Rrr2 +2(R−2r) R R( −2r)

Cách 1: Giả sửa, b, c thỏa mãn a > bc≥ nghiệm phương trình:

( ) 2 ( 4 2) 4

M X =XpX + p + Rr+r XpRr=0

>

2

Điều kiện đểa, b, c độ dài cạnh tam giác là:

0

0

b c a p a

p a b c

c c

+ > > ⎧ ⎧

⎪ ⇔⎪ ⇔ > ≥ ≥ ⎨ ⎨

> > ⎪ ⎪

⎩ ⎩ (1)

⇔ Phương trình M(X) = có nghiệm thỏa mãn (1) Ta có: M′( )X =3X2 −4pX + p2 +4Rr+r

( )2 3( 4 2) 12 3

p p Rr r p Rr r

∆ = − + + = − − ; M(X) có nghiệm ⇒∆’ ≥

Hai nghiệm M’(X) = là: 1 ; 2

3

p p

(92)

⇒ (1) ⇔

( ) ( ) ( ) ( )

1

0 0

0 M

M X M X M p

⎧ < ⎪

⎪ ≥ ⎪

≤ ⎪

> ⎪⎩

Ta nhận thấy M(0) < M(p) >

Còn ( )

( )

( )

( )

2

1

2

2

0 18

0 18 9

M X p p Rr r

M X p p Rr r

⎧ ≥ ∆ ∆ ≥′ ′ − + ⎪ ⇔⎪

⎨ ⎨

≤ ′ ′

⎪ ⎪∆ ∆ ≥ − − +

⎩ ⎩

( 18 9 2)

p p Rr r

′ ′

⇔ ∆ ∆ ≥ − +

( )3 2( 18 9 2)2 2 2(2 10 2) (4 )3 0

p p Rr r p p R Rr r r R r

⇔ ∆ ≥ − + ⇔ − + − + + ≤ (2)

( 2 2)2 ( )3 ( )3

1′ 2R 10Rr r r 4R r 4R R 2r ∆ = + − − + = − ≥0

⇒ (2) ⇔ ( ) ( )

( ) ( )

2

2 2

2 10 2

2 10 2

R Rr r R r R R r

p R Rr r R r R R

(93)

Cách 2: Cách chưa có tài liệu mang đậm sắc hình học

Ta có: p2 ≤2R2 +10Rrr2 +2(R−2r) R R( −2r)

( ) ( ) ( ) ( )

2 16 5 2 4 4 2 2 2 2

p Rr r R Rr r R r R R r R R r

⇔ − + ≤ − + + − − + − ( ) ( )

2

9.IGR 2r R R 2r ⎤ 3.IG R 2r OI

⇔ ≤⎣ − + − ⎦ ⇔ ≤ − +

Trong O, I, G tâm đường tròn ngoại tiếp, nội tiếp trọng tâm

∆ABC Trên đường thẳng IG ta lấy điểm H cho Ta dùng định lý để tính đoạn OH

3 IK = IG

JJG JJG

Theo định lý 1:

( )

2. 2. .

OI HG+OH IG= OG +IG GK IK

⇔ 2 ( 2 ) 3 2 2 2 16

9

p Rr r

a b c

R Rr +OK = ⎛⎜R − + + + ⋅ − + ⎞⎟

⎝ ⎠ (do

) , GK= IG IK= IG

⇔ 6R R( −2r)+3.OK2 =9R2 −(2p2 −8Rr−2r2)+2p2 −32Rr+10r2 ⇔ 3. 3( 4 4 ) ⇔

OK = RRr+ r2 OK2 =(R−2r)2 ⇔OK= −R 2r

Trong tam giác OIK ta ln có: OI + OK ≥ IK hay (R− 2r) + OI ≥ 3.IG Tức là: p2 ≤2R2 +10Rrr2 +2(R−2r) R R( −2r)

Đẳng thức xảy ⇔ O nằm I K

(94)

Hoàn toàn tương tự ta chứng minh được:

( ) ( )

2 2 10 2 2 2

pR + RrrRr R Rr

Điều tương đương với IK + OK ≥ OI Đẳng thức xảy K nằm O I

Bây ta sẽđi tìm điều kiện cần để:

+ O nằm I K

+ K nằm O I

* O nằm I K khi: 16 5 2 ( 2 )

pRr+ r = −R r+ R Rr

( ) ( ) ( ) ( ) ( ) ( )( ) ( )

2

2 2

16 2 2

2 2 3

p Rr r R R r R r R R r

R R r R r R r R r p R r p R r

⇒ − + ≥ − + − −

≥ − + − = − − ⇒ ≥ + ⇒ ≥ +

Theo định lý trình bày điều xảy tam giác có góc ≥ 60°, tức tam giác cân có cạnh bên lớn cạnh đáy

* K nằm O I khi: 16 5 ( 2 ) ( 2 )

pRr+ r = R RrRr

( ) ( )2 ( )( )

2 16 5 2 2 2 3 4

p Rr r R R r R r R r R r

⇒ − + ≤ − + − ≤ − −

( )2

2 3 3

p R r p R r

⇒ ≤ + ⇒ ≤ +

Theo định lý trình bày điều xảy tam giác có góc ≤ 60°, tức tam giác cân có cạnh bên nhỏ cạnh đáy

Định lý 5: a2 +b2 +c2 ≤8R2 +4r2

)

Chứng minh: Ta có nhiều cách để chứng minh định lý viết sử dụng định lý làm bổđề bổđề mạnh tính ứng dụng cao

Nhận thấy ( 2 ) ( 2 ) (

R RrR Rr +r = Rr Do đó:

( )( ) ( )

2 2 10 2 2 2 10 2 3 2

pR + Rrr + Rr RrpR + Rrr + RRr+ r ⇔ 4 4 3 2 8 8 6

pR + Rr+ rpR + Rr+ r

⇔ 2 8 2 8 8 6 2 2 8 4

(95)

Lại có 2 16 4 2( )

a +b +c + Rr+ r = ab+bc+bc

( ) ( )2

2

8R 16Rr 8r ab bc bc R r ab bc ca

⇒ + + ≥ + + ⇒ + ≥ + +

Định lý 4: p2 ≥2R2 +8Rr+3r2 mọi tam giác nhọn

Các bất đẳng thức tương đương: 2 4( )2

a +b +cR+r

2

2 12 ab+bc+caR + Rr+ r

Những bất đẳng thức gặp nhiều sách nên xin không đưa chứng minh

Định lý 2: Nếu tam giác ABC có: Hai góc ≥ 60° p≥ 3(R+r) Hai góc ≤ 60° p≤ 3(R+r)

Một góc 60° p= 3(R+r)

Chứng minh: Ta có: 3( ) 1( )

2

p R r a b c r

R R R

− + + +

= − +

( )

3

sin sin sin cos cos cos

2

A+ B+ C A B C

= − + +

( ) ( ) ( )

sin sin sin

3

A π B π C

= − + − + −

3

π (1)

Đặt , ,

3

x= −A π y= −B π z= =C

π ta có x+ + =y z 0

Khơng tính tổng quát ta giả sử: x≥ ≥y z

( )1 sin sin sin sin sin sin( ) 2sin cos 2sin cos

2 2

x y x y x y x y

x y z x y x y + − +

= + + = + − + = − +

2sin cos cos 4sin sin sin

2 2 2

x+ yxy x+ yx+y x

= ⎜ − ⎟=

⎝ ⎠

y

Do x+ + =y z x≥ ≥y z nên x+ ≥y x≥ 0, x < π, x+y < π suy sin sin

2 x+ y x

− Nếu y≥ ⇔

B≥π sin

y ≥ đó 3( ) 4sin sin sin 0

2 2

p R r x y x y

R

− + +

2

(96)

Tức p≥ 3(R+r) ∆ABC có góc ≥

π

− Nếu y≤ sin y

≤ , đó: 3( ) 4sin sin sin

2 2

p R r x y x y

R

− + +

2

= ≤

Tức p≤ 3(R+r) ∆ABC có góc ≤

π − Nếu y= p= 3(R+r) sin

2 y =

Định lý 6: 16 5 r2(R 2r)

p Rr r

R

≥ − + (*)

CM: Ta ln có: ( 2 ) 9 ( 2 2)

3

IGIOOGIGR RrRa +b +c

( ) ( )

( ) ( )

2 2 2 2

2 2

18

3

3

a b c Rr

IG R R r R a b c IG

R R r R a b c

+ + − ⇔ ≥ − − − + + ⇔ ≥

− + − + + (1)

Do 9.IG2 =p2 −16Rr+5r2 nên p2 ≥16Rr−5r2 ⇒ a2 +b2 +c2 =2p2 −8Rr−2r2 ≥24Rr−12r2 (2) Từ (1), (2) ⇒

( ) ( )

2

2

6 12 12

3

6

3 24 12

Rr r Rr r R r

IG r

R

R R r

R R r R R r

− − −

≥ =

− + − + =

( )

2

2

2 9.IG r R r

R

⇒ ≥ Vậy (*) chứng minh Đẳng thức xảy ⇔∆ABC

Comment: Định lý chặt BĐT quen thuộc p2 ≥16Rr−5r2 chút xíu đặc biệt quan trọng “đương đầu” với BĐT chặt Như bạn

đã biết BĐT p2 ≤4R2 +4Rr+3r2 một BĐT tương đối chặt những vẫn chưa đủ

độ mạnh để khuất phục “cứng đầu” Tuy nhiên cần làm chặt chút xíu:

(**) 2 10 2( 2 ) ( 2 )

pR + Rrr + Rr R Rr lại giải tốn

đó cách “ngon lành” Định lý có tầm quan trọng không so với (**) Thực làm chặt BĐT (*) thành:

( )

2 2 10 2 2 2

(97)

Tóm lại ta có BĐT kẹp:

( ) ( ) ( )

2

2 2 2

16Rr 5r r R r p 2R 10Rr r R 2r R R 2r R

− + ≤ ≤ + − + − −

Định lý 7: Cho tam giác ABC thỏa mãn a≥ ≥b c a+ ≥b 3c CMR: r R ≤ Chứng minh: Ta có: ( )( )( )

2

a b c b c a c a b

r

R abc

+ − + − + − =

Đặt ( ) ( )( )( )

a b c b c a c a b

f c

abc

+ − + − + − =

⇒ ( ) ( ) ( )( )2 ( )

2

2 0

2

a b c c a b a b a b c c

f c

abc abc

+ − + + − + −

′ = ≥ ≥

Do ƒ(c) đồng biến theo c Thay a b

c= + vào ƒ(c) ta được:

( ) ( ) 2( )(2 ) 2( )2

3 9

b a a b a b

a b

f c f

ab ab

− − −

+

≤ = = −

9

Vậy r

R≤ Đẳng thức xảy ⇔

(98)

C Xây dựng đẳng thức

Đây phần “xương sống” phương pháp Chỉ cần nắm vững

đẳng thức phần nhiều tập khó phần sau trở nên đơn giản

y Xét a, b, c >

Nhưđã nói phần A, sau đặt:

, ,

x= +b c y= +c a z= +a b x, y, z trả thành độ dài cạnh tam giác Ta chuyển sốđại lượng đại số hình học thơng qua p, R, r nửa chu vi, bán kính đường tròn ngoại tiếp nội tiếp tam giác XYZ

1 Tính 2

a +b +c2 ab+bc+ca

Ta có: 2 ( )2 ( )2 ( )2 2( 2 ) x +y +z = a+b + b+c + c+a = a +b +c +ab+bc+ca (1)

( )( ) ( )( ) ( )( )

( ) ( )

2 2 3 2

xy yz zx a b b c b c c a c a a b

a b c ab bc ca

+ + = + + + + + + + + = + + + + +

Từ (1) (2) suy ra:

y 4(a2 +b2 +c2)=3(x2 +y2 +z2)−2(xy+yz+zx)

⇔ 4( 2 2) ( )2 2 2( ) ( 2 2)

a +b +c = x+ +y z − ⎡⎣ xy+ yz+zxx + y +z ⎤⎦

⇔ 4( 2 2) 4 2 16( 4 2) 2 2 8

a +b +c = pRr+ ra +b +c = pRr−2 r2

y 4( ) 2( ) ( 2 2)

ab+bc+ca = xy+ yz+zxx +y +z

( ) 2

4 ab+bc+ca =16Rr+4rab+bc+ca=4Rr+r

y

2 2

2 2

2

8

2

4

p Rr r p

a b c

ab bc ca Rr r Rr r

− − + + = =

+ + + + −

)

2 Tính ( )(abc)( a+b b+c c+a

Ta có: ( )( )( ) ( )( )( )

x y z y z x z x y

abc

xyz

a b b c c a

+ − + − + − =

+ + +

( )( )( )

4

p x p y p z pr r

xyz Rrp R

− − −

(99)

3 Tính a b b c c a

c a b

+ + + + +

( )( )

3 a b c ab bc ca

a b c a b c a b c

c a b abc

+ + + + + + + + + +

= + + − = −

( )

( )( )( )

( ) ( )

2

2

4 3 3 3 2

p Rr r p Rr r R r R r

r r

p x p y p z pr

+ + + −

= − = − = − =

− − −

4 ( ) ( ) ( )

( ) (

3 3 2

2 2 2

3

8 12

a b c a b c a b c ab bc ca abc

p p Rr r Rr r pr p p Rr

⎡ ⎤

+ + = + + ⎣ + + − + + ⎦+ = − − − − + = − )

)

5 4 ( 2 2)2 2( )2 4 (

a +b +c = a +b +cab+bc+ca + abc a+ +b c

( ) ( )

( ) ( ) ( ) ( )

2

2 2 2

2

4 2 2 2 2

8 2 4

4 4 4 16

p Rr r Rr r p r

p Rr r p Rr r Rr r p r p Rrp Rr r

= − − − + +

= − + + + − + + = − + +

6

2

2

3

p p

a b c

r

abc pr

+ + = =

7 Gọi S diện tích tam giác XYZ có độ dài cạnh , ,x y z Ta có: 16S2 =(x+ +y z)(x+ −y z)(y+ −z x)(z+ −x y)

( ) ( ) ( ) ( ) ( )

( )

( ) ( )

2

2 2 2 2 2 4 2 2 2

2 2 4 2 2 2

2 2 2 4

2 2

2

2

x y z z x y x y z z x y z x y

x y xy z z x y x y z x xyz z y

x y y z z x x y z

⎡ ⎤ ⎡ ⎤

=⎣ + − ⎦ ⎣ − − ⎦= + − − − + − = + + − − − + + − +

= + + − + +

Trong số toán trường hợp x2 ≥y2 +z2 (

x max {x y z, , }) ta nhận thấy tốn Do cần xét thêm trường hợp x2 <y2 +z2

toán giải hồn tồn

Đặt ta lại có độ dài cạnh tam giác Nếu gọi

2, 2,

m=x n=y p=z2 m n p, ,

1,

R r bán kính đường trịn ngoại tiếp nội tiếp tam giác MNP có

đọ dài cạnh m n p, , thì:

( ) ( )

2 2 2

1 1

(100)

Vậy nhiều toán biến qua lần đặt ẩn ta qui toán biến Do toán ta nghien cứu bất đẳng thức đối xứng nên sau chuẩn hóa tốn cịn biến Mà toán biến thương giải cách dễ

dàng

8 a b c (a b c)( 1 ) b+c+ c+a+ a+b= + + a+b+b+c +c+a

1 1 3 p

x y z

⎛ ⎞ = ⎜ + + ⎟−

⎝ ⎠

( ) ( 4 2) 4 2 8

3 3

4

p xy yz zx p p Rr r

4

p Rr r p Rr r

xyz Rrp Rr Rr

+ + + + + + − +

= − = − = − =

9 1 ab bc ca 4Rr 2r2 4R r

a b c abc pr pr

+ + + + + + = = =

10 1 a b c p2 12

ab bc ca abc pr r

+ +

+ + = = =

11 ( ) ( )

2 2 2 2

2 2 2 2 2

2

1 1 a b b c c a ab bc ca abc a b c

a b c a b c a b c

+ + − + + + +

+ + = =

( )2 ( )2

2 2

2 2

4

r R r p r R r p

p r p r

+ − + −

= =

12 1 1 1 a+b +b+c +c+a= +x y + z

2 4

4

xy yz zx p Rr r

xyz Rrp

+ + + +

= =

13 ( ) ( )

2 2 2 2

2 2 2 2 2

2

1 1 x y y z z x xy yz zx xyz x y z

x y z x y z x y z

+ + − + + + +

+ + = =

( 2 2)2 2 ( 2 2)2

2 2 2

4 16 1

16 16

p Rr r Rrp p Rr r

Rr

R r p R r p

+ + − + +

= = −

14 (a+b)(b+c)(c+a)=xyz=4Rrp

( )( )( ) S2

abc p x p y p z pr

p

= − − − = =

15 ab ac bc a b2 b c2 c a2

c b a abc

+ + + + =

( )2 ( ) 2( )2 2 ( )2

4

2 r R r p r R r p

ab bc ca abc a b c

abc pr p

+ − + − + + − + +

(101)

16 a b3 3+b c3 3+c a3 3=(ab+bc+ca)3 −3abc a( +b b)( +c c)( +a)

3

( )3

3 4 12

r R r p r R

= + −

17 2 2 2 2 2 2 a +b +b +c +c +a

( )( ) ( )( ) ( )( )

( )( )( )

2 2 2 2 2 2

2 2 2

a b a c b c b a c a c b

a b b c c a

+ + + + + + + + =

+ + +

( )

( )( )

2

2 2 2 2 2 2 2 2 2 2 2

a b c a b b c c a

a b c a b b c c a a b c

+ + + + + =

+ + + + −

( ) ( )

( ) ( )

( ) ( )

( ) ( )

2

2 2

2 2 2 2

3

2 2 2

2 2 2

8

4 2

8

p Rr r Rr r p r p R r rp r R r

r R Rr r p p r r R r

p Rr r Rr r p r p r

− − + + − − + + +

= =

⎡ ⎤ + + − − +

− − ⎣ + − ⎦−

18 (a b b c c a a c b a c b2 + + )( + + )=a b3 3+b c3 3+c a3 3+3a b c2 2+abc a( 3+b3+c3)

( ) ( )

( )

3

3 2

3

2 2

4 12 12

4 24

r R r R r p p r p r p Rr

r r R r p r p Rrp

= + − + + −

⎡ ⎤

= ⎣ + + + − ⎦

Phần xây dựng công thức xin dừng lại ởđây Trong trình làm tập cần cơng thức khác bạn đọc dễ dàng tự xây dựng Do đa thức đối xứng qui tổng tích đại lượng

nên qui

,

(102)

D MỘT SỐ BÀI TOÁN SƯU TẦM

Phần gồm tốn tiếng có nhiều cách giải Tuy nhiên viết ta dùng phương pháp GLA để giải

Bài (Iran 1996)

Cho a, b, c > Chứng minh rằng:

( )2 ( )2 ( )2 ( )

9

1 1

4 ab bc ca

a b b c c a

+ + ≥

+ + + + +

Gii

Áp dụng công thức 13 phần C ta cần phải chứng minh:

( )

( ) ( ) ( )

2

2 2

2 2 2

4 1 9 1 9

4

16 4 16

p Rr r p Rr r

Rr R R r

R r p Rr r R rp

+ + + +

− ≥ ⇔ − ≥

+ +

Xét ( )

2

2

2

4 16

p Rr r

A

R rp

+ +

= Ta chứng minh A đồng biến theo p

C1: Tính đạo hàm

C2:

( ) ( ) ( )

( )

2

2 2

2 2 2

2

2

4 2 4 8 2 4

2

9

16 16

Rr r Rr r

p Rr r p Rr r

p A

R r R r

+ +

+ + + + +

= ≥ +

2

Đến ta nhận thấy A đồng biến theo p

p2 −16Rr+5r2 =9.IG2 ≥ ⇒0 p2 ≥16Rr−5r Do đó

( )

( ) ( ( ) )

( )

( )

2 2

2 2 2

2 2

2

16 20 25 10

16 16 16 16

16 16

Rr r Rr r Rr r R r R Rr r

A

R r R r R R r R R r

R r Rr r

− + + − − − +

≥ = = =

− − −

Cơng việc cịn lại ta chứng minh:

( ) ( )

( )( ) ( )

( )

( ) ( ) ( )

2 2

3

2

3 2 2 3

2

3 2 3

25 10 9

4 4

16 16

4 9 16

4 36 20 16

4 36 11 16

R Rr r R Rr r

R R r R r

R R r R R r

R r R Rr r R R r

R R r R r Rr Rr r R R r

R R r Rr r R R r r R r

(103)

Đẳng thức xảy ⇔ , 0( )

2

r a b c

R r a b c

= = = ⎡

⎢ ⇔ ⎢

⎢ ⎢ = = =

và hoán vị

Comment: Lời giải toán lời giải tốn sau

đây sẽđược trình bày cách tỉ mỉđể bạn tiện theo dõi Tuy nhiên, thấy, lời giải sáng sủa gọn gàng Bạn yêu thích bất đẳng thức hình cảm nhận vẻđẹp lời giải bạn chưa quan tâm thực đến bất đẳng thức hình cho khơng cịn ởđằng sau sau đọc viết bạn thay đổi nhìn bất đẳng thức hình

Bài Cho a, b, cab+bc+ca=1 CMR: 1 a+b+b+c+ c+a

Gii

Áp dụng đẳng thức 12 phần C ta có tốn sau:

Cho x y z, , >0 4Rr+r2 =1 CMR:

4

p Rr r

Rrp

+ +

≥ ⇔ p2 −10Rrp+ ≥1 0 (1)

Xét phương trình: f x( )=x2 −10Rrx+ =1 0; ∆ =′ 25R r2 −1

⇒ 2 2

1 25 ; 25

x = RrR rx = Rr+ R r −1

Để chứng minh (1) ta phải chứng minh 2 25

px = Rr+ R r −1 Ta có:

( )2 ( )2 ( )

2 2 2 2

25R r − =1 25R r − 4Rr+r =r 9R −8Rrrr 3Rr =r 3Rr

⇒ 5Rr+ 25R r2 − ≤1 5Rr+r(3Rr)= −2 3r2

Lại có p2 −16Rr+5r2 =9.IG2 ≥ ⇒ ≥0 p 16Rr−5r2 = 4 9− r2

Mà 4 9− r2 ≥ −2 3r2 ⇔ −4 9r2 ≥(2 3− r2)2 ⇔ −4 9r2 ≥ −4 12r2 +9r4

( )

2 1 3 0

r r

⇔ − ≥ Đẳng thức xảy ⇔

2

0

2, 16

r

x y z

p Rr r

⎧ =

⎪ ⇔ = = = ⎨

⎪ = − ⎩

Vậy 1

(104)

Mở rộng: Cho a, b, c≥ thỏa mãn ab+bc+ca=1 b+ ≥ ≥ ≥c a b c

CMR: 1

a+b+b+c+ c+aa+ + +b c abc

Giải: Trước tiên ta nhận thấy toán chặt toán ban đầu lẽ:

( ) ( )( )( )

1=ab+bc+caa b+ca ⇒ ≤ ⇒ −a 1 a 1−b 1−c ≥0 a b c ab bc ca abc a b c abc

⇔ − − − + + + − ≥ ⇔ ≥ + + +

Với toán ban đầu ta chứng minh nhiều cách đại số mà lời giải gọn gàng với toán mở rộng lời giải đại số phức tạp Ta dùng G.L.A để chứng minh toán này.Ta nhận thấy với điều kiện toán a, b, c độ dài cạnh tam giác (Trường hợp b+c= a tam giác suy biến thành đường thẳng) Áp dụng công thức phần C ta cần phải CM:

( )( ) ( )( )( )

( )

( )( )

2 2

5

3

a c b c

a b c abc

a b b c c a

a b c ab bc ca

a b c abc

a b c ab bc ca abc

+ + ≥

+ + + + + +

+ + + + +

⇔ ≥

+ + + + + + + −

( )( ) ( )( ) ( ) ( ) (

2

2

5 16 5 16 4 1 2 5 2

1 2

Rr r Rr r Rr Rr)

a b c Rr a b c Rr

− − ⎡ ⎤

⇔ ≥ ⇔⎣ − + ⎦ + ≥

+ + − + + + −

( )( 2) ( )( 2)

5 10Rr 2Rr 16Rr 4r 10Rr 20Rr 2Rr 16Rr 4r

⇔ + − + + ≥ − ⇔ ≥ + + (1)

Ta nhận thấy: (1 2+ Rr)(16Rr+4r2)≤18 2( + Rr Rr) =18Rr+2 18Rr Rr

( )

18Rr 2Rr ab bc ca 20Rr

≤ + + + = Do (1) chứng minh

Vậy 1

a+b +b+c+c+aa+ + +b c abc Đẳng thức xảy ⇔a=b= 1, c=

Bài 3. Cho x y z, , >0 Tìm điều kiện để bất đẳng thức sau ln

(x y)x(x z) 3(x 4y z) (x y)(yxyzz)(z x)

⎛ ⎞

≥ ⎜ + ⎟

+ + + + ⎝ + + + ⎠ ∑

Gii

Đứng trước toán này, phương pháp đại số nên bắt đầu từđâu đề xuất thức khó hịu điều kiện để tốn theo dựđốn khơng đơn giản Nhưng dạng chắn dùng G.L.A

(105)

Đặt y+ =z a z, + =x b x, + =y c Bài toán trở thành:

( )( )( )

4 1

p a p b p c

p a

bc p abc

⎛ − − − ⎞ − ≥ +

⎜ ⎟

⎝ ⎠

( ) 4 ( )( )( ) 4 ( )

1 cos

2

3

p p a p a p b p c A A

bc abc

⎛ ⎞

− − − −

⇔∑ ≥ ⎜⎝ + ⎟⎠⇔ ≥ 1+∏sin

2

(1)

(Trong p nửa chu vi ∆ABC có độ dài cạnh a b c, , )

Bạn đọc nhận (1) bất đẳng thức Jack Garfulkel Điều kiện để toán tam giác ABC nhọn Tức

(giả sửa= max{a, b, c}) ⇔

2

b +c >a (x+ y)2 +(x+z)2 >(y+z)2

( ) yz

x x y z yz x y z

x

⇔ + + > ⇔ + + > (trong x=min , ,{x y z})

Việc ta chứng minh (1) với điều kiện ∆ABC nhọn Đây chặt nên chứng minh G.L.A không hềđơn giản

Gii

Ta có: cos sin( ), cos sin( ), cos sin( )

2 2 2 C2 C

A= π− A B= π−B = −

2

π

Đặt , ,

2 2A B2 C2 A′= −π B′= −π C′= −π

(1) ⇔ sin (1 cos )

A′≥ + A

∑ ∏ (2)

trong A B C′ ′ ′, , sốđo góc tam giác nhọn có min{ , , } A B C′ ′ ′ ≥π

(2) ⇔ ( )

2

2

2

2

4

3 4

p p R r

p Rp Rr r

R R R

⎛ − + ⎞

≥ ⎜ + ⎟⇔ − − − ≤

⎝ ⎠

2 2 2

2

3 16 48 12

2

2

R R Rr r R Rr r R R Rr r

p + + + p + + + + +

⇔ ≤ ⇔ ≤ (3)

Khơng tính tổng quát ta giả sử

4 A B C

(106)

TH1:

B′ ≤π Áp dụng định lý ta có p2 ≤3(R+r)2 Do ta cần chứng minh: 6( )2 3 8 2 9 48 12

R+rR + Rr+ r +R R + Rr+ r

2 2 2

2 2

6 12 48 12

3 4 48 12

2

R Rr r R Rr r R R Rr r

R Rr r R R Rr r

⇔ + + ≤ + + + + + ⇔ + + ≤ + +

Mà 48 12 36 36 ( ) 4

R R + Rr+ rR R + Rr+ r = R R+ rR + Rr+ r

Suy (3) chứng minh

TH2:

B′ ≥π ⇒ sin sin sin sin sin sin5 0,116 2 C2 24

AB′ ′≥ π π π>

⇒ 2,16 sin sin sin

2 2 R

r = ABC′ < Áp dụng định lý ta cần chứng minh:

( ) ( ) ( )

( ) ( )

2 2 2

2 2

2 10 2 48 12

12 4 2 48 12

2

R Rr r R r R R r R Rr r R R Rr r

R Rr r R r R R r R R Rr r

+ − + − − ≤ + + + + + ⇔ + − + − − ≤ + +

( ) ( )

2 12 4 4 2 2 9 48 12

t t t t t t t t

⇔ + − + − − ≤ + + t R,t [2; 2,16) r

= ∈

( ) ( ) ( )

2 12 4 4 2 2,16 2,16 2 12 4 2, 4 2 14, 4 8,8

VTt + t− + t− − ≤t + t− + t− =t2 + t

( )2 ( )( ) ( )

3, 5, 9, 28 1, 24 3, 5,

VPt t+ + −ttt t+

Mà (3, 2 5, 6) ( 14, 4 8,8) 2, 2( 2)2 0

t t+ − t + t− = t− ≥ , VP ≥ VT

Đẳng thức xảy ⇔t=

Vậy cosA (1 sin ) ≥ 3 + 2A

∑ ∏ Đẳng thức xảy ⇔∆ABC

Bài 4. Cho x y z, , >0 CMR:

( )3 ( ) (2 )2( )2 ( )2 ( )( )

8 x+ +y z xyz x+y y+z z+x ≥⎣⎡∑ x+y ⎦⎤ ∏⎣⎡ x+y x+z −2yz⎤⎦ (1)

(107)

Đặt , ,

2

b c a c a b a b c

x= + − y= + − z= + −

2 ta có a, b, c độ dài cạnh tam giác và: a= +y z b, = +z x c, = +x y

( )( )( )

,

x+ + =y z p xyz= pa pb pc

(p nửa chu vi tam giác ABC có độ dài cạnh a, b, c)

( )( ) ( )2 ( ) (2 )2 2

2⎡ x+y x+z −2yz⎤= x+z + x+ yy+z =b +ca2

⎣ ⎦

Tương tự 2⎡(y+z)(y+x)−2zx⎤=c2+a2 −b2 , 2⎡(z+x z)( +y)−2xy⎤=a2+b2−c

⎣ ⎦ ⎣ ⎦

Vậy (1) ⇔ 8 3( )( )( ) 2 ( 2 2)3 2

2

a b c

p pa pb pc a b ca +b +c ∏ + −

⇔ 8( )3( )( )( ) 2 ( 2 2)3 ( 2

a+ +b c pa pb pc a b ca +b +ca +bc ) Khơng tính tổng qt ta giả sử a≥ ≥b c

Nếu 2 VT ≥ ≥ VP ⇒ (đpcm) b +ca2

2 p

Nếy 2 đặt

b +c >a a2 =m b, =n c, = Ta có: m n p, , độ dài cạnh tam giác Mặt khác theo bất đẳng thức Holder ta có:

(a2 +b2 +c2)3=(∑3a 3a 3a4)3 ≤(a+ +b c)2(a4 +b4 +c4) (3)

Từ (2) 93) suy để chứng minh toán ta cần chứng minh:

( )( )( )( ) ( ) ( )

( ) ( )

2 2 4 2

2 2 2 4 2 4 2

2 2

a b c a b c b c a c a b a b c a b c a b c

a b b c c a a b c a b c a b c a b c

+ + + − + − + − ≥ + + + − ⇔ + + − − − ≥ + + ( + − )

∏ ∏ (2 2 2 2 2) ( 2 2) ( )

mn np pm m n p mnp m n p m n p

⇔ + + − − − ≥ + + ∏ + −

( 2) ( 2 2) 2

1 1 1 1 1 1

16R r 4r 4R r p m n p 8p r 8R 2R r m n2 p2

⇔ + ≥ + + ⇔ + ≥ + + (4)

(4) theo định lý

Tức ta chứng minh xong (1) Đẳng thức xảy ⇔a= b= c

(108)

Comment: Tạm thời bỏ qua độ cồng kềnh tốn độ chặt đủ

làm điêu đứng phương pháp đại số Tôi thửđi tìm lời giải đại số kết sau nhiều cố gắng có lời giải dài gấp lần lời giải Có lẽ

bài sinh để dành riêng cho G.L.A

Bài 5. Cho a, b, c > CMR: a b b c c a 4( a b c )

c a b b c c a a

+ + + + + ≥ + +

b

+ + + Gii

Áp dụng công thức ta cần chứng minh: ( )

2 8

2 4

4

p Rr r

R r

r R

− + − ≥ ⋅

r

2 ⇔ 2 (2 ) 8 4 6

R R− ≥r pRr+rR + Rrrp (đúng theo định lý 3)

Đẳng thức xảy ⇔a= b= c

Bài 6. Chứng minh ∀a, b, c không âm ta có BĐT:

( )

2 2 2 1 2

a +b +c + abc+ ≥ ab+bc+ca

Gii

Nếu sốa, b, c có số ta có đpcm

Nếu số có số≠ áp dụng công thức 14 ta cần chứng minh:

( )

2 8 2 2 1 4 2 2 1 16 4

pRrr + pr + ≥ Rr+rp + pr + ≥ Rr+ r

Ta có: 2pr2 + =1 pr2 +pr+ ≥ ⋅1 3 p r2 ≥ ⋅3 327 r r2 =9r2 (1), 16 5

pRrr (2) Từ (1) (2) ta có đpcm Đẳng thức xảy ⇔a=b=c=

Xin giới thiệu bạn đọc cách chứng minh khác trình bày “Sáng tạo bất đẳng thức” Phạm Kim Hùng

Cách 1: Sử dụng tam thức bậc

Chuyển tam thức bậc a là: f a( )=a2 +2(bc− −b c a) +(bc)2 +1

( )2 ( )2 ( )(

1 2

bc b c b c bc b c

∆ = − − − − − = − − )−1 Nếu bc− − ≥b c ta có đpcm

Nếu bc− − ≤b c hay (b−1)(c− ≤1) Ta chia làm trường hợp

(109)

+ Cả hai số b c, nhỏ Theo bất đẳng thức AM − GM ta có:

(2 ) 1, (2 )

bbcc ≤1 ⇒ ∆ ≤′ Vậy bất đẳng thức chứng minh Cách 2: Đặt k= + +a b c Sử dụng bất đẳng thức quen thuộc

( )( )( ) ( )( )( )

abca+ −b c b+ −c a c+ −a babcka kb kc Rút gọn lại ta được: 4(ab bc ca) k2 9abc

k

+ + − ≤ (*)

Bất đẳng thức toán tương đương với:

( )2 (

2

a+ +b c + abc+ ≥ ab+bc+ca) ⇔ 4(ab+bc+ca)−k2 ≤ +1 2abc Sử dụng (*) ta cần chứng minh: ( )9 abc

k − ≤

Theo bất đẳng thức AM − GM ta có: ( ) ( )9 (9 ) 27 27

k k k

abc

k k

− 1 − ≤ − = ≤

Vậy ta có đpcm Đẳng thức xảy ⇔a=b=c=

Bài 7. (Phạm Kim Hùng) Giả sử a, b, c số thực không âm Chứng minh bất

đẳng thức:

( )

2 2 2 2

10

1 1

a b b c c a a b c

+ + ≥

+ + + + +

Cách 1: (Của PKH)

Khai triển hai vế cách gui đồng mẫu số:

( )

4 3 2 2 10 2 2 2

sym sym sym sym

a a b a aba b c a b c

⎛ + ⎞⎛ + ⎞≥ + +

⎜ ⎟⎜ ⎟ ⎜

⎝∑ ∑ ⎠⎝∑ ∑ ⎠ ⎝∑

2 ⎟ ⎠

⇔ 3 2( ) 4( 2) , ,

2 6a b c 11

sym sym

a + a ab+ a b + abc c a b+ ≥ a b +c + a b c

∑ ∑ ∑ ∑ ∑ ∑ 2

Khơng tính tổng quát giả sử Để chứng minh bất đẳng thức ta tìm cách loại bỏ biểu thức chứa c Ta có:

a≥ ≥b c

( ) ( )

3 3 2

4c a +b ≥4c a +b

( )( ) ( ) ( ) ( ) ( )

( )( ) ( )( )

4 3 2 4

4 3 4 3

2

0

c a b a b c a b a b c c a b c a b

a b ca c a a c a b cb c b b c

+ + + + + + − + − + = + − − − + + − − − ≥

( ) ( )( ) ( ) ( ) ( )

3 3 4 2 4

(110)

Ngoài dễ thấy: 2( ) 12 2

sym

abcc a+ba b c ≥ 1a b c2 2

Như với phần lại ta cần chứng minh bất đẳng thức biến (thực chất chứng minh cho trường hợp c= 0) sau:

( ) ( ) ( )

6 2 4 6 3 2 6 2 2

a +b + ab a +b + a b + c aba b a+b +c a +b Bất đẳng thức tương đương với:

( )( ) ( )( ) ( ) ( )

( ) ( )( ) ( )

2

2 4 3 2

2 3 2 2 2

2

2

a b a b ab a b a b c a b a b a b

a b a b a b ab a b ab a b ab c a b

− − + − − ≥ − + −

⎡ ⎤

⇔ − ⎣ + + + + + + + − − ⎦≥0 Hiển nhiên Đẳng thức xảy a= b, c= hốn vị Cách 2: Áp dụng cơng thức 17 ta cần CM:

( ) ( )

( ) ( )

2

2 2

3

2 2

8 10

4 2

p R r rp r R r

2

p

r R Rr r p p r r R r

− + + +

+ + − − + (1)

Đặt ƒ(p) = VT − VP Tính ƒ’(p) với ý p2 ≥16Rr−5r2 ta ƒ’(p) ≥

f p( )≥ f ⎣⎡(16Rr−5r2 2)1⎤⎦ Đến đây ta chỉ việc thay 16 5

p = Rrr vào (1) rút gọn ta đpcm việc rút gọn thời gian dễ nhầm lẫn Chính ta thay 16

p = Rrr vào đẳng thức thứ công thức 17 Ta cần chứng minh:

( ) ( ) ( )

( ) ( ) ( ) ( )

2

2

2 2 2

8 16 10

16

8 16 16

R r R r Rr r

R r

R r R r Rr r R r r

− + + − − ≥ −

⎡ ⎤

− ⎣ + − − ⎦− − ⇔ ( )

( )( ) ( )

2 2 2

2 2

8 16 24 11 10

16 16 24 11 16

R r R Rr r

R r

R r R Rr r R r r

− + − + ≥ − − − + − −

(16 5 16)( 24 112 10 2) (8 7 10 16) ( 24 11 2) (16 5 8)( 7 )

R r R Rr r r R rR Rr r R r R r

⇔ − − + + ≥ − ⎣ − + − − − ⎦

(16 5 )(16 24 21 2) (8 7 )(32 88 75 2)

R r R Rr r R r R Rr r

⇔ − − + ≥ − − +

(16 5 )(32 48 42 2) (16 14 )(32 88 75 2)

R r R Rr r R r R Rr r

⇔ − − + ≥ − − +

Ta thấy 16R−5r≥16R−14r=2 8( R−7r),

( 2)

(111)

Từ điều ta có đpcm, tức (1) giải

Đẳng thức xảy

2

0

, 16

r

a b c

p Rr r

=

⎧⎪ ⇔ = = ⎨

= −

⎪⎩ hoán vị Bài (Vũ Đình Quý) Cho a, b, c > CMR: a b c 2 3abc2 2

b + + +c a a b+b c+c a

Gii

Đặt a x,b y,c z xyz (x y z, , )

b = c = a = ⇒ = >0

Bài toán trở thành: Cho xyz = CMR: x y z xy yz zx

+ + + ≥ + +

Chuyển , ,p R r ta toán: Cho pr2 =1 CMR: 2 4

p

Rr r

+ ≥ +

Ta có: p2 ≥16Rr−5r2 ≥3 4( Rr+r2)≥27r2 ⇒ p3 ≥27pr2 =27⇒ ≥p 3

2 2

3 3 4

3

p p

p p

Rr r p p

+ ≥ + = ⋅ + ≥ ⋅

+ ≥4 Đẳng thức xảy ⇔a= b= c

Bài 9. Cho , ,x y z độ dài cạnh ∆ k số thực ≥ 2,6 CMR:

2 2

3 y

x z

k

x kyz y kzx z kxy

+ + ≥

+

+ + + (*)

Gii

Theo bất đẳng thức B.C.S ta có: ( )2 ( )

x

x x x kyz

x kyz

⎛ ⎞

≤⎜ ⎟ + +

⎝ ⎠

∑ ∑ ∑

( )2 ( ) (

2 )

x x +kyzx x +kxyz

∑ ∑ ∑ Suy ra:

( )2

2 3 3

x x

x kyz x x y z kxyz

+ + +

∑ ∑

∑ + (1)

Mặt khác:

( )( ) ( )

3 3 3 2 3 1

x + y +z + kxyz= x+ +y z x + y +zxyyzzx + k+ xyz

( )

( 2)

2p p 6Rr k 3r

(112)

Từ (1) (2) ta thấy để chứng minh (*) cần chứng minh:

( ) ( ) ( )

2

2

2 3

4

1

6

p

k p p Rr k r

k

p Rr k r

2

⎡ ⎤

≥ ⇔ + ≥ ⎣ + − − ⎦ +

+ − −

(4k 5)p2 54Rr k( 1) 27r2 ⇔ − − − +

Đặt f k( ) (= 4k−5)p2 −54Rr k( − +1) 27r2 ⇒ f′( )k =4p2 −54Rr≥0 (do

2 ( )

f k 16

pRrr ) ⇒ đồng biến theo k

⇒ƒ(k) ≥ƒ(2,6) = 5, 4p2 −86, 4Rr+27r2 =5, 4(p2 −16Rr+5r2)≥0 Vậy (*) chứng minh Đẳng thức xảy ⇔k= 2,6 a= b= c

Bài 10. Cho a b c, , >0;a+ + + =b c 4abc CMR: a b c 1

a b c

+ + ≥ + + Gii

Chuyển , ,p R r ta toán tương đương sau: Cho p+ =1 4pr2 CMR: p r2 ≥4Rr+r2

Ta có:

3

2 27 1 3

27 p

pr ⇒ ≥ + ⇒ ≥p p

Ta cần chứng minh: p p( + ≥1) 4( Rr+r2) Mặt khác: p2 ≥16Rr−5r2 (1)

Do p≥ nên 4p2 ≥9(p+1) ⇒ 4p2 ≥ ⋅9 4pr2 ⇒ ≥p 9r2 (2)

Từ (1) (2) suy p p( + ≥1) 4 4( Rr+r2) tức toán đã được giải quyết

Đẳng thức xảy ⇔a= b= c=

Bài 11 (Nguyễn Duy Khánh).

Cho a b c, , ≥0 CMR: ( ) ( ) (

( )

)2

2

32

12 abc a b c a b

a b

ab a b

+ + − + ≥ +

+ ∑

∑ ∏

(113)

BIến đổi vế trái:

( )2 ( )2 ( )( )( ) ( )

2

4

4

a b c Rrp pr

a b a b b c c a abc R r

ab abc abc pr r

+ +

+ =∑ = + + + + = = + ∑

( ) ( ) ( )

( ) ( )

( )

2 2 2 2 2 2

2 2

2

2

32 32 4

12 12

16

4 12 12

abc a b c a b p r p Rr r Rr r

p R r a b

p Rr r

R

⎡ ⎤

+ + − ⎣ − − − + ⎦

+ = +

+ − − = +

∑ ∏

Theo định lý ta có p2≤2R2 +10Rrr2 +2 R R( −2r) (R−2r)

Suy ra: ( ) ( )

( )( ( )) ( )( )

2

2

2

4 2 2

12

8 2 16 2

12 12

R Rr r R r R R r

VP

R

R r R r R R r R r R r

R R

⎡ − − + − − ⎤

⎣ ⎦

≤ +

− − + − − −

≤ + ≤ +

Do ta cần chứng minh:

( ) ( )( ) ( ) ( )( ) ( )2

2

4 R r 12 16 R 2r R r R 2r 16 R 2r R r 4 R 2r 0

r R r R

+ ≥ + − − ⇔ − ≥ − − ⇔ − ≥

Đẳng thức xảy ⇔ R=2r⇔ = =a b c

Bài 12 (Nguyễn Duy Khánh).

Cho a b c, , ≥0 CMR: ( ) ( )

2 2

8

a b a b b c

b c a b

a b c

+ + +

≥ + + +

∏ ∏

Gii

Chuyển , ,p R r ta cần chứng minh:

( )

( 2)( 2)( 2)

2 2

2 2

8x y z x y y z z x

x y z

x y z

+ + + ≥

+ −

∏ Dễ thấy

2

2R VT

r

=

(114)

( ) ( ) ( )

( ) ( )

2 2

2 2

2

2 2 2

2 2

2

1

2 16

1 16

x y z xy yz zx xyz x y z

x y z

p Rr r p Rr r Rrp

R r p

⎡ ⎤

+ + ⎣ + + − + + ⎦

= −

⎡ ⎤

− − ⎣ + + − ⎦

= −

Ta có: (p2 +4Rr+r2)2 −16Rrp2 =p4 +2 4( Rr+r2)p2 +(4Rr+r2)2 −16Rrp2

Mặt khác: 2 4( 2) 9 2; 4( 2)2

3 2

p Rr Rrp

Rr+r pRrp Rr+r ≤ ⋅ =

( 2 2)2 2 2( 2 7 )

4 16

2 Rr

p Rr r Rrp p p

⇒ + + − ≤ −

Lại có 4 4 3 4

2 Rr

pR + Rr+ rR + Do x y2 + y z2 +z x2 ≤4R p2 Mà x2 + y2 +z2 ≤8R2 +4r2 Vậy ( )

2 2 2

2 2

8 4 2

1 16

R r R p R

VP VT

R r p r

+

≤ − = =

(115)

E CÁC BÀI TỐN TỰ SÁNG TẠO

Phải nói “sáng tạo” ởđây khơng có nghĩa tơi người tìm chúng mà độc lập tìm tốn Trong hàng triệu người u tốn khắp nước khơng thiếu người tìm phương pháp G.L.A chí tìm từ lâu phát triển tầm cao tơi nhiều Tuy nhiên, chưa có tài liệu trình bày cách hệ thống Cũng có người phương pháp khác tìm tốn để minh họa cho phương pháp họ Do hạn chế việc đọc tài liệu nên chúng xuất ởđâu chưa Có điều tốn tìm cách dễ dàng từ công thức đơn giản xây dựng phần C Chính

thừoi gian để tìm tốn cịn nhanh việc tìm tốn tài liệu nhận để bị mang tiếng

I CÁC BÀI TỐN KHƠNG ĐIỀU KIỆN

1 Cho a b c, , >0 CMR:

2 2

3

2

8

a b b c c a ab bc ca

c a b a b c

+ + + + + + + ⋅ + + ≥

+ + (*) Gii

Áp dụng công thức C ta cần phải chứng minh:

( )

3

2

2 2

8 8 2

R r Rr r

r p Rr r

− + + ⋅ + ≥ − −

9

2

(1)

Theo định lý ta có: p2 ≤4R2 +4Rr+3r Do để chứng minh (1) ta cần CM:

( )

2

3

2

2 2

3 19

4 4 2 1

8 4 4 3 8 2 8 4 4

R Rr r R Rr r

r R Rr r Rr r r R Rr r

⎛ ⎞

+ +

+ ⋅ ≥ ⇔ − ≥ ⎜ − ⎟

− +

⎝ ⎠

+ + − −

( )

( )

( )

( ) ( ) ( )

2

2 3

3

2

3

4 8 2 3 2 4

8 2 2 4

R r R R r R r Rr R R

r r

R r

R R

r

r r

⎡ ⎤

− −

⇔ ≥ ⋅ ⇔ − ≥ ⎢ + ⋅ + ⎥

⎡ ⎤ − ⎣ ⎦

+ ⋅ +

⎢ ⎥

⎣ ⎦

Ta nhận thấy: 3( )4R 2 4R 4 3 3( )4R 6R

r + ⋅ r + ≤ ⋅ rr

Suy ra: 3 18 8( )3 8 2( )2

2

R R

VP Rr R R r VP

r

≤ ⋅ = = ≤ − =

(116)

2 Cho a b c, , >0 CMR: 8(a+ +b c)2(a b2 +b c2 +c a2 2)+64a b c2 2

2

( ) ( )2 ( )2 ( )2 ( ) (2 ) (2 )

6 a b c abca b b c c aa b b c c a

≥ + + ⎣ + + + + + ⎦+ + + +

Áp dụng công thức 1, 5, 14 C ta cần phải chứng minh:

( )2 ( )

2 2 2 2 2 2

8p ⎣⎡ 4Rr+r −2p r ⎤⎦+64p r ≥6p r 2p −8Rr−2r +16p R r

( )2 2 2 ( 2 2) 2 2 2

2 4R r 4p 16r p 4Rr r 4R 4R 4Rr 3r ⇔ + − + ≥ − − + ⇔ + + ≥ p (đúng theo định lý 3)

Comment: Nếu toán giải phương pháp đại số lời giải dài dịng với G.L.A lời giải lại ngắn gọn đẹp mắt Qua đó, ta rút điều không thểđánh giá tốn qua vẻ bề ngồi (trơng đẹp mắt hay cồng kềnh) mà phải dựa vào nội dung (tức giải)

3 Cho a, b, c > CMR: ( 8(a)(3 b3 )(c3) ) 4(a2 b2 c2) ab bc ca

a b b c c a

+ + ≥ + + − + + + + +

Gii

Áp dụng công thức 1, 14 C ta cần chứng minh:

( ) ( 2) 2

2

8 12 2

1 24

4 4 4

p p Rr p Rr r p p

pRr Rr r Rr Rr r

− − −

≥ − ⇔ − ≥

+ + −9

2

2

2

15

p p

Rr Rr r

⇔ − ≥ +

( ) ( )

2 15

2

Rr R r

p

R r

+ ⇔ ≥

+

Ta biết 16

pRrr việc chứng minh:

( ) ( )

2 15

16

2

Rr R r

Rr r

R r

+ − ≥

+ đơn giản (⇔

2

4R ≥3Rr+10r )

Đẳng thức xảy ⇔a= b= c

4 Cho a, b, c > CMR: ( )

3 3

5

a b c abc

b+c+ c+a +a+b+ a +b +c

Gii

Áp dụng công thức 4, 8, 14 ta cần chứng minh:

( )

2 2

2

8 5

4 2 12

p Rr r pr

Rr p p Rr

− +

+ ≥

(117)

( ) ( )

2 2 2

2

8 3 1 14

4 2 12 6 12

2

12

p Rr r r p Rr r p Rr r

Rr p Rr Rr p Rr

− + − + − −

⇔ − ≥ − ⇔ ≥

− −

( 2)( ) ( )

3 p 14Rr r p 12Rr 2Rr p 12Rr 3r2

⇔ − + − ≥ − − (*)

Do p2 ≥16Rr−5r2 nên

( ) ( ) ( ) ( )

( ) ( ) ( ) ( )

2 2 2 2

2

2 14 12 16 12

3 12 15

p Rr r p Rr r p Rr r p Rr r

p Rr Rr r Rr r R r Rr

⎧ − + = − − + − + ≥ − − ⎪

⎪ − ≥ − = + − ≥ ⎩

Nhân vế theo vế (1) (2) ta (*) tức toán chứng minh

Đẳng thức xảy ⇔a= b= c

5. Cho a, b, c > CMR:

2 2

3

2

8

a b b c c a ab bc ca

c a b a b c

+ + + + + + + ⋅ + + ≥ + + 2

9

a b b c c a ab bc ca

c a b a b c

+ + + + + + + + ≥

+ + (1)

Gii

Áp dụng công thức ta cần chứng minh:

( )

2

2 9 1

8

R r Rr r

r p Rr r

− + + ≥ − −

2

(*)

Theo định lý ta có: p2 ≤4R2 +4Rr+3r Do để chứng minh (*) ta cần chứng minh: 15

2

Rr r Rr r

R r R r

r R r r R

⎛ ⎞

+ +

− + ⋅ ≥ ⇔ − − ≥ ⎜ − ⎟

− ⎝ −r

( ) ( )

( )( ) ( )( ) 2

4 9 2 2

2

R r R R r 4 9

R r R r Rr r Rr

r R r R r Rr r

− −

⇔ ≥ ⋅ ⇔ − − + + ≥

− − + +

Ta có: VT ≥(2Rr)(2R− +r 9r2)=4R2 +2Rr−2r2≥9Rr=VP

Vậy (1) chứng minh Đẳng thức xảy ⇔ a= =b c

6. Cho a, b, c > CMR:

( )

3 3

3

162 9

a b c abc

abc a b c

(118)

Áp dụng công thức 14 ta cần chứng minh:

( ) 2 2

2 2

12 162 12 162

9

p p Rr pr p Rr r

pr p r p

− −

+ ≥ ⇔ + ≥

Đặt ( )

2 2

2

12 162

p Rr r

f p

r p

= + ⇒ ( ) ( )

4

2

2 3

2 162 324

0

p r

p r

f p

r p p r

′ = − = ≥

Do ƒ(p) đồng biến theo p tức f p( )≥ f( 16Rr−5r2)

Để chứng minh toán ta cần chứng minh

2

2

16 12 162 9 3 6 162 16 16

Rr r Rr r R r r

r R

r Rr r

− − + ≥ ⇔ − − ≥ −

r

− −

( ) ( )

4 6 16 16 5 24 16 29 16

R r R r

R r r R

r R r

− −

⇔ ≥ ⋅ ⇔ − ≥ ⇔ ≥

r (đúng)

Vậy toán chứng minh Đẳng thức xảy ⇔ a= =b c

Mở rộng: Tìm sốk tốt cho BĐT sau đúng:

( )

3 3

3 27

a b c kabc k

abc a b c

+ + + ≥

+ + +

Với cách làm tương tự ta dễ dàng tìm 729 k =

8. Chứng minh với a, b, c dương ta có:

( ) ( )

( )( )

3 3

4 4 2

12 a b c 5 abc a b c

a b c a b c a b c

+ +

+ + + ≥

+ + + + + + (*)

Gii

Áp dụng công thức 1, 4, 14 ta cần chứng minh:

( )

( )

( )

2 2

2 2

4 2

12 12

5 16

p p Rr

p r

p p Rr r

p Rrp Rr r

+ ≥

(119)

( )

( )

( ) ( ) ( )

( )

2 2

2 4 2 2

2

2 2

2 4 2 2

12 12

4

8 16 2 4

4 28 16

8 16 2 4

p Rr p r

p Rr r p Rrp Rr r

p Rr r p Rr r p Rr r

p Rr r p Rrp Rr r

− ⇔ − ≥ − − − − + + − + − + + + ⇔ ≥ − − − + + ( ) ( ) ( ) ( )

2 2

2 4 2 2

8 14 16

0

8 16 2 4

p Rr r p Rr r p Rr r

p Rr r p Rrp Rr r

− + − + + +

⇔ −

− − − + + ≥ (1)

Đặt VT (1) ƒ(p) Ta có ƒ\(p) ≥ 0, f p( )≥ f( 16Rr−5r2)

Mà ( ) ( ) ( ) ( )

( ) ( )

2

2

2 2

2 16

16

8 2 4 5 16 5

R r Rr

Rr r

f Rr r

Rr r

r

R r Rr

+ − − −

− = −

− + − − r

( ) ( )( ) ( ) ( )

( )( )

2

2 2

32

16 16 2 0

8 32 64 27 8 7 32 64 27

R r R r

R r R r R r

R r R Rr r R r R Rr r

− −

− − −

= − =

− − + − − + ≥

Vậy (*) chứng minh Đẳng thức xảy ⇔ a= =b c

9. Cho a, b, c > CMR: 2

3

7

2

a b b c c a a b c a b c

c a b ab bc ca abc

+ + + + + ≥ ⋅ + + + ⋅ + +

+ + (*)

Gii

Áp dụng công thức 1, ta cần chứng minh:

( ) ( ( ) )

( ) ( ) ( )( )

2 2

3

2

2

3 2

2

8

7

4

2 4

4

6 21 4 3

4

4

4

24 21

4

p Rr r p

R r

r Rr r r

R Rr r

R r R Rr

R r

R r R r

R R r

R r

R r A rA r

− − − ≥ ⋅ + ⋅ + − + ⇔ − ≥ ⋅ + + + − + − + − ⇔ − ≥ ⋅ + ⋅ + + +

r r r

(trong A=3(4R2 +4Rr+3r2)r )

( ) ( )( ) ( )( 2) ( )( )

2

5

6 21 3

3

r R r R r

R r R R r A rA r r R r R r

A rA r

+ +

⇔ + ≥ + ⇔ + + + ≥ + + + +

(120)

Suy ra: 3(A2 +3rA)≥2r 27 4( R2 +4Rr+3r2)≥2 10r( R+7r)

Do (2) VTr R( +2r)(20R+41r)≥VP

Vậy (*) chứng minh Đẳng thức xảy ⇔ a= =b c

Bài 11. Cho a, b, c≥ CMR:

2 2 2 2

a b c a b c

ab bc ca

a bc b2 ca c ab

+ +

+ + ≤

+ +

+ + +

Gii

Trước tiên ta thấy 2

p VP

Rr r

=

+ Công việc ta cần làm trước tiên qui VT

về , ,p R r

Ta có: ( )( )

( )( )( )

2

2 2

2

2 2

a b ca c ab

VT

a bc b ca c ab

⎡ + + ⎤ ⎣ ⎦ = + + + ∑ ( )( ) ( ) ( ) ( ) ( )

2 2 2 2 2

2 2 3 3 3 3

2

9

a b c a b b c c a abc a b c abc ab bc ca

a b c abc a b c a b b c c a

+ + + + + + + − + + =

+ + + + + +

Áp dụng đẳng thức 1, 4, 14, 16 phần C ta được:

( ) ( ) ( )

( ) ( )

2

2 2 2

3

2 2

2 4

9 12 12

p Rr r p r pr p Rr r pr R r

VT

p r p r p Rr r R r p R

⎡ + − ⎤+ − − − + ⎣ ⎦ = ⎡ ⎤ + − + ⎣ + − ⎦ ( ) ( ) ( ) ( )

2 3 3

3

2 4 3

2 4 4

9 48 24

pr R r p r p r pr R r pr R r

p r p r Rr p r R r Rp r

+ − + − + − + = + − + + − ( ) ( ) ( ) ( ) ( ) ( ) 2 3

2 4 3 2

2 4

9 4 72 4 72

pr R r pr R r p R r r R r

p r p r r R r Rp r p r p r R r Rrp

⎡ ⎤

+ − + ⎣ + − + ⎦

= =

+ + + − + + + −

Ta cần CM: ( ) ( )

( )

2

2 2 2 4 2

2

4 9 4 2 4 72

p R r r R r

VT 2

Rr r p r p r R r Rrp Rr r

+ − +

≤ ⇔ ≤

+ + + + − +

Đặt p2 =t ( )

( )3 2

1

9 4 72

f t

tr t r R r Rrt

= + + + − ( ) ( ) 2 2

8 72 0

9 4 72

t r Rr

f t

tr t r R r Rrt

+ −

′ = − < ⎡ + + + − ⎤

⎣ ⎦

(121)

Do ƒ(t) nghịch biến theo t tức f t( )≤ f(16Rr−5r2) Suy ta cần chứng minh

( ) ( )

( ) ( ) ( ) ( )

( ) ( ) ( ) ( ) ( ) (

2

2 3

2 2

3 2

2

4 16 16 72 16

2 4 16 16 72 16

R r r R r

Rr r

Rr r r Rr r r R r Rr Rr r

)

R r r R r R r r R r r R r Rr R r

+ − + ≤

+ − + − + + − −

⇔ + − + ≤ + + − + − − −

( )2 ( ) ( )2 ( )

4 16R 5r 16r R 5r 4R r 72 16R R 5r

⇔ − + − + + − − ≥0

( ) ( )

( )

2 2 2

2

2

4 256 160 25 144 45 16 1152 360 16 64 64 16

R Rr r Rr r R Rr r R Rr

R Rr r R r

⇔ − + + − + + + − + ⇔ − + ≥ ⇔ − ≥

Comment: Để chứng minh toán ta vài lời giải khác nhiên lời giải phải dùng đến phân tích phức tạp dễ nhầm lẫn Giải G.L.A ta dễ xây dựng sẵn công thức nên tiện cho việc kiểm tra ta lại xét đạo hàm xem biểu thức sau qui , ,p R r đồng biến hay nghịch biến để qui p vềR r cách thích hợp Việc giảm biến toán cồng kềnh có ý nghĩa lớn lao

Đẳng thức xảy ⇔

( ) 2 16 , 0 R r

a b c

I G

p Rr r

a b c r = ⎡ ⎢ ⎡ = = ⎢⎧⎪ = − ⇔ ≡ ⇔⎢ ⎢⎨ ⎢ = = =

hoặc hoán vị

Bài 12. Cho a, b, c > CMR: ( )

2

3 2 2 3

2

a b c

a abc b abc c abc

b c c a a b

+ + + + + + + ≥

+ + + (1)

Gii

( ) ( )( ) ( )

( )( ) ( )

2 3

3 3

3 3 2

1 1

1

2

1 1

2

2

a b c

a b

a b c abc

a b b c c a a b

a b c abc a b c

a b b c c a

+ + + ⇔ + + + + + − ≥ + + + + ⇔ + + + + + ≥ + + + + + ∑

Áp dụng đẳng thức 1,3,12,14 phần C ta cần chứng minh:

( 12 ) 2 2 5( 8 2 )

4

p Rr r 2

p p Rr pr p Rr r

Rrp

+ +

⎡ − + ⎤⋅ ≥ − −

⎣ ⎦

(122)

Đặt p2 =t ( )

f t =VTVP Ta có: ( ) 12 2

4

t Rr r t Rr r

f t

Rr

+ + + − +

′ = −5

2

2 18 0

4

t Rr r t Rr r

Rr Rr

− + − +

= − = ≥ (do t≥16Rr−5r2)

Do ƒ(t) đồng biến theo p tức để (*) ta cần chứng minh trường hợp

2 16 5

p = Rrr ( )

2

2 16 5 r R 2r

p Rr r

R

= − + (tùy độ chặt)

Mặt khác:

( ) ( ) ( )

( )( ) ( ) ( )( ) ( )

2 2

2

2

2

16 12 16 5 16

5

4

4 20 8 7 5 8 7

4 2

Rr r Rr r Rr r Rr r

Rr r Rr r

Rr

Rr r Rr r Rr r R r R r R r

Rr R

⎡ − − + ⎤ ⎡ − + + ⎤

⎣ ⎦ ⎣ ⎦ ≥ ⎡ − − − ⎤

⎣ ⎦

− − − −

⇔ ≥ − ⇔ ≥ −

( 2) 2

2 20R 19Rr 3r 40R 35Rr 6r 3Rr 2r R

⇔ − + ≥ − ⇔ ≥ ⇔ ≥

(**)

Dễ thấy (**) BĐT sai thay p2 =16Rr−5r vào (*) ta thấy độ chênh VT VP nhỏ Do ta hồn tồn tự tin thay

( )

2 16 5 r R 2r

p Rr r

R

= − + (*) ta có điều phải chứng minh mặc

r2(R 2r) R

− một đại lượng rất bé Trong trình giải việc biến đổi dẫn

đến bất đẳng thức ngược dấu (**) tránh khỏi Rõ ràng để

“chắc ăn” ta thay p2 16Rr 5r2 r2(R 2r)

R

= − + (*) việc làm lại

cồng kềnh so với thay p2 =16Rr−5r2 rất nhiều Qua phân tích ta đã

thấy bắt buộc phải thay p2 16Rr 5r2 r2(R 2r) R

= − + vào liệu có phải

cần thiết ởđâu có p2 đều thay bằng 16Rr 5r2 r2(R 2r)

R

− + hay khơng? Với bạn có “sức khỏe” khả tính tốn tốt việc thay hết vào cá nhân tơi có ngun tắc biến đổi đơn giản tốt Quay trở lại việc thay 16

p = Rrr vào (*), liệu có phải việc làm vo tác dụng khơng? Xin trả lời “khơng” qua q trình ta ước lượng độ

chênh VT − VP có “cảm nhận” cần thay

( )

2

2 16 5 r R 2r

p Rr r

R

(123)

2 4

p + Rr+r cần thay p2 =16Rr−5r2 thơi tất nhiên VP buộc phải thay 16 5 r2(R 2r)

p Rr r

R

= − + Tiếp tục q trình phân tích, việc tính tốn

thì thay p2 16Rr 5r2 r2(R 2r) R

= − + vào hai biểu thức

2 4

p + Rr+r p2 −12Rr+2r2 biểu thức lại p2 =16Rr−5r2 độ phức tạp tính tốn tương đương thay

( )

2 16 5 r R 2r

p Rr r

R

= − + vào biểu thức p2 +4Rr+r2 ta dơi được

lượng p2 −12Rr+2r2 vào biểu thức p2 −12Rr+2r2 lượng dôi (so

với thay p2 =16Rr−5r2) p2 +4Rr+r2 Chính việc thay

( )

2 16 5 r R 2r

p Rr r

R

= − + vào biểu thức 12 2

pRr+ r tốt nhiều so với thay vào biểu thức p2 +4Rr+r2 Với những bạn đã “dày dạn” kinhnghiệm

“chiến đấu” nhìn lướt qua biết thay cho hợp lý rồi, với bạn làm quen với BĐT trình phân tích giúp cho bạn phần giải tập sau Bây ta bắt đầu trình thay nhé!

( ) ( )

( )

2

2 2

2

2

2

16 12 16

4

5 16

2

r R r

Rr r Rr r Rr r Rr r

R

Rr

r R r

Rr r Rr r

R ⎛ − + − − + ⎞ − + + ⎜ ⎟ ⎝ ⎠ ⎛ − ⎞ ≥ ⎜ − + − − ⎟ ⎝ ⎠ ( ) ( ) ( ) ( ) ( ) ( ) ( ) ( )( ) ( ) 2 2

2 2

2

4 20

2

4

2

4

2

2 2

2 20 19 40 35

r R r

Rr r Rr r

R r

Rr r

Rr R

r R r

R r R r

R r R r

R r

R R

r R r R r

R r

R Rr r R Rr r R r

R ⎛ − + − ⎞ − ⎜ ⎟ ⎛ − ⎞ ⎝ ⎠ ⇔ ≥ ⎜ − ⎟ ⎝ ⎠ ⎛ − + − ⎞ − ⎜ ⎟ ⎛ − ⎞ ⎝ ⎠ ⇔ ≥ ⎜⎝ − + ⎟⎠ − − ⇔ − + + ≥ − + − + ( )( ) ( )

2r R 2r 5R r 8r R 2r 5R r 4R R r R

− −

⇔ ≥ − ⇔ − ≥ ⇔ ≥ (đúng)

(124)

Đẳng thức xảy ⇔

2

, 0

R r

a b c

I G

a b c r

= ⎡

⎢ ⎡ = = ⎢⎧⎪ ≡ ⇔⎢

⎢⎨ ⎢ =⎣ = ⎢⎪⎩ =

(hoặc hoán vị)

Bi 13. Cho a, b, c > CMR:

( )

1 1

4

a+bc+b+ca +c+abab+bc+ca với a+ + =b c (*)

Gii

Biến đổi vế trái Do a+ + =b c nên p=

( )( ) ( )( )( )

( ) [ ( ) ( ) ( )] ( )

( ) ( )

( )( )( )

( ) ( )

2 2

2 2 2 2 2 2

2

2

2 2 2 2

1 1

4

4

a bc b ca

a bc b ca c ab a bc b ca c ab

ab bc ca a b c b c a c a b abc a b c

abc a b b c c a abc a b c a b c

Rr r a b b c c a abc p r

pr r R r p r pr p Rr r p r

+ + + + =

+ + + + + +

+ + + + + + + + + + + =

+ + + + + + + + + + + + − +

=

+ + − + − − +

( ) ( ) ( ) ( )

( ) ( )

2 2

2

2 2 2 4

2 2 3

8

4

4 8

8

2 16

4

Rr

Rr r Rr r r

r r R r r r Rr r r r R r r R r r

R R

Rr R r

r R r r R r r

+ + − +

= =

+ + − + − − + + − + +

= = =

+ − + +

Vậy ta cần phải chứng minh:

( 2)

9

1 2

2Rr ≤4 4Rr+rrR (đúng) Tức (*) chứng minh Đẳng thức xảy ⇔ a= =b c

Bài 14. Cho a b c, , >0;ab+bc+ca=1 CMR: ( ) ( )

3

2 2

25

a b c

a b c

abc

+ + ≥ + + + (*)

Gii

Trước tiên dựa vào điều kiện ta tìm tốn gốc (*)

( )* (a b c)3 24( 2 2) ( )2

a b c a b c

abc

+ +

(125)

( )( ) ( )

( )

2 2

24 a b c 1

a b c ab bc ca

abc a b c

+ + + + + +

⇔ ≥

+ + +

( )( )( ) ( )

( )

2 2

24 a b c

a b b c c a

abc a b c

+ + + + +

⇔ ≥

+ + (1)

(1) toán gốc sau loại bỏđiều kiện Với bạn làm bất

đẳng thức việc đưa dạng (1) gặp nhiều khó khăn Nhưng với

những bạn luyện nhiều bất đẳng thức qui dạng (1) chưa biết nên làm theo cách Dạng (1) dạng sở trường S.O.S lần lựa chọn S.O.S để giải lại không sáng suốt Các bạn giải thử thấy khó khăn gặp phải Khi biết G.L.A việc giải khơg

khó khăn Ta cần phải chứng minh: ( )

2

2

24 4Rrp p Rr r

pr p

− − ≥

( 2)

6 p 8Rr 2r R

r p

− −

⇔ ≥ (**)

Ta thấy VP đồng biến theo p, vấn đề chọn cận để giải toán Dùng định lý p2 ≤4R2 +4Rr+3r2 có đủ mạnh để giải quyết

tốn khơng? Áp dụng định lý (2) p2 ≤2R2 +10Rrr2 +2(R−2r) R R( −2r)

chắc chắn giải toán toán Tuy nhiên ta hạn chế sử dụng định cồng kềnh Quan sát tí ta thấy bất đẳng thức (1) trở thành đẳng thức a= =b c a=2b=2c haớn vị định lý 4

pR + Rr+ r xảy đẳng thức tam giác tam giác

đều Do ta áp dụng định lý dù không muốn ta buộc lòng phải áp dụng (2)

(**) sẽđược chứng minh ta chứng minh được:

( ) ( )

( )

( ) ( )

2

2

6 10 2

2 10 2

2

R Rr R r R R r r Rr r

R

r R Rr r R r R R r

⎡ + + − − − − − ⎤

⎣ ⎦

+ − + − −

( ) ( ) ( ) ( )

3 2 2

2R 10R r Rr 2R R 2r R R 2r 12R r 12Rr 18r 12r R 2r R R 2r

⇔ + − + − − ≥ + − + − −

( )( ) ( )

3 2

2R 2R r 13Rr 18r 6r R R 2r R R 2r

⇔ − − + ≥ − − −

( 2 )(2 2 9 2) 2 6( )( 2 ) ( 2 )

R r R Rr r r R R r R R r

(126)

( ) ( )

2

2R 2Rr 9r 6r R R R 2r

⇔ + − ≥ − − (3)

Nếu 6rR ta có VT ≥ ≥ VP Nếu 6r>R (3) tương đương với:

(2 2 9 2) (2 4 48 144 2)( 2 )

R + RrrRRr+ r Rr R

( )

4 2 3 2

3 2 2

4 81 36 36

4 48 144 96 288

R R r r R r Rr R r

R R r Rr R r Rr r R

⇔ + + + − − ≥ − + − + −

( )

4 2 2

3 2 2

4 32 36 81 56 240 288

64 272 252 81 64 272 252 81

R R r R r Rr r R R r Rr r R

R r R r Rr r R R r Rr r

⇔ + − − + ≥ − + −

⇔ − + + ≥ ⇔ − + + ≥

( )2 ( )2

9

64 16

4

R R r r R r

⇔ − + − ≥ (đúng)

Vậy ( ) ( )

3

2 2

25

a b c a b c

abc

+ + ≥ + + +

Đẳng thức xảy ⇔ a= =b c a=2b=2c haớn vị

Comment: Trường hợp đẳng thức xảy a= =b c dễ nhận thấy cịn trường hợp đẳng thức xảy a=2b=2c ta tìm sau Theo lời giải G.L.A đẳng thức xảy chỉkhi:

( ) ( )

2 2

2

i

2 10 2

9

4

R r a b c

p R Rr r R r R R r

R r

R r

=

⎡ ⎡ = =

⎢ ⎢

⎢⎧ = + − + − − ⎢⎧ ⇔

⎢⎪ ⎢⎪

⎨ ⎨

⎢ ⎢

=

⎪ ⎪

⎢ = ⎢⎩⎣

O nằm g ữ a I K

Điều kiện O nằm I K cho ta biết tam giác phải tam giác cân điều kiện

4

R= r giúp ta tìm đẳng thức a=2b=2c (Xin nhắc lại chút K điểm cho IK=3IG )

Lời giải tóm ngắn lại 10 dịng phần sau bạn

thấy giải dòng sau đưa thêm số lý thuyết từ , ,p R r vào G.L.A

(127)

( )

2 2 1

5 5 8 3

a b c

a+ + b+ + c+ ≤ ab bc ca+ +

Gii

Biến đổi VT:

( )( ) ( )( )( )

( ) [ ( )] ( )

( ) ( )

2 5 1 5 1 25 5

5 5 125 25

a b c abc a b c a b c a b c

VT

a b c abc ab bc ca a b c

+ + + + + + + + +

= =

+ + + + + + + + + +

∑ ∑ 2

( )

( )

2 2 2

2

25

125 25

p r Rrp pr p Rr

pr Rr r p

+ − + − − = + + + + r ( ) ( ) 2

2 2

2

2

13 12 25 13 12

150 100 150 100 125 25

r Rr p

r Rr r Rr r r Rr

r Rr r Rr

r Rr r

+ + + − + − − + +

= = =

+ + + +

+ + + + p2

Ta cần chứng minh:

( ) ( )

2

2 2

13 12 1

150 100 8 4

r Rr p p

r Rr p ab bc ca Rr r

+ +

≤ =

+ + + + +

Xét f p( )=VP VT− , suy

( )

( )

( ) ( )

( )

2 2

2

2 2

2 150 100 12 13 12

8 150 100

2

p r Rr p p r Rr p

f p

Rr r r Rr p

+ + − + + ′ = − + + + ( ) ( ) ( ) 2 2 36 14

8 75 50 3

p r Rr

r R r r Rr p

+ −

+ + + Xét ( )

( ) ( ) 2 2 36 14 75 50

p r Rr

g p

r Rr p

+ = + + ⇒ ( ) ( )( ) ( ) 2 2

36 14 75 50 75 50

r Rr r Rr p

g p

r Rr p

+ + − ′ = + + 16

Do p2 ≥ Rrr2 R≥2r nên g’(p) ≤ ⇒ g(p) nghịch biến theo p, tức là:

( ) ( ) ( ) ( ) ( ) ( ) ( ) ( ) ( ) 2 2 2

16 36 14

8 75 50 3 16 5

16 36 14

8 60 98

R r r r Rr

f p

r R r r Rr Rr r

R r r r Rr

r R r r Rr

− + ′ ≥ − + ⎡⎣ + + − ⎤⎦ − + = − + +

(128)

( ) ( ) ( )

2 4r R+r 16R−5r r≤2 16r R−5r ≤98Rr+60r

Từ điều thấy ƒ’(p) ≥ ⇒ƒ(p) đồng biến theo p

Vậy ta chứng minh ƒ(p) ≥ chứng minh f( (16R−5r r) )≥0

( )

( )

( ) ( )

2

16 13 12 16 16 56 16 49 30

150 100 16

R r r Rr R r r R r R r

R r

R r

r Rr R r r

R r

− ≥ + + − ⇔ − + + +

+ + −

+ ≥

( ) ( ) (( )) ( ( )( ) )

2

2

7 14 105 46

16 1 56 16 1 49 30

3 4 49 30

R r R r

R r

R r R r

R r

R r R r R r

− + −

− +

⇔ − ≥ − ⇔ ≥

+

+ + +

( )2 ( )( ) 2 2

4 49R 30r 21 105R 46r 4R r 784R 5571Rr 2604r

⇔ + ≥ + + ⇔ + + ≥ (đúng)

Vậy toán chứng minh Đẳng thức xảy ⇔

3 a= = =b c

Bài 16. Cho a b c, , ≥0 CMR: 2 16 ( )( )( )

5

a b c abc

ab bc ca a b b c c a

+ + + ⋅ ≥ + + + + +

Gii

Áp dụng công thức 14 phần C ta cần chứng minh:

2 2

2

8 4 7 2

1

5 5

4

p Rr r r p Rr r 4r

R R

Rr r Rr r

− − + ≥ ⇔ − − − ≥ −

+ +

Theo định lý ta có: p2 16Rr 5r2 r2(R 2r)

R

(129)(130)(131)(132)(133)(134)(135)(136)(137)(138)(139)(140)(141)(142)(143)(144)(145)(146)(147)(148)(149)

đôi ựiều tản mạn bất ựẳng thức Jack Garfulkel

Phan Thành Nam

April 26,2009

(150)

ðơi điều tản mạn về bất đẳng thức

(BðT) Jack Garfulkel

Phan Thành Nam

Li ta Khi tơi cịn học sinh, BðT hình học Jack Garfulkel gây ấn

tượng mạnh với bất đẳng thức tuyệt diệu nhất: đẹp, khó đầy bí ẩn

Vậy mà, sau vài năm, bất đẳng thức khơng cịn gây "khó dễ" với

nhiều người Bây nhìn lại điều ñáng mừng thấy bất ngờ

Tuy nhiên, với tơi bất đẳng thức thực sựđáng nhớ Vẫn cịn vẻđẹp giản

dị khiết dù độ khó giảm nhiều; cịn băn khoăn, trăn trở đứng trước vấn đề hóc búa; cịn niềm vui nhẹ nhàng mà sâu lắng

những tìm tịi khám phá nhỏ

Tơi xin chép đơi điều tản mạn vài bất ñẳng thức Jack Garfulkel, gồm

một số suy nghĩ tơi cịn học phổ thơng, số tơi tham gia diễn đàn

Phn 1.T mt li gii “kì l

Xin bắt đầu tốn quen thuộc Jack Garfulkel Bài toán 1. Cho tam giác ABC Chứng minh rằng:

sin( ) sin( ) sin( ) 4(1 sin( ).sin( ).sin( ))

2 2 2

A B C A B C

+ + ≥ +

Ta kí hiệu

Khi A, B, C góc tam giác ta có x, y, z > (*)

Khi đó, Bài tốn 1 viết lại thành

Bài tốn 1a. Cho thỏa

CMR:

Ta chứng minh kết mạnh hơn:

Bài toán 1b. Cho thỏa

CMR:

(151)

(1)

Mặt khác, dễ thấy: nên

Cụ thể hơn, xét hiệu

Ta có: , ,

suy ra:

(2) Cộng (1) (2) vế theo vế, ta có đpcm

Cách thay yếu tố lượng giác biến thực x, y, z kèm điều kiện

tạm gọi “đại số hóa lượng giác” (ngược với cách

làm thơng thường lượng giác hóa đại số) Chúng ta có lời giải đại số kiểu

vậy cho hai toán sau, Jack Garfulkel (thật Bài tốn yếu - tức có

thể xem hệ quả- Bài toán 1) Lời giải chi tiết xin dành cho bạn

Bài toán 2. Cho tam giác ABC Chứng minh rằng:

cos( ) cos( ) cos( ) (1 cos( ).cos( ).cos ))

2 2 2

A B C A B C

+ + ≥ +

Bài toán Cho tam giác ABC Chứng minh rằng:

Như thấy, lời giải "kì lạ" tốn 1.b nói thật chẳng kì lạ

chút Lời giải chẳng qua viết lại dạng ñại số lời giải dựa biến ñổi

lượng giác ñã ñăng THTT 12/2001, có điều lời giải lượng giác cần ñiều kiện

, lời giải ñại số bất ngờ điều kiện Cịn viết

biến ñổi lượng giác dạng ñại số ta sẽđề cập sau

Phn 2.Ti mt tốn Olympic

Chúng ta có ví dụ khác, trình bày cụ thể hơn, cho mối liên hệ cách làm ñại

số lượng giác Sau tốn đề dự tuyển IMO 1995

(152)

trong a, b, c số thực dương cho trước

Nhận xét ñặt

, ,

2 2

a b c

yz zx xy

α = β = γ =

thì hệđã cho trở thành

2 2

2 2 (1)

2 1(2)

x y z xyα yzβ zxγ

α β γ αβγ

 + + = + +

 

+ + + =



Hệ thuộc loại "khơng mẫu mực" có tới ẩn có phương trình,

thực chất tốn cực trị Cụ thể hơn, ta thấy ñặt

với A, B, C góc tam giác, "cốt lõi" tốn BDT quen thuộc:

(*) Tới ñây, có lẽ bạn ñã thấy rõ lời giải toán

Bây giờ, ta khái quát lại toán thành

Bài toán 2. Cho số thực không âm a, b, c, x, y, z thỏa mãn

CMR:

ðây tốn hay Tất nhiên dùng lượng giác hóa phân tích

trên ñể giải, từñẳng thức (2) ta hi vọng có lời giải khác cho

bằng ñại số

Trước hết, thử nhìn BDT (*) quan điểm đại số

Bài toán 3. Cho thỏa , số thực a,b,c

CMR:

Chng minh.

Xuất phát từ lời giải lượng giác

ta thay cho ñể thu ñược biến ñổi

=

2 2 2

2 2 2

[ ( ) ( ) ] [(1 ) (1 ) ] ( )

( ) ( 1 ) ( (1 )(1 ) )

a a c b c b b c bc

a b c b c bc

β γ β γ γ β α βγ

γ β γ β β γ α βγ

− + + + + − + − − +

(153)

Bài toán 3 chứng minh xong

Bây ta áp dụng biến ñổi vào tốn

Giả sử ,

khi a, b, c ñều dương

Với , ,

2 2

x y z

bc ca ab

α = β = γ =

Sử dụng phép biến ñổi chứng minh Bài tốn 3, ta có

2 2 2

( a bγ cβ) ( γ b β c) bc( (1 β )(1 γ ) α βγ)

⇔ − − + − + − + − − − − >

Từđó, ta có lời giải ngắn gọn cho Bài tốn 2 sau

Li gii toán

Tất nhiên ta cần xét Khi ta có

2 2 2

4ax 2yz (2a y z) (4ab z ) (4ac y ) (4ab z )(4ac y )

⇒ + ≥ − − + − + − ≥ − −

Suy ñpcm

ðối với tơi, lời giải thật sựấn tượng Nó kết “chu trình”:

chuyển từđại số qua lượng giác, chuyển ngược trở lại ñại số Tuy nhiên, khơng

hẳn đường để có lời giải Trước đây, có lần tơi đưa tốn lên

diễn ñàn toán học nhận ñược lời giải giống mặt ý tưởng (chỉ dùng BDT

Cauchy) bạn Trn Quc Hồn (K09) kỉ niệm thú vị

Cuối cùng, xin nêu tốn để bạn suy nghĩ

Bài tốn 4. Cho tứ diện vng O.ABC Giả sử góc nhị diện cạnh BC, CA,

AB CMR

2( )

tgα+tgβ +tgγ ≥tg tgα βtgγ + cotgα cotgβcotgγ ≥ cotgα+cotgβ+cotgγ

Phn 3.Vài vn ñề vi ñường trung tuyến

Sau ñây bất ñẳng thức ñẹp khác của Jack Garfulkel: Bài toán 1.

(154)

ñường cao, nửa chu vi tam giác

Cách gần 10 năm tốn khó Một lời giải ñầu tiên

cho chứng minh BDT mạnh

với c cạnh lớn cạnh tam giác Chứng minh dựa bổñề

(*)

Bổñề ñược ñề xuất chứng minh dựa theo ý tưởng hình học (áp dụng BDT

Ptoleme cho tứ giác lồi) Tuy nhiên, ta chứng minh trực tiếp dựa biểu diễn

tường minh ñường trung tuyến

với a=y+z,b=z+x,c=x+y p=x+y+z Chúng ta trở lại Bổñề sau

Trong viết THTT, anh Phm Gia Vĩnh Anhñã ñưa chứng minh

kết mạnh

Bài toán 2.

Hơn nữa, ñây chứng minh ngắn gọn BDT Cauchy (dựa biểu diễn

ñánh giá quen thuộc )

ðối với tốn 2, ta chứng minh ngắn gọn nhờ BDT Bunhiacopski

Lời giải sau ñây dựa theo ý bạn Phùng Trng Thc

Li gii tốn 2.ðểđơn giản, ta cho p=1 Ta có

Kết quảở tốn chặt, biết, bất đẳng thức sau khơng Cũng viết mình, anh Vĩnh Anhđã đưa

BDT sau nhằm “bù đắp” cho khơng BDT

Tuy nhiên, số k=1/4 tốt Thực ta có

Bài tốn 3. Chứng minh tam giác

với

(155)

khó Tuy nhiên, giờđây có lẽ lời giải toán nằm khả bạn

Một hướng khác ñể “bù ñắp” cho BDT khơng

là sau Ta viết lại BDT thành

Từđó xuất câu hỏi giảm hệ số k=1/4 cơng thức ñường trung tuyến ñể

BDT trở thành ñúng Một câu trả lời k=1/12

Bài toán 4. (VMEO I, 1) Cho a, b, c số thực khơng âm có tổng CMR

Bài tốn đặt dựa hướng tiếp cận ban đầu cho tốn Jackgarfulkel Từ

bổđề (*) ta khái qt thành

Bổđề A. Cho số thực a, b, u, v thức có nghĩa Khi điều sau

là tương ñương

(i) (ii)

(ñiều kết luận ñúng ta thay dấu thành )

ðể chứng minh bổñề ta việc liên tục bình phương đơn giản vế

Chng minh tốn 4.

Áp dụng bổđề A với , ta thu ñược

Cùng với BDT tương tự ta có đpcm

Mặt dù số k=1/12 dẫn tới biến ñổi ñại số đẹp lời giải trên, khơng phải

là số tốt Cũng nhưđối với Bài tốn 3, trước chí tìm số tốt

nhất tốn khó, giải khơng phải điều q

khó Cụ thể, kimluan tìm kết sau dồn biến

Bài toán 5. Cho số thực khơng âm a, b, c có tổng Chứng minh

với

Chú ý tốn số tốt đẳng thức xảy

a=b=c=1/3 a=1,b=c=0 hốn vị Một điều thú vị ñây số tốt

(156)

Khi đổi dấu BDT Bài tốn 5 ta tốn sau (đẳng thức đạt ñược

tại chỗ), mà lời giải – ñơn giản – xin ñược dành lại cho bạn

Bài toán 6. Cho a, b, c số thực khơng âm có tổng CMR

Phn 4.Mt dng BDT cha căn

Bổñề A cho ta tiêu chuẩn dễ kiểm tra BDT “khơng tầm

thường” sau

và từđó dẫn tới nhiều tốn thú vị Bây xuất câu hỏi liệu có kết

quả nào, tương tự Bổđề A, để áp dụng cho nhiều biến khơng? Nói riêng,

trường hợp số, liệu có tiêu chuẩn (tương đối dễ kiểm tra) áp ñặt lên số

a, b, c, x, y, z cho ta có

Ởđây thay Tuy nhiên, tiêu chuẩn tổng quát chưa tìm ra,

các BDT dạng tốn khó, gần lại cần cách giải

riêng Chẳng hạn, ta thấy BDT nói mục trước nằm dạng tổng quát này: với

a, b, c không âm, a+b+c=1

với ,

Sau số ví dụ khác cho BDT dạng

Bài tốn 1. Cho số khơng âm x, y, z có tổng CMR:

Nhận xét BDT có dạng

,

ðồng thời ñẳng thức xảy x=y=z=1/3 x=1,y=z=0

Chng minh toán 1.

Ta quan sát mối tương quan biểu thức

(157)

Vậy: , với

Ta có bổđềđơn giản sau cho phép hốn vị biểu thức dấu

Bổñề. Cho số không âm a, b, c, d thỏa mãn a+b=c+d Thì

Trở lại tốn, giả sử c=min(a,b,c) Ta kiểm tra Ta có:

Vì x-z x(y-z) khơng âm nên ta có đpcm

Từđó, áp dụng bổđề ta có:

Và suy

Bài tốn chứng minh xong!

Bài toán 2. (VMEO III, 8) Cho x, y, z số thực khơng âm có tổng

Chứng minh rằng: Chng minh.

Nhận xét dấu "=" xảy x=y=z x=1,y=z=0 (cùng hoán vị) Ta giải

bài cách hốn đổi biểu thức dấu Ta có bổđề sau

Bổđề. Cho só thực A, B, C, D thỏa mãn:

Khi đó:

Chứng minh đơn giản xin dành lại cho bạn

Trở lại toán, ta đặt

Nếu có, chẳng hạn, , , nên

ta có đpcm

Do đó, từ trởñi ta cần xét , , Vì BDT ban đầu có dạng hốn vị vịng quanh nên ta giả sử Khi ta cần

xét trường hợp

(158)

nên áp dụng Bổñề ta có:

(2), với

Lại có:

nên áp dụng Bổđề ta có:

(3) Từ (2) (3) ta có đpcm

*Trường hợp Xét Ta có

nên áp dụng Bổđề ta có:

(4), với

Lại có

nên áp dụng Bổđề ta có:

(5) Từ (4) (5) ta có đpcm

Bài tốn chứng minh xong!

Cũng xuất phát từ dạng BDT trên, ta có tốn sau

Bài tốn

Cho , Chứng minh

Nhận xét BDT viết dạng

Trong lời giải cho tốn trên, tơi sử dụng Bổđề A để suy phải có BDT

kiểu sau (hốn vị (x,y,z) cần)

(159)

ñể thực việc dồn

Tuy nhiên, tốn đưa lên diễn đàn tốn học thầy namdung đã ñề xuất

một Bổñề khác cho phép chứng minh gọn nhiều

Chng minh toán

Trước hết ta có Bổđề sau, chứng minh đơn giản cách bình phương vế

Bổđề Cho số thực a, b, u, v cho thức có nghĩa Khi

Trở lại toán Trong số , , phải có số dấu (tức tích

của chúng ), ta giả sử Khi đó, áp dụng Bổđề, ta có

Ta có

Nếu , cịn thì:

Bài tốn chứng minh xong

Bài tốn kimluan làm mạnh thành kết sau ñây

Bài toán 4. Cho x+y+z=0 CMR

ðẳng thức xảy x = y = z = x =0,y =1,z = -1 ðây BDT ñẹp ấn tượng

nhưng chưa có lời giải ñơn giản cho

Dạng BDT xuất phát từ Bổđề A mở rộng cho nhiều số Sau

ví dụ cho trường hợp số

(160)

ðây tốn khó Lưu ý BDT viết dạng

Chng minh tốn 5.

*Trước hết, để có cảm giác toán, ta xét trường hợp riêng: cho x=z, y=t

Khi đó, với điều kiện , ta cần chứng minh

ðể ý , áp dụng Bổđề A BDT

tương đương với ðiều

*Trở lại tốn tổng quát, ta tìm cách quy trường hợp số Ta hi vọng có BDT

dạng

2 2

1−yz+z + 1−tx+x ≥ 1−xy+x + 1−zt+z (*)

2 2

2 2

2 2

( 1 ) ( 1 )

( ) ( )

0

1 1

( )

1 1

yz z zt z tx x xy x z t y x y t

yz z zt z tx x xy x

z x

t y

yz z zt z tx x xy x

⇔ − + − − + + − + + − + ≥

− −

⇔ + ≥

− + + − + − + + − +

 

 

⇔ −  − ≥

− + + − + − + + − +

 

Bằng tính tốn cụ thể, ta chứng minh thừa số thứ biểu thức vế trái dấu

với z-x

Do BDT (*) tương đương với (**) ðiều thú vị cách hoán vị

ta giả sử có điều Thật vậy, BDT ởđề khơng đổi ta làm việc với

4 số (y,z, t, x), với số BDT (**) trở thành (***) Vì (**)

và (***) phải có ñúng, nên ta gỉ sử (**) ñúng Khi (*)

Sử dụng (*) trường hợp số, ta có

(161)

Bài toán chứng minh xong ðẳng thức xảy x = y = z = t x = z,y = t,xy =1

Dưới hai tốn khác, dạng này, mà lời giải xin ñược dành lại cho

bạn

Bài toán 6. Cho số thực thỏa mãn x+y+z=0 CMR

Bài toán 7. Cho số thực thỏa mãn CMR

(162)

1

PHƯƠNG PHÁP CHUYỂN VỊ TRONG CHỨNG MINH BẤT ĐẲNG THỨC HOÁN VỊ

VÕ QUỐCBÁCẨN

Hiện có nhiều phương pháp mạnh để chứng minh bất đẳng thức

EVcủaVasile Cirtoaje,SOScủaPhạm Kim HùngTrần Tuấn Anh, Nhưng phương pháp phần lớn dùng để giải toán đối xứng, gặp bất đẳng thức hốn vị chúng thường tỏ hiệu Vậy có cách để giải bất đẳng thức hốn vị khơng? Bài viết này, xin chia sẻ bạn kinh nghiệm nhỏ để chứng minh bất đẳng thức hốn vị biến (và đơi ta áp dụng cho bất đẳng thức hoán vị biến) Rất mong nhận ý kiến đóng góp bạn!

Như nói trên, phương pháp chứng minh bất đẳng thức đối xứng nhiều nên ta chuyển bất đẳng thức hoán vị dạng đối xứng việc chứng minh khơng cịn khó khăn Đó kinh nghiệm nhỏ mà chúng tơi muốn giới thiệu bạn đọc, kỹ thuật giúp ta chuyển bất đẳng thức hoán vị thành bất đẳng thức đối xứng để giải, ta tạm gọi "phương pháp chuyển vị"

Để hiểu rõ ý tưởng nó, xét ví dụ sau

Example 0.1 Cho số thực không âma,b,cthỏa mãna+b+c=4.Chứng minh rằng

a2b+b2c+c2a+abc

(Vasile Cirtoaje, Phạm Kim Hùng)

Lời giải. Bất đẳng thức cần chứng minh có dạng

a ab+b bc+c ca+abc

Ta thấy bất đẳng thức hoán vị với đẳng thức xảy a = b = c =

(163)

2

điều này, bạn để ý đến hai biểu thức gạch chân trên, chúng có điều kì lạ? À, ta hốn đổi vị trí cho ta thu bất đẳng thức

a ab+b ca+c bc+abc

Và thật thú vị, lại bất đẳng thức đối xứng cho hai biến a vàc.Vì vậy, ta

có đánh giá kiểu nhưa ab+b bc+c ca+abc a ab+b ca+c bc+abc

là điều tuyệt vời! May mắn thay, điều tương đương vớic(a b)(b c) 0và

chúng ta hoàn tồn đạt điều cách giả sửblà số hạng nằm giữaavà c.Đến đây, ta tìm lời giải cho tốn sau:

Khơng tính tổng qt, giả sửblà số hạng nằm giữaavàc.Khi đó, ta có

a ab+b bc+c ca+abc a ab+b ca+c bc+abc

= b(a+c)2

2

2b+a+c+a+c

3

3

=4

Bài toán chứng minh xong Đẳng thức xảy a = b = c =

a=2,b=1,c=0(cùng hốn vị tương ứng)

Đây ví dụ quen thuộc, có lẽ nhiều bạn cho quen thuộc, hiển nhiên Và bạn, tinh ý thấy việc đánh giáa ab+b bc+c ca+abc a ab+b ca+c bc+abc thực việc sử dụng bất đẳng thức xếp lại cho hai số đơn điệu chiều (a,b,c)và (ab,ca,bc)(với giả thiết blà số hạng nằm giữa) Tuy nhiên, đến với ý tưởng chuyển vị hoàn toàn độc lập với bất đẳng thức xếp lại Chúng ta đến ví dụ sau để thấy rõ điều

Example 0.2 Cho số khơng âmx,y,zcó tổng bằng1.Chứng minh bất đẳng thức sau q

x+y2+qy+z2+pz+x2 2.

(Phan Thành Nam)

Rõ ràng với toán này, việc sử dụng bất đẳng thức xếp lại khó (có thể nói khơng thể), việc sử dụng phép chuyển vị ta áp dụng Và điều thú vị là, với cách phân tích khác lại có phép chuyển vị khác nhau, giúp đưa toán đến kết Chẳng hạn, ví dụ này, có hai cách chuyển vị sau

Lời giải 1. Bất đẳng thức có dạng đồng bậc (ở vế trái)

q

x2+y2+xy+xz+qy2+z2+yz+yx+qz2+x2+zx+zy 2.

(164)

3 xứng, chẳng hạn choyvàz.Để thực hiện, ta để ý biểu thứcyxvàzyđược gạch chân trên, ta chuyển vị biểu thức thu bất đẳng thức

q

x2+y2+xy+xz+qy2+z2+yz+yz+qz2+x2+zx+xy 2.

Và thật thú vị, bất đẳng thức đối xứng choyvàz.Với ý tưởng vậy, cần có

q

y2+z2+yz+yx+qz2+x2+zx+zy qy2+z2+yz+yz+qz2+x2+zx+xy.

Bình phương vế, thu gọn, ta thấy bất đẳng thức tương đương với

y(x y)(x z)(x+y+z)

Điều đạt ta giả sử x = minfx,y,zg x = maxfx,y,zg Với phân tích này, ta đến lời giải tốn sau:

Khơng tính tổng quát, giả sửx =minfx,y,zg,khi theo trên, ta có

q

y2+z2+yz+yx+

q

z2+x2+zx+zy

q

y2+z2+yz+yz+

q

z2+x2+zx+xy,

nên bất đẳng thức ta đưa

q

x+y2+px+z2+y+z 2,

tương đương q

x+y2+px+z2 2x+y+z.

Áp dụng bất đẳng thứcMinkowski, ta có

q

x+y2+px+z2

q p

x+px 2+ (y+z)2 =q4x+ (y+z)2

=

q

4x(x+y+z) + (y+z)2=2x+y+z.

Do đó, bất đẳng thức ta chứng minh xong Đẳng thức xảy x=y=z = 13hoặcx=1,y =z=0và hoán vị tương ứng

Lời giải 2. Nếu bạn khơng thích phép chuyển vị trên, thử chọn phép chuyển vị kiểu khác sau: Hãy ý đến biểu thức gạch bất đẳng thức

q

(165)

4

Nếu ta thực phép chuyển vị cho biểu thức thu bất đẳng thức đối xứng choxvàzlà

q

x2+y2+xy+xz+qz2+y2+zx+zy+qx2+z2+yz+yx 2.

Như vậy, ta cần có

q

y2+z2+yz+yx+qz2+x2+zx+zy qx2+z2+yz+yx+qz2+y2+zx+zy,

hay

x(x2 y2)(y z)

Điều đạt ta giả sửylà số hạng nằm giữaxvàz.Đến đây, ta thu lời giải sau:

Giả sửylà số hạng nằm giữaxvàz,khi dễ thấy

q

y2+z2+yz+yx+qz2+x2+zx+zy qx2+z2+yz+yx+qz2+y2+zx+zy,

nên ta cần chứng minh

q

x2+y2+xy+xz+qy2+z2+zx+zy+qx2+z2+yz+yx 2,

tương đương

q

x+y2+qz+y2+px+z 2xz 2,

hay

x+z+2y2+2

q

(x+y2)(z+y2) 2 px+z 2xz 2. Đặtt= xz(0 t y(1 2y))thì bất đẳng thức viết lại

f(t) =2t+2y2 4+2

q

t+ (1 y+y2)y2+4p1 y 2t 0.

Ta có

f00(t) =

2[t+y2(1 y+y2)]3/2

4

(1 y 2t)3/2 <0,

(166)

5 + Nếuxz=0,ta giả sửz=0,khi đóx =1 yvà bất đẳng thức trở thành

q

1 y+y2+p1 y+y 2.

Bất đẳng thức hiển nhiên đúng, theo bất đẳng thứcMinkowski, ta có

q

1 y+y2+p1 y =

r p

1 y 2+y2+

r p

1 y 2+02

r p

1 y+p1 y 2+ (y+0)2=2 y.

+ Nếu(x y)(z y) =0,ta giả sửy=z,khi đóx =1 2y 0và bất đẳng thức

trở thành q

1 2y+y2+qy+y2+q1 y 2y(1 2y) 2,

tương đương q

y+y2+q1 3y+4y2 1+y.

Nhưng bất đẳng thức hiển nhiên đúng, theo bất đẳng thức

Minkowski, ta có

q

y+y2+q1 3y+4y2 =

q

(py)2+y2+

q

(py)2+ (1 2y)2

q

(py+py)2+ (y+1 2y)2 =1+y.

Phép chứng minh ta hoàn tất

Với ý tưởng chuyển vị vậy, giải nhiều tốn đẹp khó Sau hai ví dụ khác

Example 0.3 Cho số không âmx,y,zthỏa mãnx+y+z=1.Chứng minh rằng

3

q

x y+z3+q3 y z+x3+q3 z x+y3

1

(Phan Thành Nam)

Lời giải. Ta thấy bất đẳng thức cần chứng minh có dạngp3 A+p3 B+p3 C 1,với

A = x y+z3= x3+x2y xy2 y3+2z(x2 y2) +z2(x y) +z3,

B = y z+x3=y3+y2z yz2 z3+2x(y2 z2) +x2(y z) +x3,

(167)

6

Nếu có số số A,B,Ccó tổng khơng dương bất đẳng thức ta hiển nhiên

đúng Thật vậy, giả sử A+B 0thì doC=z x+y3 z x+y 1,nên

3

p

A+p3 B+p3 C p3 B+p3 B+p3 C= p3 C

Bây ta xét trường hợp ngược lại, tức lúc ta có A+B 0,B+C

C+A 0.Khi đó, giả sửz=minfx,y,zg,và đặt

D=y3+y2x yx2 x3+2z(y2 x2) +z2(y x) +z3, vàE= x3+y3 z3

Lúc này, ta có tính chất sau:D+E=B+C 0,và

DE BC= (a c)(b c)(a2+2ab+2ac+bc)(2a2+b2+2c2+2bc+3ca+2ab)

Với tính chất này, ta dễ dàng chứng minh đượcp3 B+p3 C p3 D+p3 E,và ta đưa bất đẳng thức chứng minh

3

p

A+p3 D+p3 E 1,

tương đương

3

q

x y+z3+q3

y x+z3+q3

x3+y3 z3 1.

Thực tương tự trên, ta có

3

q

x y+z3+q3

y x+z3 p3 z3+p3 z3=2z,

nên ta cần chứng minh

x+y z q3

x3+y3 z3.

Đây bất đẳng thức hiển nhiên

(x+y z)3 (x3+y3 z3) =3(x z)(y z)(x+y)

Phép chứng minh ta hoàn tất Đẳng thức xảy x = y = z = 13

hoặcx=1,y= z=0và hoán vị tương ứng

Example 0.4 Cho số khơng âma,b,ccó tổng bằng3.Chứng minh rằng

(3a2+bc+3b2)(3b2+ca+3c2)(3c2+ab+3a2) 900

(168)

7

Lời giải. Khơng tính tổng qt, ta giả sửblà số hạng nằm giữaavàc.Khi đó, với ý đẳng thức sau

(3b2+ca+3c2)(3c2+ab+3a2) (3b2+ab+3c2)(3c2+ca+3a2) = =3a(b c)(b a)(a+b) 0,

ta đưa bất đẳng thức chứng minh

(3a2+bc+3b2)(3b2+ab+3c2)(3c2+ca+3a2) 900

Đến đây, ta thấy

(3a2+bc+3b2)(3b2+ab+3c2) =

= 9b4+3(a+c)b3+ (9a2+ac+9c2)b2+3(a3+c3)b+9a2c2

= 9b4+3(a+c)b3+9(a+c)2b2+3(a+c)3b+9ac(ac ab bc) 17b2ac

9b4+3(a+c)b3+9(a+c)2b2+3(a+c)3b

= 3b(a+3b+c) b2+ (a+c)2 ,

3c2+ca+3a2 3(a+c)2,

nên ta cần chứng minh

9x2b(x+3b)(x2+b2) 900,

vớix=a+c

Áp dụng bất đẳng thứcAM – GM, ta có

9x2b(x+3b)(x2+b2)

10

5xb+x(x+3b) +2(x2+b2)

3

3

,

5xb+x(x+3b) +2(x2+b2) = 10

3 (x+b)

2

3(x 2b)

2 10

3 (x+b)

2=30,

nên từ trên, ta

9x2b(x+3b)(x2+b2)

10 10

3=900.

Bất đẳng thức ta chứng minh xong Đẳng thức xảy a = 0,b=

(169)

8

Nhận xét 1 Bằng cách tương tự, ta giải tốn sau:

Vớia,b,clà số khơng âm có tổng bằng3klà số cho trước p2 k 13 ,tìm giá trị lớn biểu thức sau

P(a,b,c) = (a2+kbc+b2)(b2+kca+c2)(c2+kab+a2)

Khơng có bất đẳng thức hốn vị ba biến sử dụng phép chuyển vị mà phần đơng bất đẳng thức hốn vị bốn biến áp dụng Đầu tiên, sử dụng phép chuyển vị để đưa bất đẳng thức hoán vị cho ba biến, dùng đánh giá thích hợp để chứng minh toán Mời bạn đến ví dụ sau để rõ ý tưởng (đây tốn khó)

Example 0.5 Cho số không âma,b,c,dthỏa mãna+b+c+d=4.Chứng minh rằng

a3b+b3c+c3d+d3a+23abcd 27

(Phạm Kim Hùng)

Lời giải. Trước hết, ta chứng minh bổ đề sau

Bổ đề 0.1 Nếua,b,clà số khơng âm thì

a3b+b3c+c3a+473

256abc(a+b+c) 27

256(a+b+c)

4.

Chứng minh.Bạn đọc tự chứng minh lấy cách sử dụng phép chuyển vị cho biến

Quay trở lại tốn Do tính hốn vị vịng quanh nên khơng tính tổng qt, ta giả sửdlà số hạng nhỏ sốa,b,c,d.Khi đó, ta có

c3d+d3a (c3a+d4) = (c3 d3)(a d) 0,

nên để chứng minh bất đẳng thức cho, ta cần chứng minh

a3b+b3c+c3a+d4+23abcd 27

Đến đây, áp dụng bổ đề trên, ta đưa chứng minh

27

256(4 d)

4 473

256abc(4 d) +d

4+23abcd 27,

1

256(6361d 1892)abc+ 27

256(4 d)

(170)

9 Nếu6361d 1892 0thì bất đẳng thức hiển nhiên 25627(4 d)4+d4 27.Nếu

6361d 1892 0thì ta có

1

256(6361d 1892)abc+ 27

256(4 d)

4+d4 27

1

256(6361d 1892)

(4 d)3

27 +

27

256(4 d)

4+d4 27

=

27(5d

2+270d 473)(d 1)2 0.

Bất đẳng thức ta chứng minh xong Đẳng thức xảy khia=b= c=

d =1hoặca=3,b=1,c=d=0và hoán vị tương ứng BÀI TẬP ĐỀ NGHỊ

1 Cho số thực không âma,b,cthỏa mãn khơng có hai số đồng thời bằng0

Chứng minh

a2+b2+c2

ab+bc+ca+

4abc

a2b+b2c+c2a+abc

(Võ Quốc Bá Cẩn) Giả sửa,b,clà số thực không âm thỏa mãna2+b2+c2 = 3. Chứng minh các

bất đẳng thức sau

(a) a2b+b2c+c2a 2+abc;

(b) a3b2+b3c2+c3a2

(Vasile Cirtoaje) Chứng minh với số thực dươnga,b,ccó tổng bằng3,bất đẳng thức sau

luôn

a

b+c2 +

b

c+a2 +

c

a+b2

3

(Phạm Kim Hùng) Chứng minh với số thực dươnga,b,ccó tổng bằng3,bất đẳng thức sau

luôn

r

a

b2+3 +

r

b

c2+3+

r

c

a2+3

3

(171)

10

5 Giả sửa,b,clà độ dài ba cạnh tam giác có chu vi bằng1.Chứng minh

p

2a+b3+p2b+c3+p2c+a3 p2+1.

(Võ Quốc Bá Cẩn) Giả sửa,b,clà số thực không âm thỏa mãnab+bc+ca = 3.Tìm tất số

thực khơng âmksao cho bất đẳng thức sau

(ka+b)(kb+c)(kc+a) (k+1)3

(Michael Rozenberg) Choa,b,c,dlà số thực không âm thỏa mãna+b+c+d=3.Chứng minh

ab(b+c) +bc(c+d) +cd(d+a) +da(a+b)

(Phạm Kim Hùng) Choa,b,c,dlà số thực không âm thỏa mãna+b+c+d=3.Chứng minh

ab(a+2b+3c) +bc(b+2c+3d) +cd(c+2d+3a) +da(d+2a+3b) 6p3

(172)

DỒN BIẾN "THỪA – TRỪ"

Võ Quốc Bá Cẩn - ĐH Y Dược Cần Thơ

Phương pháp dồn biến từ xuất nay, thể vai trị tính hiệu việc giải tốn bất đẳng thức Tuy nhiên, phương pháp có nhiều nhược điểm mà chúng ta, sử dụng dễ dàng nhận thấy Một nhược điểm khó sử dụng với bất đẳng thức chứa số bất đẳng thức dạng phân thức (phân thức bậc cao) Bài viết nhỏ này, xin chia sẻ bạn đọc phương pháp dồn biến giúp giải nhiều toán ba biến (mảnh đất màu mỡ bất đẳng thức nay) thuộc hai dạng Phương pháp giúp chúng tơi giải nhiều tốn khó mà vài số tốn mở Chúng tơi xin gọi phương pháp "dồn biến thừa – trừ"

Để bắt đầu, ta xét ví dụ sau, tốn tưởng chừng giải dồn biến sơ cấp

Ví dụ 1 Giả sửa,b,clà số thực khơng âm thỏa mãna+b+c=1.Chứng minh rằng

1

p

a2+ab+b2 +

1

p

b2+bc+c2+

1

p

c2+ca+a2 4+

2

p

3

(Xtar)

Lời giải.Để ý bất đẳng thức cho có đẳng thức xảy a = b,c = nên dùng dồn biến để giải ý tưởng ta dồn biến trung bình cộng Khi đó, giả sử

c=minfa,b,cgvà đặtP(a,b,c) = p

a2+ab+b2 +

1 p

b2+bc+c2 +

1 p

c2+ca+a2,ta phải chứng minh

P(a,b,c) P a+b

2 ,

a+b

2 ,c

Đến đây, ta thấy việc tách bình phương(a b)2từ

1

p

b2+bc+c2 +

1

p

c2+ca+a2

2

r

a+b

2

+ c(a2+b)+c2

là phức tạp, phải trải qua nhiều lần trục Vì vậy, dù có tách thành cơng việc đánh giá ta lúc sau khó khăn nhiều (phải nói khó) Đây lí làm ta tưởng phương pháp dồn biến sơ cấp không hiệu cho

(173)

tốn đẹp khó Bây ý tưởng chúng tơi muốn giới thiệu bạn đọc: Chúng ta điều biết p

b2+bc+c2 +

1 p

c2+ca+a2

2p2 p

a2+b2+ac+bc+2c2 ( )và việc tách bình

phương từp 2p2

a2+b2+ac+bc+2c2

2

q

(a+b

2 )

+c(a+2b)+c2 dễ dàng (chỉ cần bước trục

xong), nên ta nghĩ việc dùng bất đẳng thức( )chắc có giúp ích cho việc dồn biến ta Nhưng tiếc rằng, bất đẳng thức( )không đủ mạnh để thực nhiệm vụ này, ý tưởng ta thiết lập đánh giá chặt nhiều để sử dụng

1

p

a2+ac+c2 +

1

p

b2+bc+c2

2p2

p

a2+b2+ac+bc+2c2 k(a b)2,

tương đương

2

a2+ac+c2 +

1

b2+bc+c2

4

a2+b2+ac+bc+2c2 k(a b)2

1

p

a2+ac+c2

1

p

b2+bc+c2

Bất đẳng thức thu gọn thành

2(a b)2[(a+b+c)2 k(a2+b2+2c2+ac+bc)]

(a2+ac+c2)(b2+bc+c2)[a2+b2+ac+bc+2c2 k(a b)2]

(a b)2(a+b+c)2

(a2+ac+c2)(b2+bc+c2) pa2+ac+c2+pb2+bc+c2

,

hay

2[(a+b+c)2 k(a2+b2+2c2+ac+bc)]

(a2+ac+c2)(b2+bc+c2)[a2+b2+ac+bc+2c2 k(a b)2]

(a+b+c)2

(a2+ac+c2)(b2+bc+c2) pa2+ac+c2+pb2+bc+c2

Choa = b= c,ta tìm đượck 98.Vìkcàng lớn đánh giá ta chặt, có ích cho ta nên ta thử xét xem vớik = 98 bất đẳng thức có hay khơng, tốn chưa chặt nên ta khơng cần thiết phải đánh giá "hết ga" thế, ta thử vớik = 1xem Chok = 1vào, bất đẳng thức trở thành

2(2ab+ac+bc c2)

(a2+ac+c2)(b2+bc+c2)(2ab+ac+bc+2c2)

(174)

(a+b+c)2

(a2+ac+c2)(b2+bc+c2) pa2+ac+c2+pb2+bc+c2

,

tương đương

2(2ab+ac+bc c2)

2ab+ac+bc+2c2

(a+b+c)2

p

a2+ac+c2+pb2+bc+c2

Dễ dàng đánh giá 22(2abab++acac++bcbc c+2c22) 1và

p

a2+ac+c2+pb2+bc+c2 a+b+c

nên bất đẳng thức hiển nhiên Như vậy, ta thiết lập

1

p

b2+bc+c2 +

1

p

c2+ca+a2

2p2

p

2ab+ac+bc+2c2

Và ta đến việc chứng minh

1

p

a2+ab+b2+

2p2

p

2ab+ac+bc+2c2

1

q

3(a+b)2

4

+

p

2

q

(a+b)2

2 +ac+bc+2c2

để hồn thành bước dồn biến Một cơng việc đơn giản với bạn quen với đạo hàm Thật vậy, đặtt =ab (a+4b)2 xét hàm số

f(t) = p

(1 c)2 t +

2p2

p

2t+c+c2,

ta dễ thấy

f0(t) =

2[(1 c)2 t]3/2

2p2

(2t+c+c2)3/2 <0,

bởi ta có2[(1 c)2 t] (2t+c+c2) = 2(a2+ab+b2) (2ab+ac+bc+2c2)

và 21 (22p)3/22 < 0.Điều chứng tỏ f(t)là hàm nghịch biến, thu

f(t) f (a+4b)2 Bước dồn biến hồn tất, tức ta cóP(a,b,c) P a+2b,a+2b,c

Việc lại ta chứng minh P a+2b,a+2b,c 4+ p2

3, công việc nhẹ

nhàng với phép chọn tham số bất đẳng thứcAM – GM, xin dành lại cho bạn đọc phần

Ví dụ 2 Cho số dươnga,b,cthỏa mãna+b+c=3.Chứng minh rằng

1

5(a2+b2) +8+

1

5(b2+c2) +8 +

1 5(c2+a2) +8

1

(175)

Lời giải.Để ý bất đẳng thức đạt dấu đẳng thức a = b = c = 1và

a= 135,b=c= 15 nên ý tưởng ta dồn biến trung bình cộng (dựa giả thiết toán) Giả sửa b c, đặtk = 85 vàP(a,b,c) = a2+1b2+k + b2+1c2+k + c2+1a2+k,ta

phải chứng minhP(a,b,c) P a,b+2c,b+2c k+32.Nhưng trước, ta thấy việc tách bình phương(b c)2từ hiệu

a2+b2+k +a2+1c2+k

a2+(b+c

2 )

+k phức

tạp đưa đến bậc cao khó đánh giá Vì vậy, để chứng minh nó, ta sử dụng ý tưởng sau: Tìmmnhỏ để bất đẳng thức sau

1

a2+b2+k+

1

a2+c2+k

4

2a2+b2+c2+2k m(b c)2

Nhân hai vế bất đẳng thức với(a2+b2+k) + (a2+c2+k)và sử dụng đẳng thức quen thuộc(x+y) 1x+ y1 = (x yxy)2, ta dễ dàng viết lại bất đẳng thức sau

(b2 c2)2

(a2+b2+k)(a2+c2+k)

4m(b c)2

2a2+b2+c2+2k m(b c)2,

tương đương

(b+c)2

(a2+b2+k)(a2+c2+k)

4m

2a2+b2+c2+2k m(b c)2

Choa = b = c= 1,ta tìm đượcm k+22.Như vậy, ta thử chứng minh bất đẳng thức sau

(b+c)2

(a2+b2+k)(a2+c2+k)

8

(k+2) 2a2+b2+c2+2k

k+2(b c)2

Ta có

2a2+b2+c2+2k

k+2(b c)

2 2a2+b2+c2+2k 2(a2+b2+k),

4(a2+c2+k) (k+2)(b+c)2 4(b2+c2) +k(b+c)2 (k+2)(b+c)2 = 4(b2+c2) 2(b+c)2=2(b c)2 0,

nên bất đẳng thức hiển nhiên Do đó, ta có

1

a2+b2+k+

1

a2+c2+k

4 2a2+b2+c2+2k

k+2(b c)2

(176)

Tiếp theo, để hoàn tất bước dồn biến, ta phải chứng minh

1

b2+c2+k +

4 2a2+b2+c2+2k

k+2(b c)2

1

(b+c)2

2 +k

+

2a2+(b+c)2

2 +2k

,

tương đương

(b c)2

(b2+c2+k)[(b+c)2+2k]

2(2 k)(b c)2

(k+2) 2a2+b2+c2+2k

k+2(b c)2

h

2a2+ (b+c)2

2 +2k

i,

hay

2a2+b2+c2+2k

k+2(b c)2

h

2a2+ (b+c)2

2 +2k

i

(b2+c2+k)[(b+c)2+2k]

2(2 k)

k+2

Ta có

2a2+b2+c2+2k

k+2(b c)

2 [(b+c)2+2k] = 2a2 2bc

k+2(b c)

2

2b2 2bc (b c)2 =b2 c2 0,

2a2+ (b+c)

2

2 +2k 2(b

2+c2+k) =2(a2 b2) + (b+c)2

2 2c

2 0,

nên

VT 2= 2(2 k)

k+2 +

4k k+2

2(2 k)

k+2 =VP

Phép dồn biến hồn tất Và việc cịn lại ta chứng minhP(3 2t,t,t) k+32

vớit= b+2c vàk= 85.Bằng cách khai triển biến đổi tương đương, ta thấy bất đẳng thức tương đương với

25(5t 1)2(t 1)2

6(5t2+4)(25t2 60t+53) 0(đúng)

Bài toán chứng minh xong

Nhận xét 1 Qua lời giải này, ta thấy tập hợp tất giá trị củakđể bất đẳng thức sau

1

a2+b2+k +

1

b2+c2+k +

1

c2+a2+k

3

k+2

đúng vớia,b,c>0,a+b+c=3k 85

Ngoài ra, trường hợpk=2chính tốn thi chọn đội tuyển Iran năm 2009.

(177)

Sau số hai ví dụ đẹp khác

Ví dụ 3 Cho số dươnga,b,c.Chứng minh bất đẳng thức sau

ab

a2+b2+3c2 +

bc

b2+c2+3a2 +

ca c2+a2+3b2

3

(Phạm Kim Hùng, Vasile Cirtoaje)

Lời giải.Khơng tính tổng qt, giả sửc=minfa,b,cg.Khi đó, ta chứng minh

bc

b2+c2+3a2 +

ca c2+a2+3b2

c(a+b) + 1825(a b)2

2a2+2b2+c2

Thật vậy, bất đẳng thức tương đương với

c b(2a

2+2b2+c2) b2+c2+3a2 +

a(2a2+2b2+c2)

c2+a2+3b2 a b

18

25(a b)

2.

Ta có

b(2a2+2b2+c2)

b2+c2+3a2 +

a(2a2+2b2+c2)

c2+a2+3b2 a b =

a(a2 b2)

c2+a2+3b2 +

b(b2 a2)

b2+c2+3a2

= (a b)

2(a+b)(3a2+3b2+c2+2ab)

(3a2+b2+c2)(a2+3b2+c2) ,

nên bất đẳng thức viết lại thành

c(a b)2(a+b)(3a2+3b2+c2+2ab) (3a2+b2+c2)(a2+3b2+c2)

18

25(a b)

2,

tương đương

9c(a+b) 2(3a2+3b2+c2+2ab) 18

2

25 (3a

2+b2+c2)(a2+3b2+c2).

Áp dụng bất đẳng thứcAM – GMCauchy Schwarz, ta

9c(a+b) 2(3a2+3b2+c2+2ab) 9c(a+b) +2(3a

2+3b2+c2+2ab)

2

2

,

(3a2+b2+c2)(a2+3b2+c2) (a2+b2+2ab+c2)2

(178)

Lại có

36(a2+b2+2ab+c2) 5[9c(a+b) +2(3a2+3b2+c2+2ab)] = 6a2+6b2+26c2 45c(a+b) +52ab

= 26(c a)(c b) +6a2+6b2 19c(a+b) +26ab

6a2+6b2 19c(a+b) +26ab 38ab 19c(a+b) 0,

nên bất đẳng thức hiển nhiên Tiếp theo, ta chứng minh

ab

a2+b2+3c2 +

c(a+b) + 1825(a b)2

2a2+2b2+c2

a2+b2

2(a2+b2+3c2)+

cp2(a2+b2)

2a2+2b2+c2,

tương đương

(a b)2

2(a2+b2+3c2)+

c(a b)2

(2a2+2b2+c2)hp2(a2+b2) +a+bi

18(a b)2

25(2a2+2b2+c2)

Dop2(a2+b2) +a+b 2p2(a2+b2),nên ta cần chứng minh được

1

2(2t2+3c2)+ c

4t(4t2+c2)

18

25(4t2+c2) t =

r

a2+b2

2

!

,

tương đương

2(4t2+c2)

2t2+3c2 + c t

72 25

Ta có

2(4t2+c2)

2t2+3c2 + c

t 3=

(t c)(2t2+4tc 3c2)

t(2t2+3c2) 0,

mà7225 <3nên bất đẳng thức hiển nhiên Cuối cùng, ta đến việc chứng minh bất đẳng thức sau

t2

2t2+3c2 +

2tc

4t2+c2

3

Khơng khó khăn, ta phân tích

t2

2t2+3c2 +

2tc

4t2+c2

3 =

(t c)2(2t 3c)2

5(2t2+3c2)(4t2+c2) 0,

nên bất đẳng thức hiển nhiên phép chứng minh ta hoàn tất Đẳng thức xảy khia=b=choặca =b= 32ccùng hoán vị

(179)

Ví dụ 4 Cho số dươnga,b,cthỏa mãna+b+c=3.Chứng minh rằng

4a2 27

b2+c2 +

4b2 27

c2+a2 +

4c2 27

a2+b2 +

69

(Vasile Cirtoaje)

Lời giải.Với ý 4ba22+c272 +4 =

4(a2+b2+c2) 27

b2+c2 , ta viết lại bất đẳng thức cần

chứng minh dạng

(4a2+4b2+4c2 27)

a2+b2 +

1

b2+c2 +

1

c2+a2 +

45

Khơng tính tổng quát, giả sửa=maxfa,b,cg.Áp dụng bất đẳng thứcCauchy Schwarz, ta có4a2+4b2+4c2 27 4a2+2(b+c)2 27,nên bất đẳng thức suy từ

[4a2+2(b+c)2 27]

a2+b2 +

1

b2+c2 +

1

c2+a2 +

45

Nếu4a2+2(b+c)2 27 0thì bất đẳng thức hiển nhiên Trong trường hợp ngược lại, áp dụng bất đẳng thức mà ta thiết lập trongVí dụ 2ở trên, ta có

1

a2+b2+k +

1

b2+c2+k +

1

c2+a2+k

2

a2+(b+c)2

4 +k

+ (

b+c)2

2 +k

với mọik 0.Từ đó, đặtt = b+2cvà chok =0,ta thu

1

a2+b2 +

1

b2+c2 +

1

c2+a2

2

a2+t2 +

1 2t2

Do4a2+8t2 27 0nên

(4a2+8t2 27)

a2+b2 +

1

b2+c2 +

1

c2+a2 (4a

2+8t2 27)

a2+t2+

1 2t2

Vì thế, ta cần chứng minh

[4a2+8t2 3(a+2t)2]

a2+t2 +

1 2t2 +

45

Bằng vài tính tốn đơn giản, ta thấy bất đẳng thức tương đương với(a t2t2)(2a(2a+t52)t)2

0là bất đẳng thức hiển nhiên Phép chứng minh ta hoàn tất Dễ thấy đẳng thức xảy khia=b=c=1hoặca= 157,b= c= 37 hoán vị tương ứng

(180)

Nhận xét 2 Mời bạn đọc chứng minh kết sau Tập hợp tất giá trị củakđể bất đẳng thức

a2+k b2+c2+

b2+k c2+a2 +

c2+k

a2+b2 3(k+1)

đúng với mọia,b,cdương choa+b+c=3 274 k 97

BÀI TẬP ĐỀ NGHỊ

1 Giả sửa,b,clà số thực không âm thỏa mãna+b+c=1.Chứng minh

p

a2+bc+pb2+ca+pc2+ab

2

(Phạm Kim Hùng) Giả sửa,b,clà số thực không âm thỏa mãna+b+c=1.Chứng minh

cyc

1

p

(a2+ab+b2)(a2+ac+c2) 4+

8

p

3

(Võ Quốc Bá Cẩn, Trần Quang Hùng) Cho số thực không âma,b,cthỏa mãna+b+c= 1.Tìm sốklớn đề

bất đẳng thức sau

q

a+k(b c)2+qb+k(c a)2+qc+k(a b)2 p3.

(Phan Thành Việt) Cho số thực không âma,b,cthỏa mãna+b+c= 1.Tìm sốklớn đề

bất đẳng thức sau

1

p

a+k(b c)2 +

1

p

b+k(c a)2 +

1

p

c+k(a b)2

p

3

(Võ Quốc Bá Cẩn) Giả sửa,b,clà số thực không âm thỏa mãna+b+c=3.Chứng minh

đó

1

(b+c)2+6+

1

(c+a)2+6 +

1

(a+b)2+6

3 10

(Dương Đức Lâm) Cho số thực dươnga,b,cthỏa mãnabc=1.Chứng minh bất đẳng thức sau

p

8a2+1+p8b2+1+p8c2+1 3(a+b+c).

(Gabriel Dospinescu)

(181)

Bất đẳng thức Schur phương pháp đổi biến p,q,r

Võ Thành Văn

Nhữ cĂc bÔn  biát, bĐt ng thực Schur l mởt bĐt ng thực mÔnh v cõ nhiÃu ựng dửng, nhiản nõ văn cỏn khĂ xa lÔ vợi nhiÃu bÔn hồc sinh THCS cụng nhữ THPT Qua b i viát n y, tổi muốn cụng cĐp thảm cho cĂc bÔn mởt kắ thuêt sỷ dửng tốt BDT Schur, õ l kát hủp vợi phữỡng phĂp ời biánp; q; r

Trữợc hát, tổi xin nh-c lÔi và bĐt ng thực Schur v phữỡng phĂp ời biánp; q; r

1 Bt ng thc Schur

nh lỵ (BĐt ng thực Schur) Vợi mồi số thỹc khổng ¥ma; b; c; k;ta luæn câ

ak(a b)(a c) +bk(b c)(b a) +ck(c a)(c b) 0:

Hai tr÷íng hđp quen thc ÷đc sû dưng nhi·u l k= 1v k=

a(a b)(a c) +b(b c)(b a) +c(c a)(c b) (i)

a2(a b)(a c) +b2(b c)(b a) +c2(c a)(c b) (ii)

2 Phương pháp đổi biến p; q; r

èi vỵi mët sè b i bĐt ng thực thuƯn nhĐt ối xựng cõ cĂc bián khổng Ơm thẳ ta cõ th ời bián lÔi nhữ sau

tp=a+b+c; q=ab+bc+ca; r=abc:V ta thu ữủc mët sè ¯ng thùc sau

ab(a+b) +bc(b+c) +ca(c+a) = pq 3r

(a+b)(b+c)(c+a) = pq r

ab(a2+b2) +bc(b2+c2) +ca(c2+a2) = p2q 2q2 pr

(a+b)(a+c) + (b+c)(b+a) + (c+a)(c+b) = p2+q

a2+b2+c2 = p2 2q

a3+b3+c3 = p3 3pq+ 3r

a4+b4+c4 = p4 4p2q+ 2q2+ 4pr a2b2+b2c2+c2a2 = q2 2pr

a3b3+b3c3+c3a3 = q3 3pqr+ 3r2

a4b4+b4c4+c4a4 = q4 4pq2r+ 2p2r2+ 4qr2

°tL=p2q2+ 18pqr 27r2 4q3 4p3r;khi â

a2b+b2c+c2a = pq 3r

p

L

(182)

3 CÁC VÍ DỤ MINH HỌA

Câ thº th§y lđi ½ch cõa ph÷ìng ph¡p n y l mèi r ng buëc giúa c¡c bi¸np; q; rm c¡c bi¸na; b; cban ¦u khỉng câ nh÷

p2 3q

p3 27r

q2 3pr

pq 9r

2p3+ 9r 7pq

p2q+ 3pr 4q2

p4+ 4q2+ 6pr 5p2q

Nhỳng kát quÊ trản Ơy ch-c ch-n l chữa ừ, cĂc bÔn cõ th phĂt trin thảm nhiÃu ng thực, bĐt ng thực liản hằ giỳa biánp; q; r V i·u quan trång m tỉi mn nâi ¸n l tø b§t ¯ng thùc (i) v (ii), ta câ

r p(4q p

2)

9 (tø (i))

r (4q p

2)(p2 q)

6p (tø (ii))

Tuy nhiản mởt số trữớng hủp thẳ cõ th cĂc Ôi lữủng4q p2cõ th nhên giĂ tr Ơm lăn giĂ tr dữỡng

nản ta thữớng sỷ dửng

r max 0;p(4q p

2)

4

r max 0;(4q p

2)(p2 q)

6p

Cõ l án Ơy cĂc bÔn  hiu ữủc phƯn n o và bĐt ng thực Schur v phữỡng phĂp ời biánp; q; r Sau Ơy l mởt số vẵ dử minh hồa, trữợc hát, cĂc bÔn hÂy têp l m thỷ rỗi xem Ăp Ăn sau

3 Các ví dụ minh họa

3.1 Bất đẳng thức Schur

Vẵ dử 1 Cho cĂc số dữỡnga; b; c:Chùng minh r¬ng s

(a+b)3

8ab(4a+ 4b+c)+

s

(b+c)3

8bc(4b+ 4c+a)+

s

(c+a)3

8ca(4c+ 4a+b) 1:

(Vã Th nh V«n) LÍI GIƒI °t

P =

s

(a+b)3

8ab(4a+ 4b+c)+

s

(b+c)3

8bc(4b+ 4c+a)+

s

(c+a)3

8ca(4c+ 4a+b)

Q = 8ab(4a+ 4b+c) + 8bc(4b+ 4c+a) + 8ca(4c+ 4a+b) = 32(a+b+c)(ab+bc+ca) 72abc

ãp dửng bĐt ng thực Holder, ta cõ

(183)

3.1 Bất đẳng thức Schur CÁC VÍ DỤ MINH HỌA

Ta c¦n chùng minh

8(a+b+c)3 Q

,8(a+b+c)3 32(a+b+c)(ab+bc+ca) 72abc

,(a+b+c)3 4(a+b+c)(ab+bc+ca) 9abc( úng theo bĐt ng thực Schur) Vêy ta cõ pcm

Vẵ dử 2 Cho cĂc số dữỡnga; b; c:Chựng minh r¬ng

(a2+ 2)(b2+ 2)(c2+ 2) 9(ab+bc+ca):

(APMO 2004) LI GII.Khai trin bĐt ng thực trản, ta cƯn chùng minh

a2b2c2+ 2(a2b2+b2c2+c2a2) + 4(a2+b2+c2) + 9(ab+bc+ca)

Ta câ

a2+b2+c2 ab+bc+ca

(a2b2+ 1) + (b2c2+ 1) + (c2a2+ 1) 2(ab+bc+ca)

a2b2c2+ + 3p3a2b2c2 9abc

a+b+c

4(ab+bc+ca) (a+b+c)2(theo b§t ¯ng thực Schur) ãp dửng cĂc bĐt ng thực trản, ta câ

(a2b2c2+ 2) + 2(a2b2+b2c2+c2a2+ 3) + 4(a2+b2+c2) 2(ab+bc+ca) + 4(ab+bc+ca) + 3(a2+b2+c2) 9(ab+bc+ca):

BĐt ng thực ữủc chùng minh ¯ng thùc x£y v ch¿ khia=b=c= 1:

Vẵ dử 3 Cho cĂc số dữỡnga; b; c:Chựng minh rơng

2(a2+b2+c2) +abc+ 5(a+b+c):

(TrƯn Nam Dơng) LÍI GIƒI.Sû dưng b§t ¯ng thùc AM-GM, ta câ

6V T = 12(a2+b2+c2) + 3(2abc+ 1) + 45 3(a+b+c) 12(a2+b2+c2) + 9p3a2b2c2+ 45 5 (a+b+c)2+ 9

= 7(a2+b2+c2) + p93abc

abc 10(ab+bc+ca)

7(a2+b2+c2) + 27abc

a+b+c 10(ab+bc+ca)

M°t kh¡c, sû dưng b§t ¯ng thùc Schur,

9

a+b+c 4(ab+bc+ca) (a+b+c)

(184)

3.1 Bất đẳng thức Schur CÁC VÍ DỤ MINH HỌA

Do â

7(a2+b2+c2) + 27

a+b+c 10(ab+bc+ca)

7(a2+b2+c2) + 6(ab+bc+ca) 3(a2+b2+c2) 10(ab+bc+ca) = 4(a2+b2+c2 ab bc ca) 0:

B§t ¯ng thùc ÷đc chùng minh ¯ng thùc x£y v ch¿ khia=b=c= 1:

V½ dư 4 Cho c¡c sè khỉng ¥ma; b; c;khỉng câ sè n o çng thíi b¬ng0:Chùng minh r¬ng

a b3+c3 +

b a3+c3 +

c a3+b3

18

5(a2+b2+c2) ab bc ca:

(Michael Rozenberg) LÍI GIƒI.B§t ¯ng thùc cƯn chựng minh tữỡng ữỡng vợi

X

cyc

a(a+b+c)

b3+c3

18(a+b+c)

5(a2+b2+c2) ab bc ca

,X

cyc

a2

b3+c3 +

X

cyc

a

b2+c2 bc

18(a+b+c)

5(a2+b2+c2) ab bc ca

ãp dửng bĐt ¯ng thùc Cauchy-Schwarz, ta câ

X

cyc

a2 b3+c3

(a2+b2+c2)2

P

cyc

a2(b3+c3)

X

cyc

a

b2+c2 bc

(a+b+c)2

P

cyc

a(b2+c2 bc)

Ta c¦n chùng minh

(a2+b2+c2)2

P

cyc

a2(b3+c3) +

(a+b+c)2

P

cyc

a(b2+c2 bc)

18(a+b+c)

5(a2+b2+c2) ab bc ca

Gi£ sûa+b+c= 1v °tab+bc+ca=q; abc=r)r maxn0;(4q 1)(16 q)o Ta c¦n chùng minh

(1 2q)2

q2 (q+ 2)r+

1

q 6r

18 11q

B§t ¯ng thực cuối d ng chựng minh bơng cĂch xt tr÷íng hđp1 4qv 4q ¯ng thùc x£y khia=b=choca=b; c= 0hoc cĂc hoĂn v tữỡng ựng

Vẵ dử 5 Cho cĂc số dữỡnga; b; cthọa mÂna4+b4+c4= 3 Chùng minh r¬ng

1 ab+

1 bc+

1 ca 1:

(185)

3.1 Bất đẳng thức Schur CÁC VÍ DỤ MINH HA

LI GII.Quy ỗng mău số rỗi khai trin, ta c¦n chùng minh

49 8(ab+bc+ca) + (a+b+c)abc 64 16(ab+bc+ca) + 4(a+b+c)abc a2b2c2

,16 + 3(a+b+c)abc a2b2c2+ 8(ab+bc+ca)

ãp dửng bĐt ng thực Schur v giÊ thiáta4+b4+c4= 3, ta câ

(a3+b3+c3+ 3abc)(a+b+c) [ab(a+b) +bc(b+c) +ca(c+a)] (a+b+c)

,3 + 3abc(a+b+c) (ab+bc)2+ (bc+ca)2+ (ca+ab)2

•p dưng b§t ¯ng thùc AM-GM, ta câ

(ab+bc)2+ (bc+ca)2+ (ca+ab)2+ 12 8(ab+bc+ca)

)15 + 3abc(a+b+c) 8(ab+bc+ca)

M°t kh¡c ta lÔi cõ

1 a2b2c2:

Vêy ta cõ pcm ¯ng thùc x£y v ch¿ khia=b=c= 1:

Vẵ dử 6 Cho cĂc số khổng Ơma; b; cthọa mÂnab+bc+ca= 3:Chựng minh rơng

a3+b3+c3+ 7abc 10:

(Vasile Cirtoaje)

ãp dửng bĐt ng thực Schur, ta câ

r max 0;p(4q p

2)

9 = max 0;

p(12 p2)

9

Ta cƯn chựng minh

p3 9p+ 10r 10

Náup 2p3th¼ ta câ

p3 9p+ 10r 10 p3 9p 10 12p 9p 10 = 3p 10>0

Náup 2p3<4thẳ

p3 9p+ 10r 10 p3 9p+10

9 p(12 p

2) 10 =

9(p 3)[(16 p

2) + 3(4 p) + 2] 0:

Vªy ta câ pcm ¯ng thùc x£y v ch¿ khia=b=c=

Vẵ dử 7 Cho cĂc số dữỡnga; b; cthọa mÂna+b+c= 3:Chựng minh rơng

3 + 12

abc

1

a+

1

b +

1

c :

(186)

3.1 Bất đẳng thức Schur CÁC VÍ DỤ MINH HỌA

LÍI GIƒI êi bi¸n theop; q; r, bƠt ng thực cƯn chựng minh ữủc viát lÔi nhữ sau

3r+ 12 5q

M°t kh¡c,theo b§t ¯ng thùc Schur, ta câ

3r 3p(4q p

2)

9 = 4q

Ta c¦n chùng minh

4q + 12 5q

,q 3( óng) Vªy ta câ pcm ¯ng thùc x£y v ch¿ khia=b=c= 1:

V½ dư 8 Choa; b; cl cĂc số thỹc dữỡng thọa mÂna2+b2+c2= 3 Chựng minh rơng

1 a+

1 b+

1 c 3:

(PhÔm Kim Hũng) Quy ỗng, rút gồn v ời bián theop; q; r, bĐt ng thực cƯn chựng minh tữỡng ữỡng vợi

8p+ 3r 12 + 5q

ãp dửng bĐt ng thực Schur, ta câ

3r p(4q p

2)

3 =

p(2q 3)

3

Tø gi£ thi¸t

p2 2q=

)q=p

2 3

2

Thay iÃu trản v o bĐt ng thực c¦n chùng minh, ta câ

8p+p(p

2 6)

3 12 +

5(p2 3)

,(2p 3)(p 3)2

BĐt ng thực cuối úng nản ta câ pcm ¯ng thùc x£y v ch¿ khia=b=c= 1:

V½ dư 9 Cho c¡c sè khỉng Ơma; b; cthọa mÂna+b+c= 3:Chựng minh rơng

1 ab+

1 bc+

1 ca

3 8:

(Crux mathematicorum) LÍI GIƒI.B i n y  ữủc anh Hũng sỷ dửng cho phƯn bĐt ng thực Chebyshev "SĂng tÔo bĐt ng thực" BƠy giớ cĂc bÔn s ữủc thĐy mởt lới giÊi khĂc vợi bĐt ng thực Schur v phữỡng phĂp ời bián

p; q; rrĐt tỹ nhiản

Bián ời bĐt ng thực cƯn chựng minh v chuyn và dÔngp; q; r, ta câ

(187)

3.2 Phương pháp đổi biếnp; q; r CÁC VÍ DỤ MINH HỌA

,243 99q+ 57r 3r2

Theo b§t ¯ng thùc AM-GM th¼

3 = a+b+c

6

3(abc)2=r2

Theo b§t ¯ng thùc Schur, ta câ

r p(4q p

2)

3 = 4q

3

)57r 19(4q 9)

N¶n ta c¦n chùng minh

72 23q 3r2

,3(1 r2) + 23(3 q) 0( úng)

Vêy bĐt ¯ng thùc ÷đc chùng minh ¯ng thùc x£y v chi khia=b=c= 1:

3.2 Phương pháp đổi biến p; q; r

V½ dư 10 Cho c¡c sè khổng Ơma; b; cthọa mÂna+b+c= 3:Chựng minh rơng

a2b

4 bc+ b2c

4 ca + c2a

4 ab 1:

(PhÔm Kim Hũng) LI GII.Quy ỗng mău số rỗi khai trin, ta cƯn chựng minh

4 X

cyc

a2b X

cyc

a2b2c

4 bc

Sû dưng b§t ¯ng thùc quen thuëc4 P

cyc

a2b abc, ta c¦n chùng minh

abc X

cyc

a2b2c

4 bc

,1 X

cyc

ab

4 bc

,64 32X

cyc

ab+ 8X

cyc

a2bc+ 4X

cyc

a2b2 abc X

cyc

a2b+abc

!

Ti¸p tưc sỷ dửng bĐt ng thực trản,ta cƯn chựng minh

64 32X

cyc

ab+ 8X

cyc

a2bc+ 4X

cyc

a2b2 4abc

,16 8q+q2 r

vợiq=ab+bc+ca; r=abc

ãp dửng bĐt ng thực AM-GM, ta cõq2 9rnản cƯn chựng minh

16 8q+q2 q

2

9

,(q 3)(q 6) 0:

BĐt ng thực cuối hin nhiản úng n¶n ta câ pcm

(188)

3.2 Phương pháp đổi biếnp; q; r CÁC VÍ DỤ MINH HA

Vẵ dử 11 Cho cĂc số dữỡnga; b; c:Chùng minh r¬ng

1

a+

1

b +

1

c

3a a2+ 2bc+

3b b2+ 2ca +

3c c2+ 2ab:

(Dữỡng ực LƠm)

ta:= 1a; b:= 1b; c:= 1c;bĐt ng thực cƯn chựng minh tữỡng ữỡng vợi

X

cyc

a 3abcX

cyc

1 2a2+bc

,X

cyc

a(a2 bc)

2a2+bc

,3X

cyc

a3

2a2+bc

X

cyc

a

ãp dửng bĐt ng thực Cauchy-Schwarz, ta câ

X

cyc

a3

2a2+bc

P

cyc

a2

!2

2P

cyc

a3+ 3abc

án Ơy, ta c¦n chùng minh

3 X

cyc

a2

!2

X

cyc

a

!

2X

cyc

a3+ 3abc

!

GiÊ sỷa+b+c= 1;chuyn và dÔngp; q; r, bĐt ng thực trð th nh

3(1 2q)2 6q+ 9r

Sỷ dửng bĐt ng thựcq2 3r;ta cƯn chựng minh

3(1 2q)2 6q+ 3q2

,3 12q+ 12q2 6q+ 3q2

,(1 3q)2 0( óng):

Vªy ta câ pcm ¯ng thùc x£y v ch¿ khia=b=c:

Vẵ dử 12 Cho cĂc số khổng Ơma; b; c:Chùng minh r¬ng

a4(b+c) +b4(c+a) +c4(a+b)

12(a+b+c)

5:

(Vasile Cirtoaje) LÍI GIƒI.Chu©n hâa chop= 1, b§t ¯ng thùc trð th nh

(1 3q)q+ (5q 1)r

12

(189)

3.2 Phương pháp đổi biếnp; q; r CÁC VÍ DỤ MINH HA

Náuq

5thẳ ta cõ

(1 3q)q+ (5q 1)r (1 3q)q=

3(1 3q) 3q

1 3q+ 3q

2

2

= 12

N¸uq > 5;ta câ

(1 3q)q+ (5q 1)r (1 3q)q+ (5q 1) q =

1 36( 88q

2+ 32q 3) +

12 < 12:

Vêy bĐt ¯ng thùc ÷đc chùng minh

¯ng thùc x£y khia= 0; b=3+6p3; c= 6p3v c¡c ho¡n

Vợi kắ thuêt xt trữớng hủp giÊi, cõ th d ng giÊi quyát cĂc b i to¡n sau

B i to¡n 1 Cho c¡c sè khổng Ơma; b; cthọa mÂna+b+c= 1:Chựng minh rơng

(a2+b2)(b2+c2)(c2+a2) 32:

HìẻNG DN NhƠn v o rỗi rút gồn, chuyn bĐt ng thực và dÔngp; q; r, ta c¦n chùng minh

q2 2q3 r(2 +r 4q)

32

án Ơy xt trữớng hñpq 14 v q > 14:

B i to¡n 2 Cho cĂc số dữỡnga; b; cthọa mÂnabc= 1:Chựng minh r¬ng

a

a2+ 3 +

b

b2+ 3 +

c c2+ 3

3 4:

(Dữỡng ực LƠm) HìẻNG DN ữa bĐt ng thực v· mët h m theop

f(p) = 27p2 (54 + 12q)p+ 9q2 58q+ 120

án Ơy chia th nh tr÷íng hđp18q 58 + 12pv 18q 58 + 12p

V½ dư 13 Cho c¡c số khổng Ơma; b; cthọa mÂna2+b2+c2= 8 Chựng minh rơng

4(a+b+c 4) abc:

(Nguy¹n Phi Hịng) LÍI GIƒI.Theo giÊ thiát, ta cõp2 2q= 8:Mt khĂc, theo bĐt ng thùc Schur bªc 4, ta câ

r (4q p

2)(p2 q)

6p =

(p2 16)(p2+ 8)

12p

Vẳ vêy, ta cƯn chựng minh

(p2 16)(p2+ 8)

12p 4(p 4)

, (p 4)

2(p2+p 8)

12p 0( óng):

(190)

3.2 Phương pháp đổi biếnp; q; r CÁC VÍ DỤ MINH HỌA

V½ dư 14 Cho cĂc số dữỡnga; b; cthọa mÂna+b+c= 1:Chựng minh rơng

p

a2+abc

b+ca +

p

b2+abc

c+ab +

p

c2+abc

a+bc

1 2pabc:

LÍI GIƒI êi bi¸n th nhp; q; r, ta câ bê ·

r q

2(1 q)

2(2 3q)

•p dưng BDT Cauchy-Schwarz, ta câ

" X

cyc

p

a2+abc

(b+c)(b+a)

#2 "

X

cyc

a

(a+b)(b+c)

# X

cyc

a+c

b+c

!

=

P

cyc

a2+P

cyc

ab

(a+b)(b+c)(c+a)

X

cyc

a+c

b+c

!

Ta câ

X

cyc

a+c

b+c =

X

cyc

1

b+c

X

cyc

b b+c

X

cyc

1

b+c

(a+b+c)2

P

cyc

a2+P

cyc

ab

Nản ta cƯn chựng minh

P

cyc

a2+P

cyc

ab

(a+b)(b+c)(c+a)

2 4X

cyc

1

b+c

1

P

cyc

a2+P

cyc

ab

3 4abc1

, q

q r

1 +q

q r

1 q

1 4r

, 4(1 q

2)

q r

q r

r

, 4(1 q

2)

q r

q

r

Sû döng bê ·, ta câ

V T 4(1 q

2)

q q2(2 32(1 qq)) q

q2(1 q)

2(2 3q)

= q(1 3q)(5 7q) (1 q)(4 7q+q2) 3:

Vªy ta câ pcm ¯ng thùc x£y v ch¿ khia=b=c= 13:

Nhên xt 1 Vợi b i toĂn n y, chúng tỉi câ c¥u häi thó xin d nh cho cĂc bÔn 1 Chựng minh bờ à m chúng tổi  nảu trản.

(191)

3.2 Phương pháp đổi biếnp; q; r CÁC VÍ DỤ MINH HỌA

V½ dư 15 Cho c¡c sè thüc dữỡnga; b; cthọa mÂna+b+c= 1 Chựng minh rơng

4

81(ab+bc+ca)+abc 27:

(Vã Th nh V«n) LÍI GII.ãp dửng bĐt ng thực Schur, ta cõ

r p(4q p

2)

9 = 4q

9

BĐt ng thực cƯn chựng minh tữỡng ữỡng vỵi

4 81q+r

5 27

Sû dưng bĐt ng thực Schur, ta cƯn chựng minh

4 81q+

4q

5 27

, 814q+4q

8 27

BĐt ng thực trản hin nhiản úng theo bĐt ng thực AM-GM nản ta câ pcm ¯ng thùc x£y v ch¿ khia=b=c=13:

Vẵ dử 16 Cho cĂc số khổng Ơma; b; cthọa mÂnab+bc+ca= 1:Chựng minh rơng

ab+

a+b +

bc+

b+c +

ca+

c+a 3:

(Nguyạn MÔnh Dụng) LI GII.Ta cõ

ab+

a+b +

bc+

b+c +

ca+

c+a

,X

cyc

(ab+ 1)(c+a)(c+b) 3(a+b)(b+c)(c+a)

,X

cyc

(ab+ 1)(c2+ 1) 3[(a+b+c)(ab+bc+ca) abc]

,(a2+b2+c2) +ab+bc+ca+abc(a+b+c) + + 3abc 3(a+b+c)

,(a+b+c)2+abc(a+b+c+ 3) + 3(a+b+c)

tp=a+b+c; q=ab+bc+ca= 1; r=abc:BĐt ng thực cƯn chựng minh trð th nh

p2+r(p+ 3) 3p+

,(p 1)(p 2) +r(p+ 3)

Náup 2thẳ bĐt ng thực hin nhiản úng Náu2 p p3;Ăp dửng bĐt ¯ng thùc Schur, ta câ

(192)

3.2 Phương pháp đổi biếnp; q; r CÁC VÍ DỤ MINH HỌA

,r 4p p

3

9

Ta c¦n chùng minh

p2 3p+ + (p+ 3) 4p p

3

9

,p4+ 3p3 13p2+ 15p 18

,(p 2)(p3+ 5p2 3p+ 9)

BĐt ng thực cuối hin nhiản óng v¼p 2v

p3+ 5p2 3p+ =p3+ 4p2+ p

2

2

+27 >0

Ta câ pcm ¯ng thùc x£y v ch¿ khia=b= 1; c= 0ho°c c¡c ho¡n

V½ dử 17 Cho cĂc số dữỡnga; b; cthọa mÂnabc= 1:Chựng minh r¬ng

1

a2 +

1

b2 +

1

c2 + 2(a+b+c):

(Vietnam MO 2006, B) LÍI GIƒI °tx=

a; y =

1

b; z=

1

c, ta cõxyz = 1, ỗng thới ời bián th nhp; q; r, ta câ b§t ¯ng thùc trð

th nh

p2 2q+ 2q

,4q p2

M bĐt ng thực trản úng theo bĐt ¯ng thùc Schur n¶n ta câ pcm ¯ng thùc x£y v ch¿

a=b=c= 1:

V½ dư 18 Cho c¡c sè khỉng ¥ma; b; c;khỉng câ số n o ỗng thới bơng0:Chựng minh rơng vợi måik 1;

ta luæn câ

a

b+c +

b

c+a+

c

a+b +k

(a+b+c)(ab+bc+ca)

a3+b3+c3

p

k+ 1:

(PhÔm Sinh TƠn) LI GII ời bián bĐt ng thực theop; q; rv chuân hõa chop= Ta cƯn chùng minh b§t ¯ng thùc

1 2q+ 3r

q r +k

q

1 3q+ 3r

p

k+

Ta câ

1 2q+ 3r

q r +k

q

1 3q+ 3r =

1 3q+ 3r

q r +k

q

1 3q+ 3r +

1 3q+ 3r

q +k

q

1 3q+ 3r +

p

k+ 1:

¯ng thùc x£y khi(a; b; c) =

p

k+2pk 3+pk+1

2 x; x;0 ho°c c¡c ho¡n t÷ìng ùng

(193)

3.2 Phương pháp đổi biếnp; q; r CÁC VÍ DỤ MINH HỌA

B i to¡n 3 Cho c¡c sè khæng Ơma; b; c:Chựng minh rơng vợi mồik 1;ta luổn cõ

a

b+c +

b

c+a+

c

a+b+k

(a+b)(b+c)(c+a)

a3+b3+c3

p

k+ 1:

(PhÔm Sinh TƠn) B i toĂn 4 Cho c¡c sè khỉng ¥ma; b; c;khỉng câ số n o ỗng thới bơng0:Chựng minh rơng

a

b+c +

b

c+a+

c

a+b +

9(ab+bc+ca)

a2+b2+c2 6:

(PhÔm Sinh TƠn) Vẵ dử 19 Cho cĂc số khổng Ơma; b; c;khổng cõ số n o ỗng thới bơng0:Chựng minh r¬ng

a b+c

2

+ b

c+a

2

+ c

a+b

2

+ 10abc

(a+b)(b+c)(c+a) 2:

(Dữỡng ực LƠm) LÍI GIƒI °tx= b2+ac; y=c2+ba; z=a2+cb, ta câ

xy+yz+zx+xyz=

B§t ¯ng thùc trð th nh

x2+y2+z2+ 5xyz

ữa bĐt ng thực và dÔngp; q; r, tứ giÊ thiát, ta cõq+r= 4v bĐt ng thực trð th nh

p2 2q+ 5r

,p2 7q+ 12

Náu4 p, sỷ dửngbĐt ng thực Schur, ta câ

r p(4q p

2)

9

)4 q+p(4q p

2)

9

,q p

3+ 36

4p+

)p2 7q+ 12 p2 7(p

3+ 36)

4p+ + 12

Nản ta ch cƯn chựng minh ữủc

p2 7(p

3+ 36)

4p+ + 12

,(p 3)(p2 16)

i·u n y úng vẳ4 p p3q 3:

Náup 4, ta câp2 16 4qn¶n

p2 2q+ 5r p2 2q p

2

2

(194)

3.2 Phương pháp đổi biếnp; q; r CÁC VÍ DỤ MINH HỌA

V½ dư 20 Cho c¡c sè khỉng Ơma; b; cthọa mÂna+b+c= 3:Chựng minh rơng

1 ab+

1 bc+

1 ca

3 5:

(Vasile Cirtoaje) LÍI GIƒI.Chuyºn êi b§t ¯ng thùc v· nh÷ sau

108 48q+ 13pr 3r2

,4(9 4q+ 3r) +r(1 r)

Ta th§y bĐt ng thực trản úng

r=abc a+b+c

3

3

=

v theo b§t ¯ng thùc Schur th¼

3r 3p(4q p

2)

9 = 4q

)3r+ 4q 0:

Vêy bĐt ng thực ữủc chựng minh

ng thùc x£y v ch¿ khia=b=c= 1ho°ca= 0; b=c= 32 hoc cĂc hoĂn v tữỡng ựng

Vẵ dử 21 Cho c¡c sè khỉng ¥ma; b; c;khỉng câ số n o ỗng thới bơng0:Chựng minh rơng

a2(b+c)

b2+c2 +

b2(c+a)

c2+a2 +

c2(a+b)

a2+b2 a+b+c:

(Darij Grinberg) LI GII.ãp dửng bĐt ¯ng thùc Cauchy-Schwarz, ta c¦n chùng minh

" X

cyc

a2(b+c)2

#2

X

cyc

a

! " X

cyc

a2(b+c)(b2+c2)

#

êi bi¸n theop; q; r, â bĐt ng thực viát th nh

r(2p3+ 9r 7pq)

•p dưng BDT Schur, ta câp3+ 9r 4pqv b§t ¯ng thùc quen thuëcp2 3q 0, ta câ pcm. ¯ng thùc

x£y v ch¿ khia=b=cho°ca=b; c= 0:

Vẵ dử 22 Cho cĂc số khổng Ơma; b; cthọa mÂna+b+c= 1:Chựng minh rơng

5(a2+b2+c2) 6(a3+b3+c3) + 1:

LI GII ời bián vÃp; q; r;ta cƯn chùng minh

5 10q 6(1 3q+ 3r) +

,18r 8q+

(195)

3.2 Phương pháp đổi biếnp; q; r CÁC VÍ DỤ MINH HỌA

V½ dư 23 Cho c¡c sè khỉng Ơmx; y; z;khổng cõ số n o ỗng thới b¬ng0 Chùng minh r¬ng

(xy+yz+zx) (x+y)2 +

1 (y+z)2 +

1 (z+x)2

9 4:

(Iran MO 1996, Ji Chen) LÍI GIƒI.Sû dưng ph÷ìng ph¡p ời biánp; q; r, ta chuyn bĐt ng thực và dÔng nhữ sau

q (p

2+q)2 4p(pq r)

(pq r)2

9

Bián ời tữỡng ÷ìng, rót gån, ta c¦n chùng minh

4p4q 17p2q2+ 4q3+ 34pqr 9r2

,pq(p3 4pqr+ 9r) +q(p4 5p2q+ 4q2+ 6pr) +r(pq 9r)

B§t ¯ng thùc cuèi óng n¶n ta câ pcm ¯ng thùc x£y v ch¿ khix=y =zho°cx=y; z = 0ho°c c¡c ho¡n tữỡng ựng

Qua cĂc vẵ dử trản, cõ l cĂc bÔn cụng  ữủc hẳnh dung ẵt nhiÃu và b§t ¯ng thùc Schur v nhúng ùng dưng cõa nâ phữỡng phĂp ời biánp; q; r: kát thúc b i viát n y, cĂc bÔn giÊi mët sè b i tªp sau

B i to¡n 5 Cho cĂc số khổng Ơma; b; cthọa mÂna3+b3+c3= 3 Chùng minh r¬ng

a4b4+b4c4+c4a4 3:

(Vasile Cirtoaje) B i to¡n 6 Cho c¡c sè khỉng ¥ma; b; c:Chùng minh r¬ng

a2+b2+c2+ 2abc+ 2(ab+bc+ca):

(Darij Grinberg) B i to¡n 7 Cho c¡c sè khỉng ¥ma; b; cthọa mÂna2+b2+c2= 3 Chựng minh rơng

12 + 9abc 7(ab+bc+ca):

(Vasile Cirtoaje) B i to¡n 8 Cho c¡c số dữỡnga; b; cthọa mÂnabc= 1:Chựng minh rơng

1

a2 a+ 1+

1

b2 b+ 1 +

1

c2 c+ 1 3:

(Vô ẳnh Quỵ) B i toĂn 9 Cho cĂc số thỹca; b; cthọa mÂna2+b2+c2= 9 Chựng minh rơng

2(a+b+c) abc 10:

(Vietnam MO 2002, Tr¦n Nam Dơng) B i toĂn 10 Cho cĂc số dữỡnga; b; cthọa mÂnabc= 1:Chựng minh r¬ng

1 +

a+b+c

6

ab+bc+ca:

(196)

3.2 Phương pháp đổi biếnp; q; r CÁC VÍ DỤ MINH HỌA

B i to¡n 11 Cho c¡c sè d÷ìnga; b; cthäa mÂnabc= 1:Chựng minh rơng

2(a2+b2+c2) + 12 3(a+b+c) + 3(ab+bc+ca)

(Balkan MO) B i to¡n 12 Cho c¡c sè khỉng ¥m a; b; c;khỉng câ sè n o ỗng thới bơng0:Chựng minh rơng vợi mồi

k 3;ta

1

a+b +

1

b+c +

1

c+a+

k

a+b+c

2pk+

p

ab+bc+ca:

(PhÔm Kim Hũng) B i to¡n 13 Cho c¡c sè khỉng ¥ma; b; cthọa mÂnab+bc+ca+ 6abc= 9 Chựng minh rơng

a+b+c+ 3abc 6:

(Lả Trung Kiản, Vó Quốc BĂ Cân) B i to¡n 14 Cho c¡c sè khỉng ¥mx; y; z;khỉng cõ số n o ỗng thới bơng0:Tẳm hơng sốanhọ nh§t º b§t ¯ng thùc sau óng

x+y+z

3

a

xy+yz+zx

3

3 a

2 (x+y)(y+z)(z+x)

8 :

(Ivan Borsenco, Irurie Boreico) B i to¡n 15 Cho c¡c sè d÷ìnga; b; cthọa mÂnabc= 1:Chựng minh rơng

a+b+c

3

10

r

a3+b3+c3

3 :

B i to¡n 16 Cho c¡c sè khỉng ¥ma; b; cthäa mÂna+b+c= 1:Chựng minh rơng

1

a+b+

1

b+c+

1

c+a+ 2abc

247 54 :

B i to¡n 17 Choa; b; c2[1;2]:Chùng minh r¬ng

a2(b+c) +b2(c+a) +c2(a+b) 7abc:

B i to¡n 18 Cho cĂc số khổng Ơma; b; cthọa mÂna+b+c= 3:Chựng minh r¬ng

5 ab

1 +c +

5 bc

1 +a +

5 ca

1 +b ab+bc+ca:

(Vasile Cirtoaje)

(197)

Dồn biến cổ điển bất đẳng thức Jack Garfunkel

Võ Quốc Bá Cẩn

Đại học Y Dược Cần Thơ Ngày tháng năm 2008

Tóm tắt nội dung

Trong này, giới thiệu cách chứng minh phép dồn biến cổ điển cho bất đẳng thức sau

a

p

a+b+ b

p

b+c+ c

p

c+a

5

p

a+b+c

Bất đẳng thức tác giả Jack Garfunkel đề nghị tạp chí Crux Magazine năm 1991 (bài tốn 1490) Đây tốn hay khó nhận nhiều lời giải cho lời giải phép dồn biến túy đến chưa nhận

Trước hết cần có kết sau làm bổ đề phụ trợ cho chứng minh bất đẳng thức Jack Garfunkel

Bài tốn Cho số khơng âma; b; c;tất không đồng thời bằng0:Chứng minh

a

4a+ 4b+c + b

4b+ 4c+a+ c

4c+ 4a+b

1 3:

(Phạm Kim Hùng)

Lời giải.Chuẩn hóa choa+b+c= 3;khi bất đẳng thức trở thành

a

3 c+ b

3 a+ c

3 b

,a(3 a)(3 b) +b(3 b)(3 c) +c(3 c)(3 a) (3 a)(3 b)(3 c)

,a2b+b2c+c2a+abc

Khơng tính tổng qt, giả sửb số hạng nằm giữaavàc;thế ta có

(198)

)b2c+c2a abc+bc2

)a2b+b2c+c2a+abc b(a+c)2=

2 2b (a+c) (a+c)

2

2b+a+c+a+c

27

3

= 4:

Bất đẳng thức chứng minh xong Đẳng thức xảy a= b =c

hoặc(a; b; c) (2;1;0):

Nhận xét Đây bổ đề chặt dùng để giải nhiều tốn khác, bạn ghi nhớ nhé! Ngồi ra, làm mạnh bổ đề sau

a2b+b2c+c2a+abc+1

2abc(3 ab bc ca)

(Võ Quốc Bá Cẩn)

Bây đến giải tốn

Bài tốn Cho số khơng âma; b; c, khơng có số đồng thời bằng0:Chứng minh

a

p

a+b+

b

p

b+c +

c

p

c+a

5

p

a+b+c:

(Jack Garfunkel)

Lời giải.Ta xét trường hợp

Trường hợp 1.c b a;khi sử dụng bất đẳng thức Cauchy Schwarz, ta có

a

p

a+b +

b

p

b+c+

c

p

c+a

2

(a+b+c) a

a+b +

b

b+c +

c

c+a

Lại có doc b anên

a

a+b +

b

b+c +

c

c+a =

3 +

1

a b

a+b +

b c

b+c +

c a

c+a

=

(c a)(c b)(b a) (a+b)(b+c)(c+a)

3 <

25 16

Nên hiển nhiên

a

p

a+b+

b

p

b+c +

c

p

c+a

5

p

a+b+c

(199)

Trường hợp 2.1 115b a;khi sử dụng bất đẳng thức Cauchy Schwarz, ta có

X

cyc

a

p

a+b

!2 "

X

cyc

a(4a+ 4b+c)

a+b

# X

cyc

a

4a+ 4b+c

!

=

"

3X

cyc

a+X

cyc

a(a+b+c)

a+b

# X

cyc

a

4a+ 4b+c

!

= X

cyc

a

!

3 +X

cyc

a

a+b

! X

cyc

a

4a+ 4b+c

!

Theo kết tốn trước, ta có

X

cyc

a

4a+ 4b+c

1

Nên ta cần chứng minh

X

cyc

a

a+b

27 16

,(11a2+ 6ab 5b2)c+ (ab+c2)(11b 5a) (đúng)

Trường hợp 2.2 a 115b;đặtf(a; b; c) = pa

a+b+ b

p

b+c + c

p

c+a:Vì tốn có

đẳng thức xảy tạia= 3; b= 1; c= 0nên ý tưởng dồn biến

0, tức chứng minh

f(a; b; c) f(a1; b1;0) vớia1+b1=a+b+c:

Việc làm nói đơn giản thực hiện, bạn thấy khó biểu thức khó cho ta để đánh giá chúng, "cố chấp" giá trịa1; b1 hồi dồn biến khó mà ta phải linh động hơn, tùy theo trường hợp cụ thể mà chọna1; b1thích hợp ứng với trường hợp Chúng ta xét trường hợp nhỏ sau

Trường hợp 2.2.1.a 3b;khi ta chứng minh

a

p

a+b

a+c

2

p

a+b+c

,a2(a+b+c) a2+ac+c

2

4 (a+b)

,

4c

(200)

b

p

b+c +

c

p

c+a

r

b+c

2

Doa 3b nên ta cần chứng minh

b

p

b+c +

c

p

3b+c

r

b+c

2

, b

2

b+c +

c2

3b+c+

2bc

p

(b+c)(3b+c) b+

c

2

, c

2

3b+c+

2bc

p

(b+c)(3b+c)

c

2+

bc b+c

, 3bc+c+p 2b

(b+c)(3b+c)

b

b+c +

1

Dob c nên3b+c 2(b+c);suy

2b

p

(b+c)(3b+c)

p

2b b+c

3b

2(b+c)

Lại có

1 +

b b+c

c

3b+c

3b

2(b+c) =

c(b c)

2(b+c)(3b+c)

Từ đây, ta đến

f(a; b; c) f a+c 2; b+

c

2;0

Trường hợp 2.2.2.3b a 52b;khi đó, ta chứng minh

a

p

a+b

a+38c

p

a+b+c

,a2(a+b+c) a2+3 4ac+

9 64c

2 (a+b)

,

64c

2(a+b) +1

4ca(3b a) (đúng)

b

p

b+c+

c

p

c+a

r

b+5

http://www.vnmath.com http://book.vnmath.com

Ngày đăng: 29/04/2021, 04:50

w